Download as pdf or txt
Download as pdf or txt
You are on page 1of 174

QUESTION BANK

3rd Professional, Part-2 QUESTION BANK


2015
[WBUHS]

Compiled and Edited by MEHBOOB-UL ISLAM


BMC
QUESTION BANK

 Contents:
 University Examination Papers of 3rd
Professional Part- II (2008-2014)
 University Examination Papers of 3rd
Professional Part- II, Supplementary (2008-
2014)
 Semester Questions (8th & 9th Sem) of
2014-15 of all 11 colleges of WBUHS (having
batch : 2010-11)

 Groupwise Distribution
of all the questions

 University questions &


Semester questions have
been given separately

COMPILED AND EDITED BY MEHBOOB-UL ISLAM 1


QUESTION BANK

THANKS GIVING
I am thankful to the persons who have provided me with the questions of some supplementary
MB examinations & Semesters of their respective colleges either personally or by posting them
on facebook. So, I would like to thank them:

 Mandira Mitra, BSMCH


 Janmejoy Mondal, BSMCH
 Barnini Ghosh, MMC
 Imrose Ali, NBMC
 Amrita Dey, Kalyani
 Tousif Islam, KPC
 Siddhartha Nandi, IPGMER
 Tithi Debnath, CNMC
 Amurta Banerjee, RG Kar
 Pooja Ghosh, MC,K
 Subhadeep De, NRS

Special thanks to Sunavo, Mainak & Aniruddha (BMC) for helping me in collecting some
semester questions.

A lot of thanks to Md. Salim Usuf, BMC for helping in me in Chapterwise distribution of the
questions.

Thanks to The Almighty for allowing me to compile this Question Bank.

And finally, thank you for deciding to go through this question bank.

I do hope that you will find this question bank helpful in your preparation of 3 rd
Professional, Part - II MBBS (WBUHS). Questions have been presented here groupwise
(particularly, the university papers) so as to give you an orientation regarding the
university(West Bengal University of Health Science) Question pattern of individual
subjects.

May The Almighty bless you

Regards

Mehboob-ul Islam, BMC

(immehboob@gmail.com)

COMPILED AND EDITED BY MEHBOOB-UL ISLAM 2


QUESTION BANK

GLOSSARY:

T/t : Treatment
M/m : Management
C/f : Clinical Features
D/d : Differential diagnoses
Yr/yrs : Years
Wt : weight
Ds: Disease
DM : Diabetes Mellitus
AF : Atrial Fibrillation

 IMPORTANT NOTES

COMPILED AND EDITED BY MEHBOOB-UL ISLAM 3


QUESTION BANK



o
o



o
o



o
o



COMPILED AND EDITED BY MEHBOOB-UL ISLAM 4


QUESTION BANK

COMPILED AND EDITED BY MEHBOOB-UL ISLAM 5


QUESTION BANK

COMPILED AND EDITED BY MEHBOOB-UL ISLAM 6


QUESTION BANK

LONG QUESTION
Gr-A
1. Classify Shock. Discuss the patho-physiology and management of Septic
Shock. (3+6+6)[14]
2. What are the indications of blood transfusion? What are its complications?
What do you understand by massive blood transfusion? (3+8+4) [14, supple]

3. Discuss the assessment of Burn wound. Write in short pathophysiology of


Burn Injury. How would you treat 30% burn in 50 kg body wt female
patient?(3+6+6) [13]

4. What are the normal value of different body electrolytes? What are the
electrolyte changes in a patient of long standing pyloric stenosis and how do
they occur? How do yuou prepare such a patient before elective operation?
(5+5+5) [13, supple]

5. What are the types of Haemorrhage? What are the methods of determining
acute blood loss? How would you treat haemorrhage? (3+6+6) [12]

6. Define shock. Discuss the pathophysiology of septic shock. How would you
manage a patient suffering from septic shock? (2+6+7) [12, supple]

7. Define and classify wound. Discuss the factors influencing wound healing.
Write in brief management of Diabetic ulcer affecting foot, in middle aged
man. (4+5+6) [11]

8. Define shock. What are its different types? Outline the management of a
patient presenting with features of septic shock. (2+4+9) [11, supple]

9. What are the coagulation factors? Write in detail about the mechanism of
Hemostasis. (5+10) [10]

COMPILED AND EDITED BY MEHBOOB-UL ISLAM 7


QUESTION BANK

10. Define claudication. What are the grades of claudication? How will you
manage a case of buerger’s disease with dry gangrene of foot? (2+4+9) [10,
supple]

11. Define and classify wound. Discuss various factors influencing wound healing.
Discuss in brief the management of “Diabetic foot”. (5+5+5) [09]

12. Classify haemorrhage. How will you determine the amount of blood loss?
(3+5+7) [09, supple]

13. Define and classify shock. How will you assess and treat a case of
haemorrhage shock? Mention the complication of blood transfusion.
(2+2+5+3+3) [08]

14. Classify burn. How will you assess and manage a 35 year old woman weighing
60 kgs admitted with 40% burn? (3+5+7) [08, supple]

COMPILED AND EDITED BY MEHBOOB-UL ISLAM 8


QUESTION BANK

LONG QUESTION
Gr-B
1. Describe the clinical features, investigation and management of
carcinoma of Stomach. (4+4+7) [14]

2. A 50 years male patient presents with bleeding per rectum. How will you
investigate and manage the patient. (7+8) [14]

3. What are the diseases of umbilicus? What are the presentation and
treatment of Meckel’s diverticulum? (3+7+5) [14, supple]

4. How will you assess a breast lump in an elderly woman? Outline the
treatment option of an early breast cancer in an woman of 50 yrs. (6+9)
[14, supple]

5. Classify colonic tumours. How will you manage a 60 years old man
presenting with fresh bleeding per rectum. (5+10) [13]

6. Describe the clinical features, diagnosis and management of


Choledocholithiasis. (4+4+7) [13]

7. What are the causes of lump in right iliac fossa? Outline the diagnosis
and management of appendicular lump. (3+6+6) [13, supple]

8. A 30 yr old lady presented with severe abdominal pain and shock. Discuss
the differential diagnosis and outline the management. (6+9) [13, supple]

9. Middle aged patient presented with a big tense cystic lump in the upper
abdomen following an attack of acute abdomen. How would you investigate
the patient and plan the management? (8+7) [12]

10. What are the causes of benign biliary strictures? Discuss the
management of retained stone in common bile duct. (5+10) [12]

11. How would you proceed to investigate and manage a 50 year old man
presenting with a lump in left iliac region and irregular bowel habit. (6+9)
[12, supple]

COMPILED AND EDITED BY MEHBOOB-UL ISLAM 9


QUESTION BANK

12. A male patient presented with irreducible inguinal hernia on the right
side of 6 hrs duration. How would you proceed to manage the patient?
(15) [12, supple]

13. A forty five year old lady presents with acute upper abdominal pain,
discuss the differential diagnosis and management of such apatient. (7+8)
[11]

14. Enumerate the causes of intestinal obstruction in an infant. Write the


clinical features, investigations and management of intussusception in a 7
months old child. (3+4+3+5) [11]

15. Describe the clinical features, investigations and management of acute


pancreatitis. (5+5+5) [11, supple]

16. What are the causes of lump in RIF in a male patient of 40 years old?
How do you investigate and treat such a patient? (5+5+5) [11, supple]

17. What are all the causes of upper GI bleed? How will you manage acute
variceal bleeding? (8+7) [10]

18. What are the causes of obstructive jaundice? Write about the
management of carcinoma head of pancreas. (5+10) [10]

19. Enumerate the different diagnosis of painless fresh bleeding per rectum.
Plan the investigation and the treatment of carcinoma of sigmoid colon.
(3+5+7) [10, supple]

20. How ill you evaluate, grade and manage a case of blunt splenic trauma?
(3+4+8) [10, supple]

21. A middle aged male patient presents with an epigastric pain. Discuss the
differential diagnosis. How would you investigate to come to a diagnosis?
(8+7) [09]

22. What are the causes of weeping umbilicus? Discuss the problems related
to vitelline intestinal duct and their remedy. (5+5+5) [09]

23. Give an account of pathogenesis, clinical features and management of


acute pancreatitis. (5+5+5) [09, supple]

COMPILED AND EDITED BY MEHBOOB-UL ISLAM 10


QUESTION BANK

24. Define and classify intestinal obstruction. How will you diagnose and treat
small intestinal obstruction? (5+5+5) [09, supple]

25. Enumerate the causes of bleeding per rectum. Mention how it is


diagnosed. Outline the management for bleeding haemorrhoid. (5+5+5)
[08]

26. What are the causes of obstructive jaundice. How do you establish the
diagnosis. Discuss the various option in the management of
choledocholithiasis. (5+5+5) [08]

27. Enumerate the causes of gastro-intestinal bleeding. Discuss how a patient


with this should be diagnosed and managed? (3+4+8) [08, supple]

28. A 40 yr old male patient has come to the emergency with acute pain in
the right hypochondrium. How will you make a clinical diagnosis? Outline
the treatment strategy in such a patient. (5+10) [08, supple]

COMPILED AND EDITED BY MEHBOOB-UL ISLAM 11


QUESTION BANK

SHORT NOTES
Gr-C
1. Breast Biopsy [14]

2. Incarcerated Hernia [14]

3. Blood Substitute [14]

4. Volvulus Neonatoram [14]

5. Amoebic Liver Abscess [14]

6. CT scan [14, supple]

7. Pheochromocytoma [14, supple]

8. Pleomorphic adenoma [14, supple]

9. Gall stone ileus [14, supple]

10. Femoral hernia [14, supple]

11. Causes and treatment of metabolic acidosis [13]

12. Venous ulcer [13]

13. Fournier’s gangrene [13]

14. Anorectal malformation [13]

15. Torticollis [13]

16. Complication of splenectomy [13, supple]

17. Pilonidal sinus [13, supple]

18. Intercostal drain [13, supple]

19. Core needle biopsy [13, supple]

COMPILED AND EDITED BY MEHBOOB-UL ISLAM 12


QUESTION BANK

20. Post-operative pain management [12]

21. Creating Pneumoperitoneum in Lap. Surgery (procedures only) [12]

22. Burst Abdomen [12]

23. Decubitus ulcer [12]

24. TPN [12, supple]

25. Metabolic acidosis [12, supple]

26. Prophylactic antibiotics [12, supple]

27. Lipoma [12, supple]

28. Pre-operative preparation of a case of obstructive jaundice [11]

29. Epigastric hernia [11]

30. D.V.T. [11]

31. Active immunization against Tetanus [11]

32. Pre-op preparation of a patient of pyloric stenosis [11, supple]

33. Marjolin’s ulcer [11, supple]

34. Hyponatremia [11, supple]

35. Blood fractions [11, supple]

36. Hemangioma [10]

37. Carotid body tumor [10]

38. Brachial sinus [10]

39. Carcinoid tumor [10]

40. Melanoma [10, supple]

41. Blood substitute [10, supple]

COMPILED AND EDITED BY MEHBOOB-UL ISLAM 13


QUESTION BANK

42. Trophic ulcer [10, supple]

43. Systemic inflammatory response syndrome [10, supple]

44. Arteriovenous fistula [09]

45. Basal cell carcinoma [09]

46. Preoperative preparation of a patient of Pyloric stone [09]

47. Auto transfusion [09]

48. Oesophageal varices [09, supple]

49. OPSI [09, supple]

50. Intermittent claudication [09, supple]

51. Methods of sterilization [08]

52. Biochemical abnormality in pyloric stenosis [08]

53. Universal precaution [08]

54. Nipple discharge [08]

55. Fibroadenoma of the breast [08, supple]

56. Meckel’s Diverticulum [08, supple]

57. Keloid [08, supple]

58. Anal fissure [08, supple]

COMPILED AND EDITED BY MEHBOOB-UL ISLAM 14


QUESTION BANK

BRIEF NOTES
Gr-D
1. Fractures occurring due to fall on outstretched hand [14]

2. Osteochondroma [14]

3. Greenstick fracture [14]

4. Fracture of patella [14] , mechanism of [11]

5. Spina bifida [14],[09]

6. Compound fracture [14, supple]

7. Volkmann’s ischaemic Contracture [13], [10, supple]

8. Ewing’s tumour [13],[11] , sarcoma [10], [14, supple]

9. Core needle biopsy [13]

10. Colles fracture [13], [08, supple]

11. Bone graft [13]

12. CTEV [13, supple]

13. Monteggia fracture [13, supple]

14. Non union of fracture [13, supple]

15. External fixation [13, supple]

16. Recurrent dislocation of shoulder [13, supple]

17. Fracture clavicle [12]

18. Tennis Elbow [12]

COMPILED AND EDITED BY MEHBOOB-UL ISLAM 15


QUESTION BANK

19. Supra condylar fracture of Humerus [12]

20. Dupytren’s contracture [12], [10, supple]

21. Ruptured Tendoachilles [12]

22. Mallet finger [12]

23. Pyogenic Osteomyelitis. [12]

24. Compartment syndrome [12, supple]

25. Indications of limb amputation [12, supple]

26. Fracture olecranon [12, supple]

27. Slipped disc [12, supple]

28. Radiological features of osteosarcoma [12, supple]

29. Trigger finger [11]

30. Brodie’s Abscess [11], [14, supple]

31. Carpal tunnel syndrome [11],[09], [13, supple], [14, supple]

32. Shoulder dislocation [11]

33. Telepes equinovarus [11, supple]

34. SP Nail [11, supple]

35. Codman’s triangle [11, supple]

36. Exostosis [10]

37. Brown tumor [10]

38. Spondylolisthesis [10]

39. Bone scan [10]

40. TB spine [10], clinical feature [08, supple]

COMPILED AND EDITED BY MEHBOOB-UL ISLAM 16


QUESTION BANK

41. TB hip joint [10, supple]

42. Club foot [10, supple]

43. Giant cell tumor [10, supple]

44. Avascular necrosis of femoral head [10, supple]

45. Myositis ossificans [09]

46. Pathological facture [09]

47. Fracture neck femur-types and complication [09]

48. Aetiopathogenesis of acute osteomyelitis [09]

49. Indications for amputation [09, supple]

50. Non union of closed fracture [09, supple]

51. Paget’s disease of bone [09, supple]

52. Management of osteosarcoma [09, supple]

53. Tension band wiring [09, supple]

54. Frozen shoulder [08], [11, supple], [14, supple]

55. Complication of supracondylar fracture of humerus [08], [09, supple]

56. Sequestrum [08], [11, supple], [12, supple]

57. Volkmann’s contracture [08]

58. De Quervan’s disease [08], [11, supple]

59. ….problems of spinal paraplegia [08]

60. Fracture of patella [08, supple]

61. Bone cyst [08, supple]

62. Gibbus [08, supple]

COMPILED AND EDITED BY MEHBOOB-UL ISLAM 17


QUESTION BANK

63. Stress fracture [08, supple]

COMPILED AND EDITED BY MEHBOOB-UL ISLAM 18


QUESTION BANK

COMPILED AND EDITED BY MEHBOOB-UL ISLAM 19


QUESTION BANK

LONG QUESTIONS
Gr-A
1. Discuss the clinical features, investigation and treatment of Thyrotoxicosis.
(5+5+5) [14]

2. Discuss causes, investigations and treatment of haematuria. (4+3+8) [14,


supple]

3. Define Hydronephrosis. Discuss the cause and management of unilateral


hydronephrosis. (2+5+8) [13]

4. How do you classify goiter? Give an outline of investigations and management


of a solitary nodular goiter. (5+5+5) [13, supple]

5. Describe Thyroid Neoplasms. Write clinical features, investigation and


management of papillary carcinoma of the Thyroid gland(A lady of 25 years
old) (3+4+3+5) [12]

6. A 40 years old gentleman presented with bilateral knobby Renal lump in the
abdomen. How do you investigate and treat such a patient (Operational
details not required). (7+8) [12]

7. What are the different types of nipple discharges with their clinical
importances?how would you manage stage I carcinoma breast in a lady aged
40 years. (6+9) [12, supple]

8. A 65 year old man presented to the emergency with acute retention of urine.
How would you investigate and manage the patient? (7+8) [12, supple]

9. Classify carcinoma of breast. How will you investigate and manage a case of
early Carcinoma of Breast in a 40 year old lady. (4+5+6) [11]

10. Classify Renal neoplasms. How will you diagnose and manage a case of renal
carcinoma? (4+5+6) [11]

11. Describe the management of a lady of 35 years presenting with toxic


multinodular goiter. (6+9) [11, supple]

COMPILED AND EDITED BY MEHBOOB-UL ISLAM 20


QUESTION BANK

12. A 70 yr old man presents with acute retention of urine. How would you
investigate the patient? Outline the management in brief the benign
hyperplasia of prostste. (7+8) [11, supple]

13. What are the functions of parathormone? Write in detail about clinical
features, investigation and treatment of hyperparathyroidism. (3+4+4+4)
[10]

14. Give differential diagnosis of Scrotal Swelling. Write in detail about


management of testicular tumor (5+10) [10]

15. Classify thyroid malignancies. How will you manage a case of follicular
carcinoma of thyroid? (5+10) [10, supple]

16. A 40 year old patient presents with hematuria. Enumerate the differential
diagnosis, plan the investigations and treatment. (4+7+4) [10, supple]

17. A 35 year old lady presents with a Solitary thyroid nodule in right lobe. How
would you come to diagnosis and manage such a patient? (8+7) [09]

18. Classify kidney tumours. Mention the different modes of presentation of


Renal Adeno Carcinoma. Outline the management of such a patient. (3+5+7)
[09]

19. Define hydronephrosis. What are the causes of unilateral hydronephrosis?


Discuss management of stone of a patient in middle 3rd of ureter. (2+4+9)
[09, supple]

20. A 55 yr old lady presents with a lump in upper and outer quadrant of right
breast. Discuss briefly the management of such a patient. (15) [09, supple]

21. Enumerate the causes of haematuria. How will you confirm the diagnosis?
What will you do for a patient diagnosed to have carcinoma of urinary
bladder? (5+5+5) [08]

22. Discuss the pathogenesis of multinodular goiter. Mention the complications


of such a goiter. How do you manage such a patient? (5+5+5) [08]

23. Enumerate the causes of acute retention of urine. What are the pathological
changes associated with prostatic hypertrophy? How will you manage such a
patient? (5+5+5) [08, supple]

COMPILED AND EDITED BY MEHBOOB-UL ISLAM 21


QUESTION BANK

24. Discuss the clinical features of pheochromocytoma. How do you diagnose this
condition? What will you do to manage it? (5+5+5) [08, supple]

COMPILED AND EDITED BY MEHBOOB-UL ISLAM 22


QUESTION BANK

LONG QUESTIONS
Gr-B
1. Describe the pathophysiology of BHP. Mention the medical and Surgical
management of BHP. (6+4+5) [14]

2. Classify testicular tumours. Discuss investigation necessary to plan the


treatment for a suspected testicular tumour. What are the treatment
options available. (5+5+5) [14]

3. What are the different types of renal calculi? Discuss the clinical
features and management of renal calculi. (3+4+8) [14, supple]

4. What are the clinical features of primary hyperparathyroidism? Discuss


the investigation and management of primary hyperparathyroidism.
(6+5+4) [14, supple]

5. Describe lymphatic drainage of breast. Mention the risk factors of breast


carcinoma. How to manage a 52 year old female patient with locally
advanced breast carcinoma? (4+3+8) [13]

6. A 50 year old gentleman presented with painless hematuria. What may be


the possible causes? How would you investigate the case? Give an outline
of the management. (5+5+5) [13]

7. Classify renal neoplasms. Write clinical features, investigation and


management of renal cell Ca in a 40 yr old male patient. (3+5+3+4) [13,
supple]

8. Discuss the clinical features of phaeochromocytoma. How will you diagnose


this condition? Give an outline of management. (5+5+5) [13, supple]

Before 2013, Gr-B consisted of Short Notes (each carrying 5 marks)

1. Breast Biopsies [12]

2. Causes of Haematuria [12]

COMPILED AND EDITED BY MEHBOOB-UL ISLAM 23


QUESTION BANK

3. Antegrade Pyelography [12]

4. Stress Gastritis [12]

5. PSA [12]

6. CABG [12, supple]

7. Tracheostomy [12, supple]

8. Extradural haemorrhage [12, supple]

9. DVT [12, supple]

10. Variants of melanoma [12, supple]

11. Flail chest [11]

12. Post operative pyrexia [11]

13. Brain death [11]

14. Split thickness skin graft [11]

15. Omphalocoele [11]

16. Stove in chest [11, supple]

17. Paraphimosis [11, supple]

18. Bedsore [11, supple]

19. Ulnar nerve injury [11, supple]

20. Hydrocephalus [11, supple]

21. Ectopiavesicae [10]

22. Neurofibromatosis [10]

23. Paget’s disease of nipple [10]

24. Fistula in ano [10]

COMPILED AND EDITED BY MEHBOOB-UL ISLAM 24


QUESTION BANK

25. Varicocele [10]

26. Mixed salivary tumor [10, supple]

27. Sentinel lymph node biopsy [10, supple]

28. Gas gangrene [10, supple]

29. Primary hyperparathyroidism [10, supple]

30. Hypersplenism [10, supple]

31. Salivary calculi [09]

32. Fournier’s Gangrene [09]

33. M.E.N. syndrome [09]

34. Complication of undescended testis [09]

35. Hypospadias [09, supple]

36. Marjolin’s ulcer [09, supple]

37. Collar stud abscess [09, supple]

38. Venous ulcer [09, supple]

39. Management of Hirschprung’s disease [09, supple]

40. Venous ulcer [08]

41. Epididymal cyst [08]

42. Tetany [08]

43. Thyroglossal cyst [08]

44. Dermoid cyst [08]

45. Carcinoid tumor [08, supple]

46. Raynaud’s phenomenon [08, supple]

COMPILED AND EDITED BY MEHBOOB-UL ISLAM 25


QUESTION BANK

47. Ranula [08, supple]

48. Abdominal compartment syndrome [08, supple]

49. Desmoid tumor [08, supple]

COMPILED AND EDITED BY MEHBOOB-UL ISLAM 26


QUESTION BANK

SHORT NOTES
Gr-C
1. Paget’s disease of Nipple [14]

2. Electric burn [14]

3. Dentigerous cyst [14]

4. Lucid interval [14],[12]

5. Fistula in ano [14]

6. Parotid abscess [14, supple]

7. PDA [14, supple]

8. Chordee [14, supple]

9. QUART [14, supple]

10. Chronic subdural hematoma [14, supple]

11. Cleft lip [13]

12. Thyroglossal cyst [13]

13. Spinal Anaesthesia [13]

14. Types of Skin graft [13]

15. Role of ERCP in obstructive jaundice [13]

16. Posterior urethral valve [13, supple]

17. Hutchinson’s pupil [13, supple]

18. PCNL [13, supple]

19. Type of renal stone [13, supple]

COMPILED AND EDITED BY MEHBOOB-UL ISLAM 27


QUESTION BANK

20. Paraphimosis [12]

21. Chest Drain [12]

22. Torsion of Testes [12]

23. Tissue expansion [12]

24. Bladder changes in BHP [12, supple]

25. Epididymal cyst [12, supple]

26. ESWL [12, supple]

27. Nephroblastoma [12, supple]

28. Hematuria – causes [12, supple]

29. Retrosternal Goiter [11]

30. Parotid abscess [11]

31. Alvardo score of Acute Pancreatitis [11]

32. T.U.R.P [11]

33. Oxalate stone [11]

34. Blood discharge per nipple [11, supple]

35. Hydronephrosis [11, supple]

36. Parotid fistula [11, supple]

37. Hypospadias [11, supple]

38. Oschner Sherren Regime [11, supple]

39. Subdural hematoma [10]

40. Muscle relaxant [10]

41. I131scan [10]

COMPILED AND EDITED BY MEHBOOB-UL ISLAM 28


QUESTION BANK

42. Congenital hypertrophic pyloric stenosis [10]

43. Lumbar puncture [10]

44. Meconium ileus [10, supple]

45. Hydrocephalus [10, supple]

46. Intravenous anesthetics [10, supple]

47. Post burn contracture [10, supple]

48. Fast [10, supple]

49. Anorectal malformation [09]

50. Extrduralhaematoma [09]

51. Cardiopulmonary Resuscitation(CPR) [09]

52. MRI [09]

53. Complications of Radiotherapy [09], [09, supple]

54. Cleft palate [09, supple]

55. Target FNAC [09, supple]

56. Regional anesthesia [09, supple]

57. Skin grafting [08]

58. Clinical features of Hirsprung’s disease [08, supple]

59. Cleft lip management in children [08, supple]

60. Tetanus prophylaxis [08, supple]

61. Diagnostic use of ultrasound [08, supple]

62. Empyema thoracis [08, supple], [09, supple], [13, supple]

COMPILED AND EDITED BY MEHBOOB-UL ISLAM 29


QUESTION BANK

BRIEF NOTES
Gr-D
1. Penile carcinoma [14]

2. Muscle Relaxant [14]

3. Flail Chest [14], [08]

4. Epidural Anaesthesia [14]

5. Compartment syndrome [14]

6. Local anaesthesia in inguinal hernia surgery [14, supple]

7. Venous ulcer [14, supple]

8. Transluminal USG [14, supple]

9. Radioactive iodine [14, supple]

10. Oral submucous fibrosis [13]

11. Wax bath [13],[11]

12. Subdural haematoma [13]

13. Intussusceptions [13]

14. Marjolin’s ulcer [13]

15. Causes of scrotal swelling [13, supple]

16. Pressure sore [13, supple]

17. Hypospadias [13, supple]

18. Keloid [13, supple]

19. Tension Pneumothorax [13, supple]

COMPILED AND EDITED BY MEHBOOB-UL ISLAM 30


QUESTION BANK

20. Anaesthesia Monitoring devices [12]

21. Radiotherapy in treatment of Carcinoma Breast [12]

22. Ameloblastoma [12]

23. Transluminal USG [12]

24. Short Wave Diathermy [12],[09]

25. Epidural anaesthesia [12, supple]

26. Dentigerous cyst[12, supple], [14, supple]

27. Small bowel enema [12, supple]

28. Frozen shoulder- physiotherapy [12, supple]

29. Epulis [11], [10, supple]

30. MRI scan in surgery [11]

31. Radiation Dermatitis [11]

32. Spinal Anaesthesia [11]

33. Cervical traction [11, supple]

34. USG for hepatobiliary disease [11, supple]

35. Local anaesthesia [11, supple]

36. Brachytherapy [11, supple], [12, supple]

37. Ludwig’s angina [10]

38. Meningo-mylocele [10]

39. Empymathoracis [10]

40. Referred pain [10]

41. Hospice [10, supple]

COMPILED AND EDITED BY MEHBOOB-UL ISLAM 31


QUESTION BANK

42. GCS [10, supple]

43. Differential diagnosis of intracranial space occupying lesions [10,


supple]

44. Patent ductusarteriosus [10],[08]

45. Fat embolism [09]

46. Odontomes [09]

47. Tension pneumothorax [09], [10, supple]

48. Hypokalemia [09]

49. Mammography [09, supple]

50. Nerve injury [09, supple]

51. Ultrasonic therapy [09, supple]

52. Adamantinoma [09, supple], [11, supple]

53. Solitary thyroid nodule [09, supple]

54. Dental cyst [08]

55. Glasgow coma scale [08]

56. Therapeutic use of ultrasound [08]

57. Epulis [08, supple]

58. Wax bath [08, supple]

59. Extradural hematoma [08, supple]

60. Care of a paraplegic patient [08, supple]

61. Axial flap [08, supple]

COMPILED AND EDITED BY MEHBOOB-UL ISLAM 32


QUESTION BANK

COMPILED AND EDITED BY MEHBOOB-UL ISLAM 33


QUESTION BANK

LONG QUESTIONS
(GENERAL)
1. Define and classify shock. Discuss the pathogenesis and management of a patient
with a septic shock. (2+3+10) (MC,K) [8th Sem]

2. Define shock. What are the clinical features of septic shock? Outline the
management of septic shock. (3+5+7) (RG Kar) [8th Sem]

3. Give the detailed protocol for initial resuscitation for a moderately severe burn
patient (flame burn).how would you evaluate and manage associated inhalation
injury, if any? Discuss the role of topical antibiotics. (5+5+5) (IPGMER) [8th Sem]

4. How is the acid balance maintained in the body normally, with special reference to
the role of lung and kidney? What are the common causes of metabolic acidosis and
its management? What is anion gap and what is its clinical significance? (5+5+5)
(IPGMER) [8th Sem]

5. Classify shock. describe the pathology and management of septic shock. (3+6+6)
(BSMC) [8th Sem]

6. Give a brief account of electrolyte balance in the human body. What are the
electrolyte changes in a patient of long standing pyloric stenosis? How do you
prepare such a patient of long standing pyloric stenosis? How do you prepare such a
patient before an elective operation? (6+4+5) (MMC) [8th Sem]

7. What is tetanus ? describe clinical features and management. (2+6+7) (BSMC) [9th
sem] {paper 1}

8. Enumerrate the diferent blood fractions and discuss the indications of their use.
Discuss in brief the complication of whole blood transfusion. (10+5) (RG Kar) [9th
sem] {Paper 1}

9. Classify burns. What are the immediate and delayed complications of burn injury?
How will you manage a patient with 40% burns and 60 kg body wt. (5+5+5) (NRS)
[9th sem]

COMPILED AND EDITED BY MEHBOOB-UL ISLAM 34


QUESTION BANK

10. Classify wounds. Describe the pathophysiology of wound healing. What are the
common causes of burst abdomen? (4+6+5) (NRS) [9th sem]

11. Define and classify Surgical Site Infections. Enumerate the risk factors for the
development of surgical site infections. Discuss the role of antibiotic prophylaxis in
respect different class of surgical wounds. (2.5+2.5+5+5) (IPGMER) [9th sem]

12. Define burns. Mention different types of burns. Describe management of 40% 2nd
degree burn. (3+5+7) (CNMC) [9th Sem] {Paper 1}

13. How will you assess the severity of thermal burns? How will you manage such a
patient in emergency? What are the complications of burn? (4+6+5) (CMC) [9th Sem]
{Paper 1}
14. Mention the complication of blood transfusion. What are the blood substitutes?
(10+5) (Kalyani) [9th Sem] {Paper 1}

COMPILED AND EDITED BY MEHBOOB-UL ISLAM 35


QUESTION BANK

LONG QUESTIONS
(DIRECT QUESTIONS)
1. Enumerate the causes and clinical features of acute intestinal obstruction. Discuss
the management of acute intestinal obstruction. Discuss the management of
intestinal obstruction due to a malignant tumour in the recto sigmoid junction.
(6+3+6) (MC,K) [8th Sem]

2. Enumerate the causes of obstructive jaundice. Discuss the clinical features,


investigations and pre operative management of a patient with CA head of
Pancreas. (5+10) (MC,K) [8th Sem]

3. Enumerate different thyroid swellings. Discuss the treatment and follow up of


differentiated thyroid carcinoma. (6+9) (MC,K) [8th Sem]

4. Describe the presenting features, diagnosis and outline the management in a


patient of Carcinoma stomach. (5+5+5) (NRS) [8th Sem]

5. Describe the clinical presentation and management of a case of Gastric ulcer.


(10+10) (CNMC) [8th Sem]

6. Enumerate the causes of Breast lump. How will you manage a case of single lump in
right breast.(CNMC) [8th Sem]

7. What is meant by appendicular lump? What are the clinical features and outcomes?
Outline the management of appendicular lump. (2+7+6) (RG Kar) [8th Sem]

8. Describe the clinical features, investigation and management of CA stomach.


(4+4+7) (RG Kar) [8th Sem]

9. An anorexic, asthenic and anaemic patient was persistent recurrent non-bilious


vomiting with visible peristalsis from left to right epigastrium. How will you proceed
to diagnose and manage the patient with particular reference to the preparation of
patient for operation. (5+5+5) (IPGMER) [8th Sem]

10. How will you approach a solitary thyroid nodule? (15) (KPC) [8th Sem]

COMPILED AND EDITED BY MEHBOOB-UL ISLAM 36


QUESTION BANK

11. Enumerate the clinical features of, pathology and management of RCC. (4+3+8)
(KPC) [8th Sem]

12. Classify shock. Describe the etiopathogeness and management of hypovolemic


shock.(7+8) (BMC) [8th Sem]

13. What are the peritoneal spaces? Describe the management of case of generalized
peritonitis following duodenal ulcer perforation. (5+20) (BMC) [8th Sem]

14. Discuss the clinical features of Phaeochromocytoma. How will you diagnose this
condition. Give an outline of management? (5+5+5) (MMC) [8th Sem]

15. Classify renal injury. Discuss clinical features and management of a patient having
injury in the left kidney following blunt trauma in left loin. (4+5+6) (MMC) [8th
Sem]

16. What is hydatid cyst? Give a short account of the life cycle of the parasite.write in
short the pathology and management of hydatid cyst. (1+4+10) (MMC) [8th Sem]

17. Classify goiters. How will you investigate and treat a 60 year old male presenting
with right sided thyroid swelling? (5+4+6) (NBMC) [8th Sem]

18. Define intestinal obstruction. Enumerate the common causes of intestinal


obstruction in adult. Briefly discuss the diagnosis and management of sigmoid
volvulus. (2+4+4+5) (NBMC) [8th Sem]

19. What is early breast carcinoma? How will you manage it? (5+10) (Kalyani) [8th Sem]

20. Enumerate the causes of obstructive jaundice. How will you investigate? Describe
the preoperative management of such a jaundiced patient. (5+5+5) (Kalyani) [8th
Sem]

21. Define hydronephrosis. Describe causes and its management. (15) (BSMC) [9th sem]
{paper 2}

22. Classify thyroid carcinoma. Describe the management of papillary carcinoma of


thyroid. How do you follow up a postoperative patient with papillary thyroid
carcinoma after discharge (3+8+4) (RG Kar) [9th sem] {Paper 2}

COMPILED AND EDITED BY MEHBOOB-UL ISLAM 37


QUESTION BANK

23. Discuss the clinical features, investigation and management of acute cholecystitis.
(4+4+7) (RG Kar) [9th sem] {Paper 2}

24. What are the common causes of obstructive jaundice? How will you investigate a
patient with obstructive jaundice? What are the surgical management of
choledocholithiasis? (5+5+5) (NRS) [9th sem]

25. Classify different types of Thyrotoxicosis based on etiology. What are the reasons
for doing a total (or near total) thyroidectomy in large diffuse goiter with
thyrotoxicosis? How would you prepare such a patient for surgery? Enumerate the
post-thyroidectomy complication and the brief outline of their management(outline
only). (2.5+2.5+5+2.5+2.5) (IPGMER) [9th sem]

26. Mention the causes of acute pancreatitis. How will you establish the diagnosis?
Discuss the management of acute gall stone pancreatitis. (5+5+5) (CNMC) [9th Sem]
{Paper 1}

27. Mention the etiopathogenesisof urinary bladder stones. Describe the symptoms of
stone in lower 3rd of ureter and its management. (5+5+5) (CNMC) [9th Sem] {Paper
2}

28. Classify breast cancer. Write the clinical features, investigation and management
of stage –II Breast cancer. (3+4+3+5) (CNMC) [9th Sem] {Paper 2}

29. Define cyst. Classify cyst with examples. Discuss the management of a surgically
relevant parasitic cyst. (2+6+7) (CMC) [9th Sem] {Paper 1}

30. Classify carcinoma stomach. Mention risk factors. Discuss investigations. (7+4+4)
(CMC) [9th Sem] {Paper 1}

31. Mention the clinical features, types, management and complications of choledochal
cyst. (3+5+4+3) (CMC) [9th Sem] {Paper 1}

32. Define hydronephrosis. Discuss the causes of hydronephrosis. Discuss the


management of PUJ obstruction. (2+6+7) (CMC) [9th Sem] {Paper 2}

33. Describe the types of bladder and urethral injuries with clinical features and
management. (3+5+7) (CMC) [9th Sem] {Paper 2}

COMPILED AND EDITED BY MEHBOOB-UL ISLAM 38


QUESTION BANK

34. Enumerate the different adrenal swellings with clinical features and investigations
to be done for diagnosis. How do you prepare a patient of phaeochromocytoma for
surgery? (4+6+5) (CMC) [9th Sem] {Paper 2}

35. Discuss the types with management of different thyroid malignancies. (15) (CMC)
[9th Sem] {Paper 2}

36. Describe and classify wounds. Discuss various factors influencing wound healing.
Describe in brief M/m of a “diabetic foot “. (5+5+5) (BMC) [9th Sem] {Paper 1}

37. Classify benign breast ds. Briefly describe the M/m of a case of early breast
cancer. (5+10) (BMC) [9th Sem] {Paper 2}

38. Discuss pathogenesis of tetanus. Write briefly the clinical features and T/t of an
established case of tetanus. (5+5+5) (MMC) [9th Sem]

39. Describe the etiopathology of acute extradural hematoma. Mention the symptoms
and signs. Outline the principle of M/m of it. (5+5+5) (MMC) [9th Sem]

40. Mentions the complications of hernia. Discuss the C/f & M/m of strangulated
inguinal hernia. (5+5+5) (NBMC) [9th Sem]

41. What is locally advanced breast carcinoma? Outline the M/m of such a Case. (7+8)
(Kalyani) [9th Sem] {Paper 2}

42. Discuss the types, mechanisms, presentations and management of rupture urethra.
(2+3+5+5) (Kalyani) [9th Sem] {Paper 2}

COMPILED AND EDITED BY MEHBOOB-UL ISLAM 39


QUESTION BANK

LONG QUESTIONS
(CLINICAL CASES)
1. Classify Burn. A patient of 60 years is presented to you with a history of 50% burn
two hours back. How will you manage the patient? Describe the management
protocol for inhalation burn injury. (3+9+3) (NRS) [8th Sem]

2. Describe the causes, investigations and treatment of a 40 year old male patient
presenting with hematuria. (5+5+5) (NRS) [8th Sem]

3. A female patient of age 35 years is presented with a lump in umbilical region. How
will you manage the patient? (20) (CNMC) [8th Sem]
4. Classify salivary gland tumours. How will you manage a case of parotid swelling in a
50 year aged male patient? (CNMC) [8th Sem]

5. A male patient of 40 years age presented with an epigastric lump. Mention the
probable causes and the management. (CNMC) [8th Sem]

6. What are the causes of nipple discharge? How will you manage a case of advanced
carcinoma of breast in a 45 years aged female. (CNMC) [8th Sem]

7. A 40 year old man presents with severe pain starting over the epigastrium and
spreading all over. How would you proceed with the diagnosis and management?
(8+7) (RG Kar) [8th Sem]

8. A 40 year male alcoholic was admitted with severe pain abdomen with radiation to
the back and relief with stooping. Physical examination was grossly insignificant.
How will you proceed to diagnose, asses the severity and manage the patient.
(5+5+5) (IPGMER) [8th Sem]

9. What are the causes of a painless breast lump in an elderly lady of 65 years of age?
How would you investigate and manage such a case? (5+10+10) (BMC) [8th Sem]

10. A 50 year old female patient presented with obstructive jaundice. How will you
investigate the case to reach a diagnosis? (8+7) (BSMC) [8th Sem]

COMPILED AND EDITED BY MEHBOOB-UL ISLAM 40


QUESTION BANK

11. A 10 year old boy brought to emergency in shock stage with history of blunt trauma
in left upper quadrant of abdomen. How will you prefer to treat the patient and
follow up subsequently? (10+5) (NBMC) [8th Sem]

12. A 60 years male patient presented with haematuria, how do you investigate the
case to come to a diagnosis. How will you manage the case. (8+7) (BSMC) [9th sem]
{paper 1}

13. A 30 years female patient presented with acute lower abdomen pain. How will you
investigate and manage the patient. (8+7) (BSMC) [9th sem] {paper 1}

14. A 40 year’s women presented with solitary thyroid nodule. How will you manage the
case? (15) (BSMC) [9th sem] {paper 2}

15. Describe the important clinical features of a 50 years old male patient with gastric
outlet obstruction? How do you investigate the patient to come to a diagnosis?
Describe the management of gastric carcinoma in antrum? (4+4+7) (RG Kar) [9th
sem] {Paper 1}

16. Enumerate the common causes of bleeding per rectum in a 40 years old male
patient. How do you examine and investigate the patient to come to a diagnosis?
Outline the management of Carcinoma of upper third of rectum without any
metastasis. (3+4+4+4) (RG Kar) [9th sem] {Paper 1}

17. What are the causes of chronic retention of urine in a 60 year old male patient?
How can you differentiate anuria from retention of urine clinically ? enumerate
three common investigations we do for a patient with chronic retention of urine.
Describe the medical and surgical management of benign hypertrophy of Prostate.
(2+2+3+2+6) (RG Kar) [9th sem] {Paper 2}

18. A 45 yr old gentleman presented with intractable anemia and a painless lump in
right iliac fossa of 3 month’s duration. How would you investigate, stage, assess the
operability and plan a comprehensive management of the problem (brief putline
only) (5+2.5+2.5+5) (IPGMER) [9th sem]

19. A 55 year male, chronic alcoholic complaints of recurrent episode of severe,


agonizing acute upper abdominal pain persisting for several hours, radiating to the
back, a little relief with stooping, subsiding with medication only, interfering with
his daily activities. How would you investigate to confirm the diagnosis,
prognosticate and manage?(brief outline only) (5+5+5) (IPGMER) [9th sem]

COMPILED AND EDITED BY MEHBOOB-UL ISLAM 41


QUESTION BANK

20. A 55 yr old lady presented with a 5cm×5cm rounded lump of 3 months duration in
upper outer quadrant of right breast with overlying skin oedema and 1 cm diameter
ulceration and a few palpable matted axillary nodes. How would you stage,
investigate to confirm the diagnosis, prognosticate and manage (brief outline only)?
(2.5+5+2.5+5) (IPGMER) [9th sem]

21. How would you proceed to investigate a 65 yr old gentleman presenting with
painless hematuria for 3 wks duration. Give the outline of management of acute
retention in this elderly gentleman. Give the brief outline of the options of
management of RCC. (5+5+5) (IPGMER) [9th sem]

22. A 35 yrs old female patient presents with abdomen pain, vomiting and fever. How
will you manage the patient. (15) (CNMC) [9th Sem] {Paper 1}

23. A 50 yrs old female patient with lump in right lumbar region presents with
hematuria. What are the causes and how will you investigate the patient? Give an
outline of management of RCC. (5+5+5) (CNMC) [9th Sem] {Paper 2}

24. What are the causes of fresh bleeding per rectum? How will you proceed to
investigate a case of rectosigmoid malignancy? (5+3+7) (KPC) [9th Sem] {Paper 1}

25. A young man of 50 kg body weight come to you with 40% burns. How will you assess
and treat and what are the complications? (3+7+5) (KPC) [9th Sem] {Paper 1}

26. Enumerate the causes of retention of urine. How will you manage a case of
retention of urine in a 60 yr old man? (5+10) (KPC) [9th Sem] {Paper 2}

27. Mention the causes of nipple discharge. How will you investigate and treat a case of
fibrocystic disease of breast? (5+5+5) (KPC) [9th Sem] {Paper 2}

28. Outlin ethe M/m of a 50 yr old female patient with 65 kg body wt, who has
presented with 30% B.S.A. burn. (15) (BMC) [9th Sem] {Paper 1}

29. What are the causes of Right iliac fossa pain? Describe the M/m of acute
appendicitis. (7+8) (BMC) [9th Sem] {Paper 1}

30. What are the causes of acute retention of urine in an elderly male patient. Plan the
investigation & T/t in such a case. (3+6+6) (BMC) [9th Sem] {Paper 2}

COMPILED AND EDITED BY MEHBOOB-UL ISLAM 42


QUESTION BANK

31. Enumerate the causes of lump in right iliac fossa and discuss the differential
diagnosis. How will you manage a case of appendicular lump? (3+7+5) (MMC) [9th
Sem]

32. What are the causes of acute abdominal pain arising from epigastric region? How
chronic alcoholism leads to acute pancreatitis? Describe briefly the clinical
features and investigations required in case of acute pancreatitis. (3+3+5+4)
(MMC) [9th Sem]

33. What are the common causes of hematuria? Briefly discuss the C/f and M/m of
renal carcinoma. (5+5+5) (NBMC) [9th Sem]

34. What is the ‘triple assessment’ of a breast lump? Give outline of treatment of
malignant breast lump of a size 2 cm x 2 cmin a 50 year old female. (5+10) (NBMC)
[9th Sem]

35. What are the causes of lump in right iliac fossa? Describe the management of acute
appendicular lump. (7+8) (Kalyani) [9th Sem] {Paper 1}

36. What are the causes of bleeding per rectum? Outline the M/m of a patient aged 60
yrs suffering from bleeding per rectum. (5+10) (Kalyani) [9th Sem] {Paper 1}

COMPILED AND EDITED BY MEHBOOB-UL ISLAM 43


QUESTION BANK

SHORT NOTES
1. Pyloric Infection (MC,K) [8th Sem]
2. Appendicaular Lump (MC,K), (BMC) [8th Sem]
3. Treatment of Early Breast Carcinoma (MC,K) [8th Sem]
4. Various Presentation of Ca Stomach (MC,K) [8th Sem]
5. Manifestation of Gall Bladder Stone (MC,K) [8th Sem]
6. Phaeochromocytoma (MC,K) [8th Sem]
7. Treatment of CA Urinary Bladder (MC,K) [8th Sem]
8. Dermoid Cyst (MC,K), (R G Kar) [8th Sem]
9. Autosomal Dominant Polycystic Kidney Disease (MC,K) [8th Sem]
10. Thyroglossal Fistula (MC,K) [8th Sem]
11. Phylloide’s Tumor (NRS) [8th Sem]
12. Solitary Thyroid Nodule (NRS) [8th Sem]
13. Vitello-intestinal Duct (NRS) [8th Sem]
14. Complication of Total Parenenteral Nutrition (NRS) [8th Sem]
15. ERCP (NRS) [8th Sem]
16. Splenorrhaphy (NRS) [8th Sem]
17. Sebaceous Cyst (CNMC) [8th Sem]
18. Gastric Outlet Obstruction (CNMC) [8th Sem]
19. Bleeding Per Rectum (CNMC) [8th Sem]
20. Bed Sore (CNMC) [8th Sem]
21. Varicose Vein Complication (CNMC) [8th Sem]
22. Principles of Laparoscopic Surgery (CNMC) [8th Sem]
23. Day Care Surgery (CNMC) [8th Sem]
24. Hypospadius (CNMC), (BMC) [8th Sem]
25. Cleft Lip (CNMC) [8th Sem]
26. Post Burn Contracture (CNMC), (BMC) [8th Sem]
27. AIDS and the surgeons (CNMC) [8th Sem]
28. Varicocele (R G Kar) [8th Sem]
29. Dupuytren’s Contracture (R G Kar) [8th Sem]
30. Hemangioma (R G Kar) [8th Sem]
31. Post Anal Dermoid (R G Kar) [8th Sem]
32. Venous Ulcer (R G Kar) [8th Sem]
33. Cholesterosis of Gall Blader (R G Kar) [8th Sem]
34. Acute Retention of Urine (R G Kar) [8th Sem]

COMPILED AND EDITED BY MEHBOOB-UL ISLAM 44


QUESTION BANK

35. Marjolin’s Ulcer (R G Kar), (MMC) [8th Sem]


36. IVU (R G Kar) [8th Sem]
37. Advantages of USG (R G Kar) [8th Sem]
38. Meckel’s Diverticulum (R G Kar) [8th Sem]
39. Ganglion (R G Kar) [8th Sem]
40. Secondary Hyperparathyroidism (IPGMER) [8th Sem]
41. Estrogen and Progeterone Receptor (IPGMER) [8th Sem]
42. Glasgow Coma Scale (IPGMER), (KPC), (Kalyani) [8th Sem]
43. Bladder outlet Obstruction (IPGMER) [8th Sem]
44. Anorectal Malformation (IPGMER) [8th Sem]
45. Triage (IPGMER) [8th Sem]
46. Tubercular Granuloma (IPGMER) [8th Sem]
47. Phylloides Tumor of Breast (KPC) [8th Sem]
48. Flail Chest (KPC) [8th Sem]
49. Brachytherapy (KPC) [8th Sem]
50. Ameloblastoma (KPC) [8th Sem]
51. Pott’s Paraplegia (KPC) [8th Sem]
52. Autoclaving (BMC) [8th Sem]
53. Collar Stud Abscess (BMC) [8th Sem]
54. Cerebral Contussion (BMC) [8th Sem]
55. MRCP (BMC) [8th Sem], (MMC) [9th Sem]
56. Burn Contracture (BSMC) [8th Sem]
57. Complication of Blood Transfusion (BSMC) [8th Sem]
58. Management of Appendicular Lump (BSMC), (NBMC) [8th Sem]
59. Pleomorphic Adenoma (BSMC) [8th Sem]
60. Amoebic Liver Abscess (BSMC) [8th Sem]
61. Achalasia Cardia (MMC) [8th Sem]
62. Medullary CA Thyroid (MMC) [8th Sem]
63. Carcinoid Tumor of Appendix (MMC) [8th Sem]
64. Spread of CA Tongue (MMC) [8th Sem]
65. Factors that Delay Wound Healing (MMC) [8th Sem]
66. Parotid Abscess (MMC) [8th Sem]
67. Liver Abscess (NBMC) [8th Sem]
68. Tension Pneumothorax (NBMC) [8th Sem], (CMC) [9th Sem] {Paper 2}, (Kalyani)
[9th Sem] {Paper 2}, (BSMC) [9th sem] {paper 2}, (MMC) [9th Sem], (RG Kar) [9th
sem] {Paper 2}, (BMC) [9th Sem] {Paper 2}
69. Cholangiogram (NBMC) [8th Sem]
70. Early Gastric Cancer (NBMC) [8th Sem]

COMPILED AND EDITED BY MEHBOOB-UL ISLAM 45


QUESTION BANK

71. Gynaecomastia (NBMC) [8th Sem]


72. Complication of Acute Pancreatitis (NBMC) [8th Sem]
73. Sentinel Lymph Node Biopsy (NBMC) [8th Sem]
74. Effects of Gall Stones (NBMC) [8th Sem]
75. Electrolyte imbalance in Gastric Outlet Obstruction (NBMC) [8th Sem]
76. Intercostal Drain (Kalyani) [8th Sem]
77. Staghorn Calculus (Kalyani) [8th Sem]
78. Congenital Hydrocele (Kalyani) [8th Sem]
79. Classification of Peripheral Nerve Injury (Kalyani) [8th Sem]
80. Fistula in Ano (Kalyani) [8th Sem], (Kalyani) [9th Sem] {Paper 2}, (BSMC) [9th
sem] {paper 1}
81. Spinal Anaesthesia (Kalyani) [8th Sem]
82. Causes of Nipple Discharge (Kalyani) [8th Sem]
83. Necrotizing fasciitis (BSMC) [9th sem] {paper 1}
84. Ingrowing toe nail (BSMC) [9th sem] {paper 1}
85. Pre operative gut preparation (BSMC) [9th sem] {paper 1}
86. MCU (Micturating cystourethrogram) (BSMC) [9th sem] {paper 2}
87. Choledochal Cyst (BSMC) [9th sem] {paper 2}
88. Hypospadius (BSMC) [9th sem] {paper 2}
89. Skin Flaps (BSMC) [9th sem] {paper 2}
90. Duct Papilloma (BSMC) [9th sem] {paper 2}
91. Varicocele (BSMC) [9th sem] {paper 2}
92. Intercostal Chest Tube (ICT) (indication and brief outlines of method of
insertion) (BSMC) [9th sem] {paper 2}
93. Extradural hematoma (BSMC) [9th sem] {paper 2}
94. Deep Vein Thrombosis (BSMC) [9th sem] {paper 2}
95. Malignant Melanoma (BSMC) [9th sem] {paper 2}
96. Epidural anesthesia(BSMC) [9th sem] {paper 2}
97. Tracheostomy (BSMC) [9th sem] {paper 2}
98. Adjuvant chemotherapy (BSMC) [9th sem] {paper 2}
99. ANDI (BSMC) [9th sem] {paper 2}
100. Marjolin’s Ulcer (RG Kar) [9th sem] {Paper 1}
101. Meckel’s Diverticulum (RG Kar) [9th sem] {Paper 1}
102. Gall Stone Ileus (RG Kar) [9th sem] {Paper 1}
103. Femoral Hernia (RG Kar) [9th sem] {Paper 1}
104. Thyroglossal Cyst (RG Kar) [9th sem] {Paper 2}
105. Congenital hypertrophic pyoric stenosis (RG Kar) [9th sem] {Paper 2}
106. Congenital inguinal hernia (RG Kar) [9th sem] {Paper 2}
107. Cleft lip (RG Kar) [9th sem] {Paper 2}
108. Torsion of testes (RG Kar) [9th sem] {Paper 2}, (MMC) [9th Sem]

COMPILED AND EDITED BY MEHBOOB-UL ISLAM 46


QUESTION BANK

109. Extradural Haemorrhage (RG Kar) [9th sem] {Paper 2}, (BSMC) [9th sem]
{paper 1}
110. Split thickness skin grafting (RG Kar) [9th sem] {Paper 2}
111. Ludwig’s angina (RG Kar) [9th sem] {Paper 2}
112. Complication of Splenectomy (RG Kar) [9th sem] {Paper 2}
113. Decubitus Ulcer (RG Kar) [9th sem] {Paper 2}
114. ERCP (RG Kar) [9th sem] {Paper 2}
115. ESWL (RG Kar) [9th sem] {Paper 2}
116. Preoperative preparation of a Jaundice Patient (RG Kar) [9th sem] {Paper 2}
117. SIRS (NRS) [9th sem], (Kalyani) [9th Sem] {Paper 1}
118. OPSI (NRS) [9th sem]
119. Complication of thyroidectomy (NRS) [9th sem]
120. Staging of Ca Breast (NRS) [9th sem]
121. GIST (NRS) [9th sem]
122. Phaeochromocytoma (NRS) [9th sem], (BMC) [9th Sem] {Paper 2}
123. Causes of hematuria (NRS) [9th sem]
124. Lucid interval (NRS) [9th sem]
125. Intercostal chest drainage (NRS) [9th sem]
126. Basal Cell Carcinoma (NRS) [9th sem], (BMC) [9th Sem] {Paper 1}
127. Triangle of Safety in intercostal chest drain (IPGMER) [9th sem]
128. Chronic subdural hematoma (IPGMER) [9th sem]
129. Ectopia vesicae (IPGMER) [9th sem]
130. Epidural anesthesia (IPGMER) [9th sem]
131. Wolfe graft (IPGMER) [9th sem]
132. Dentigerous cyst (IPGMER) [9th sem], (BMC) [9th Sem] {Paper 2}
133. Universal precaution (IPGMER) [9th sem]
134. Hyperkalemia (IPGMER) [9th sem]
135. Ludwig’s angina (IPGMER) [9th sem]
136. Axontemesis (IPGMER) [9th sem]
137. MRCP (IPGMER) [9th sem]
138. Autonomic Bladder (IPGMER) [9th sem]
139. Classification of fistula in ano (CNMC) [9th Sem] {Paper 1}
140. Squamous cell CA (CNMC) [9th Sem] {Paper 1}
141. Umbilical hernia after laparoscopic cholecystectomy (CNMC) [9th Sem]
{Paper 1}
142. Metabolic changes of acute intestinal obstruction (CNMC) [9th Sem] {Paper
1}
143. Thyroid storm (CNMC) [9th Sem] {Paper 2}
144. Complication of peptic ulcer (CNMC) [9th Sem] {Paper 2}
145. Thyroglossal fistula (CNMC) [9th Sem] {Paper 2}
146. Urethral rupture (CNMC) [9th Sem] {Paper 2}
147. Chest trauma (CNMC) [9th Sem] {Paper 2}

COMPILED AND EDITED BY MEHBOOB-UL ISLAM 47


QUESTION BANK

148. Epidural anesthesia (CNMC) [9th Sem] {Paper 2}


149. Electrolyte changes in gastric outlet obstruction (CNMC) [9th Sem] {Paper 2}
150. Causes of surgical obstructive jaundice (CNMC) [9th Sem] {Paper 2}
151. Extradural hematoma (CNMC) [9th Sem] {Paper 2}
152. ANDI (CMC) [9th Sem] {Paper 1}
153. Classification of salivary gland tumors (CMC) [9th Sem] {Paper 1}
154. Ranula (CMC) [9th Sem] {Paper 1}
155. Venous ulcer (CMC) [9th Sem] {Paper 1}
156. Haemorrhoids (CMC) [9th Sem] {Paper 1}
157. Tetralogy of Fallot (CMC) [9th Sem] {Paper 2}
158. Meconium Ileus (CMC) [9th Sem] {Paper 2}
159. Undescended testis (CMC) [9th Sem] {Paper 2}
160. Neoadjuvent therapy in colorectal Ca (CMC) [9th Sem] {Paper 2}
161. Investigation for obscure GI bleeding (CMC) [9th Sem] {Paper 2}
162. Preanaesthetic check up (CMC) [9th Sem] {Paper 2}
163. Hypercalcemia (CMC) [9th Sem] {Paper 2}
164. Management options for urinary stones (CMC) [9th Sem] {Paper 2}
165. Physiological effects of splenomegaly (KPC) [9th Sem] {Paper 1}
166. Intussusception (KPC) [9th Sem] {Paper 1}
167. Courvoisier sign (KPC) [9th Sem] {Paper 1}
168. Thyroglossal cyst (KPC) [9th Sem] {Paper 1}
169. Complication of blood transfusion (KPC) [9th Sem] {Paper 1}
170. Infantile hypertrophic pyloric stenosis (KPC) [9th Sem] {Paper 2}
171. Hutchinsons pupil (KPC) [9th Sem] {Paper 2}
172. Intercostal drain (KPC) [9th Sem] {Paper 2}
173. Phimosis (KPC) [9th Sem] {Paper 2}
174. Principle of M/m of Cleft palate (KPC) [9th Sem] {Paper 2}
175. Complications of spinal anaesthesia (KPC) [9th Sem] {Paper 2}
176. Epulis (KPC) [9th Sem] {Paper 2}
177. Mammography (KPC) [9th Sem] {Paper 2}
178. Wax bath (KPC) [9th Sem] {Paper 2}, (MMC) [9th Sem]
179. Common complications of cancer chemotherapy (KPC) [9th Sem] {Paper 2}
180. Arteriovenous fistula (BMC) [9th Sem] {Paper 1}
181. Glasgow Coma scale (BMC) [9th Sem] {Paper 1}
182. Pre-operativ epreparation of pyloric stenosis (BMC) [9th Sem] {Paper 1}
183. Carbuncle (MMC) [9th Sem]
184. Methods of Biopsy (BMC) [9th Sem] {Paper 1}
185. Strangulated hernia (BMC) [9th Sem] {Paper 2}
186. Hypospadias (BMC) [9th Sem] {Paper 2}
187. Regional anesthesia (BMC) [9th Sem] {Paper 2}
188. Testicular tumor (BMC) [9th Sem] {Paper 2}
189. Burn contracture (BMC) [9th Sem] {Paper 2}

COMPILED AND EDITED BY MEHBOOB-UL ISLAM 48


QUESTION BANK

190. Thyroglossl cyst (BMC) [9th Sem] {Paper 2}


191. Cleft lip (BMC) [9th Sem] {Paper 2}
192. Perianal fistula (BMC) [9th Sem] {Paper 2}
193. ERCP (BMC) [9th Sem] {Paper 2}
194. Subdural hematoma (BMC) [9th Sem] {Paper 2}
195. Meckel’s diverticulum (BMC) [9th Sem] {Paper 2}
196. Fournier’s gangrene (BMC) [9th Sem] {Paper 2}
197. Flail chest (NBMC) [9th Sem]
198. Pyloric stenosis in infants (NBMC) [9th Sem]
199. Significance of Glasgow coma scale (NBMC) [9th Sem]
200. Types of skin graft (NBMC) [9th Sem]
201. Treatment of Ca penis (NBMC) [9th Sem]
202. Massive hemothorax (Kalyani) [9th Sem] {Paper 1}
203. Electric burn (Kalyani) [9th Sem] {Paper 1}
204. M/m of Diabetic Foot (Kalyani) [9th Sem] {Paper 1}
205. Electrolyte disturbance in pyloric stenosis (Kalyani) [9th Sem] {Paper 1}
206. Sialolithiais (Kalyani) [9th Sem] {Paper 2}
207. Breast abscess (Kalyani) [9th Sem] {Paper 2}
208. Choledochal cyst (Kalyani) [9th Sem] {Paper 2}
209. Complication of undescended testis (Kalyani) [9th Sem] {Paper 2}
210. Fournier’s gangrene of scrotum (Kalyani) [9th Sem] {Paper 2}
211. Cardiac arrest (Kalyani) [9th Sem] {Paper 2}
212. MRI (Kalyani) [9th Sem] {Paper 2}
213. Complication of Radiotherapy (Kalyani) [9th Sem] {Paper 2}
214. Odontomes (Kalyani) [9th Sem] {Paper 2}
215. Short wave diathermy (Kalyani) [9th Sem] {Paper 2}
216. Critical limb ischaemia (Kalyani) [9th Sem] {Paper 2}
217. Skin grafting (Kalyani) [9th Sem] {Paper 2}

COMPILED AND EDITED BY MEHBOOB-UL ISLAM 49


QUESTION BANK

SHORT NOTES
(ORTHO)
1. Giant Cell Tumor (R G Kar) [8th Sem], (BMC) [9th Sem] {Paper 1}
2. Colle’s Fracture (R G Kar) [8th Sem], (BMC) [9th Sem] {Paper 1} complication
of (CNMC) [9th Sem] {Paper 1}, (BSMC) [9th sem] {paper 1}
3. Volkmann’s Ischaemic Contracture (R G Kar) [8th Sem], pathophysiology of
(CNMC) [9th Sem] {Paper 1}
4. Myositis Ossificans Traumatica (R G Kar) [8th Sem]
5. Non-union (R G Kar) [8th Sem] (BMC), (Kalyani) [8th Sem], (Kalyani) [9th Sem]
{Paper 1}
6. CTEV (R G Kar) [8th Sem], (BMC) [9th Sem] {Paper 1}, deformities of CTEV
(Kalyani) [9th Sem] {Paper 1}
7. Chronic Osteomyelitis (R G Kar) [8th Sem], (CMC) [9th Sem] {Paper 1}
8. Lateral Condylar Fracture in Children (IPGMER) [8th Sem]
9. Syme’s Amputation (IPGMER) [8th Sem]
10. Monteggia Fracture (IPGMER) [8th Sem]
11. Thomas’ Test (IPGMER) [8th Sem]
12. Pathogenesis and Clinical features of Perthe’s Disease (KPC) [8th Sem]
13. Unicameral Bone Cyst (KPC) [8th Sem]
14. Management of Fracture Shaft Humerus (KPC) [8th Sem]
15. Osteoarthrosis of Knee Joint (KPC) [8th Sem]
16. Complication of Supracondylar fracture (of humerus) in children (BMC) [8th
Sem], (BMC) [9th Sem] {Paper 1}
17. Classification of Fracture Neck Femur in Adult (BMC) [8th Sem]
18. Giant Cell Tumor of Bone (BMC), (Kalyani) [8th Sem]
19. Management of Chronic Osteomyelitis (BMC) [8th Sem]
20. Scaphoid Fracture (BMC) [8th Sem]
21. Signs and Symptoms of Rickets (BMC) [8th Sem]
22. Wrist Drop (BMC) [8th Sem]
23. Compound Fracture (BSMC) [8th Sem]
24. Carpal Tunnel Syndrome (BSMC) [8th Sem] (CNMC) [9th Sem] {Paper 1}
25. Osteochondroma (BSMC) [8th Sem]
26. Spina Bifida (BSMC) [8th Sem]
27. Ewing’s Sarcoma (BSMC) [8th Sem], (CMC) [9th Sem] {Paper 1}

COMPILED AND EDITED BY MEHBOOB-UL ISLAM 50


QUESTION BANK

28. Warthim’s Tumour (MMC) [8th Sem]


29. Complication of Supracondylar fracture of humerus (Kalyani) [8th Sem]
30. Exostosis of Bone (Kalyani) [8th Sem]
31. Osteoclastoma (BSMC) [9th sem] {paper 1}, (NBMC) [9th Sem]
32. Hip dislocation (BSMC) [9th sem] {paper 1}
33. Skin grafting(BSMC) [9th sem] {paper 1}
34. Psoas abscess (BSMC) [9th sem] {paper 1}
35. Pott’s fracture (BSMC) [9th sem] {paper 1}
36. Galeazzi Fracture (RG Kar) [9th sem] {Paper 1}
37. Sequestrum (RG Kar) [9th sem] {Paper 1}, (Kalyani) [9th Sem] {Paper 1}
38. De Quervain’s Disease (RG Kar) [9th sem] {Paper 1}
39. Sudeck’s Osteodystrophy (RG Kar) [9th sem] {Paper 1}
40. Housemaid’s Knee (RG Kar) [9th sem] {Paper 1}
41. Shoulder Dislocation (RG Kar) [9th sem] {Paper 1}
42. Pathological Fracture (RG Kar) [9th sem] {Paper 1}
43. Bone Graft (RG Kar) [9th sem] {Paper 1}
44. Congenital talipes Equinus Varus (NRS) [9th sem]
45. Osteoporosis-management, Laboratory diagnosis (NRS) [9th sem]
46. Anterior dislocation of Shoulder- clinical features, clinical tests (NRS) [9th sem]
47. Causes of non union (NRS) [9th sem]
48. Osteosarcoma (NRS) [9th sem]
49. Rickets (NRS) [9th sem]
50. Pathophysiology of chronic osteomyelitis and sequestrum (NRS) [9th sem]
51. Pott’s Fracture (IPGMER) [9th sem]
52. Mallet finger (IPGMER) [9th sem]
53. Ring sequestrum (IPGMER) [9th sem]
54. Mc Murray Test (IPGMER) [9th sem]
55. Triple deformity (IPGMER) [9th sem]
56. Volkman’s Ischaemia (IPGMER) [9th sem]
57. Tardy Ulnar palsy (CNMC) [9th Sem] {Paper 1}
58. Management of osteoclastoma of upper end of tibia (CNMC) [9th Sem] {Paper 1}
59. Osteogenesis imperfect (CMC) [9th Sem] {Paper 1}
60. Complication of supracondylar fracture of humerus (CMC) [9th Sem] {Paper 1}
61. Types of lower limb amputation (CMC) [9th Sem] {Paper 1}
62. C/f of ulnar nerve palsy (KPC) [9th Sem] {Paper 1}
63. Bone grafting (KPC) [9th Sem] {Paper 1}
64. Treatment of fractures femur neck (KPC) [9th Sem] {Paper 1}
65. Multiple nmyeloma (KPC) [9th Sem] {Paper 1}
66. Genu valgum (KPC) [9th Sem] {Paper 1}
67. Anterior dislocation of shoulder- presentation & M/m (BMC) [9th Sem] {Paper 1}
68. Frozen shoulder (MMC) [9th Sem]

COMPILED AND EDITED BY MEHBOOB-UL ISLAM 51


QUESTION BANK

69. Myositis ossificans (MMC) [9th Sem]


70. Fracture clavicle (MMC) [9th Sem]
71. Osteoid osteoma (MMC) [9th Sem]
72. Below knee amputation (MMC) [9th Sem]
73. Radiological features of chronic osteomyelitis (NBMC) [9th Sem]
74. Non union of fratures (NBMC) [9th Sem], (Kalyani) [9th Sem] {Paper 1}
75. Osteoclastoma (NBMC) [9th Sem]
76. Supracondylar fracture of humerus (NBMC) [9th Sem]
77. Septic arthritis (NBMC) [9th Sem]
78. Complication of Colles Fracture (Kalyani) [9th Sem] {Paper 1}
79. Morant Baker Cyst (Kalyani) [9th Sem] {Paper 1}

COMPILED AND EDITED BY MEHBOOB-UL ISLAM 52


QUESTION BANK

COMPILED AND EDITED BY MEHBOOB-UL ISLAM 53


QUESTION BANK

COMPILED AND EDITED BY MEHBOOB-UL ISLAM 54


QUESTION BANK

LONG QUESTIONS
Gr- A
1. What is acute coronary syndrome? How do you manage a case of acute
myocardial infarction in a 65 year old hypertensive patient? (3+7) [14]

2. What do you mean by Atrial Fibrillation (AF) and lone AF? What are the
common causes and usual consequence of AF? Enumerate the drugs used in
the T/t of AF. (2+6+2) [14, Supple]

3. Define Heart Failure. Describe its pathophysiology. Outline the management


of Acute left ventricular failure. (2+4+4) [13]

4. A 60 yr old diabetic patienton insulin therapy was brought to casualty


department with history of sudden onset altered level of consciousness.
What are the likely possibilities here? How will you investigate and manage
this case? (2+4+4) [13, Supple]

5. Define unstable angina. How will you examine, investigate and manage a case
of unstable angina in a 50 years old patient. (2+8) [12]

6. Discuss the 1st 12 hr of M/m of AMI in a 60 yr old patient. (10) [12, Supple]

7. Define heart failure. Enumerate different types of heart failure. (2+8) [11]

8. Discuss the M/m of Heart failure. (10) [11, Supple]

9. What are the causes, clinical features and diagnostic features of acute
pericarditis? (4+4+3) [10]

10. Describe the clinical features and management of a patient with AMI. (10)
[10, supple]

11. Discuss the aetiopathogenesis, clinical features and treatment of acute


rheumatic fever. (3+4+3) [09]

COMPILED AND EDITED BY MEHBOOB-UL ISLAM 55


QUESTION BANK

12. Draw a diagram of conduction system of the heart. Mention etiology of


irregular pulse. Outline the management of atrial fibrillation. (3+3+4) [09,
supple]

13. Discuss the pathogenesis, complication and management of Falciparum


malaria. (2+3+5) [08]

14. Define ascites. Enumerate various causes of ascites. What are the
treatment option for ascites? (1+3+6) [08, supple]

15. What are the clinical manifestation of infective endocarditis? What are the
diagnostic tests done for it? Discuss the principle of treatment of infective
endocarditis.(3+3+4) [08, supple]

COMPILED AND EDITED BY MEHBOOB-UL ISLAM 56


QUESTION BANK

LONG QUESTIONS
Gr- B
1. Discuss diagnosis and management of a case of 15 year old boy presenting
with severe breathlessness, puffiness of face with hypertension and active
urinary sediments on urinary analysis. (4+6) [14]

2. A male patient aged 22 years has been admitted due to sudden onset of
convulsion with fever and jaundice of 5 days prior to admission. What are
the possibilities? How routine and specialized tests will help you to arrive at
a diagnosis? (4+3+3) [14]

3. A 30 yr alcoholic male patient presents with hematemesis and melena. How


will you manage the case? (10) [14, Supple]

4. A 60 yr old female presents with anasarca. How will you proceed to diagnose
and treat the patient? (5+5) [14, Supple]

5. A 60 year old diabetic patient on insulin therapy was brought to casualty


department with history of sudden onset altered level consciousness. What
are the likely possibilities here? How will you approach this case? (3+7) [13]

6. How do you approach in a 60 years old male patient presenting with chronic
diarrhea mixed with blood with evidence of malabsorption for last 6 months?
(10) [13]

7. A 19 yr college student presents to you with a h/o jaundice, high colored


urine for 3 days preceeded by fever , nausea, malaise and myalgia for 4 days.
He gives a h/o academic trip for 1 month before his present illness. What
are the D/d? briefly discuss the investigation and M/m of the case. (2+4+4)
[13, Supple]

8. How do you approach in a 26 years female patient with bipedal swelling and
anemia with h/o fever for last 3 months? (10) [13, Supple]

9. Discuss the clinical and laboratory approach to a case of ascitis. (5+5) {12]

COMPILED AND EDITED BY MEHBOOB-UL ISLAM 57


QUESTION BANK

10. Discuss how you will approach a case of fever with polyarthritis with skin
rash of about 2 weeks duration in a young female? (10) [12]

11. What is nephrotic syndrome? How will you investigate and manage a patient
of nephrotic syndrome having normal serum creatinine? (3+7) [12, Supple]

12. A 46 yr old male patient has come to the emergency with acute upper
abdominal pain with vomiting. How do you proceed to diagnose the case? (10)
[12, Supple]

13. A 55-year old female has come to the Emergency with acute precordial
chest pain. How do you proceed to diagnose the cased? (10) [11]

14. A 30-year old male has come to the Emergency with severe epigastric pain
and vomiting. How do you proceed to diagnose the case? (10) [11]

15. A 40 yr old male patient with Chronic liver ds. And ascites was otherwise
doing well to have his ascites increased suddenly in last 7 days. Discuss the
approach. (10) [11, Supple]

16. A 35 yr old female patient presents with pain in multiple joints for last 2
months. How will you proceed to diagnose and manage the case? (10) [11,
Supple]

17. A 14-year old male was admitted in the emergency with history of weight
loss and polyuria. He was having deep breathing. How will you manage this
patient? (2+8) [10]

18. A 15 –year old female presented with a history of fever, arthralgia and
rashes. Discuss the differential diagnosis and management plan. (5+5) [10]

19. A young male was admitted in an emergency ward with fever and headache.
He has neck stiffness- discuss the salient investigations, differential
diagnosis and management. (10) [10, supple]

20. Following severe gastroenteritis a patient develops oliguria. Discuss further


how will you manage him. (10) [10, supple]

21. A 32 year female presents with high rise of temperature, haematuria and
right sided flank pain. Discuss how would you proceed to arrive at diagnosis.
Give its management plan for medical causes. (4+6) [09]

COMPILED AND EDITED BY MEHBOOB-UL ISLAM 58


QUESTION BANK

22. A 40 year old alcoholic male patient with severe pain in the epigastrium.
What are the likely causes and how would you proceed to arrive at diagnosis?
Give its management plan. (2+4+4) [09]

23. An adult brought to emergency department with fever, vomiting and


diarrhea. How will you manage the case? (10) [09, supple]

24. A 45 yr old male presented with vomiting of blood. How will you proceed to
diagnose the case? (10) [09, supple]

25. What are the clinical and laboratory (including ECG) features of acute
myocardial infarction? Discuss its management in first six hours. (2+3+5)
[08]

26. What are the causes of secondary hypertension? Discuss the treatment and
complication of malignant hypertension. (3+4+3) [08]

27. What are the common causes of chest pain in a 40 year old male? What are
the diagnostic test for acute myocardial infarction? (6+4) [08, supple]

COMPILED AND EDITED BY MEHBOOB-UL ISLAM 59


QUESTION BANK

BRIEF NOTES
(each carrying 4 marks)

Gr- C
1. Complication of Ascites [14]

2. Management of mitral stenosis [14]

3. Indication of dialysis [14]

4. Treatment of Herpes Zoster [14]

5. Management of organo phosphorous poisoning [14]

6. Heart stroke [14]

7. Beriberi [14]

8. C/f of ulcerative colitis [14, Supple]

9. Dilated cardiomyopathy [14, Supple]

10. Types of tremor [14, Supple]

11. Vit-D deficiency [14, Supple]

12. T/t of P. falciparum malaria [14, Supple]

13. Neurotoxic snake bite [14, Supple]

14. T/t of ac. Gouty arthritis [14, Supple]

15. Fluid replacement therapy in acute diarrhea [13]

16. Hypogonadism [13]

17. Prognostic parameters of acute pancreatitis [13]

18. Dengue shock syndrome [13]

COMPILED AND EDITED BY MEHBOOB-UL ISLAM 60


QUESTION BANK

19. Laboratory diagnosis of Rheumatoid Arthritis [13]

20. Treatment of vivax malaria [13]

21. Causes of secondary hypertension [13]

22. Causes of small vessel vasculitis [13, Supple]

23. Fever in old age-of 6 months duration [13, Supple]

24. Patellar clonus [13, Supple]

25. M/m of Viper bite [13, Supple]

26. Genetic linkage [13, Supple]

27. Reactive arthritis [13, Supple]

28. Diagnosis of complete heart block [13, Supple]

29. Henoch Scholen purpura [12]

30. Proton pump inhibitors [12]

31. Solitary pulmonary nodule [12]

32. Micro albuminuria [12]

33. Vitamin D deficiency [12]

34. Dwarfism [12]

35. Diuretics in clinical practice [12]

36. Drug therapy of acute peptic ulcer [11]

37. Serum ascetic fluid-albumin gradient [11]

38. Anion gap: Definition and relevance [11]

39. Hirsutism: Definition and causes [11]

40. Niacin : Deficiency symptoms [11]

COMPILED AND EDITED BY MEHBOOB-UL ISLAM 61


QUESTION BANK

41. Differential diagnosis of Mumps [11]

42. Ascariasis: Diagnosis and management [11]

43. Significance of proteinuria [11, supple]

44. M/m of cerebral malaria [11, supple]

45. Sex linked inheritance [11, supple]

46. Blind loop syndrome [11, supple]

47. Type IV hypersensitivity reaction [11, supple]

48. HLA linkage [11, supple]

49. Management of status epillepticus [10]

50. Gene therapy [10]

51. Indications of dialysis [10]

52. Complication of cirrhosis [10]

53. UTI- aetiology and management [10]

54. Signs and symptoms of Thyrotoxicosis [10]

55. Significance of ‘a’ waves in neck veins [10]

56. T/t of IBS [10, supple]

57. T/t of PSVT [10, supple]

58. Diagnosis of Ac. PSGN [10, supple]

59. Classification of DM [10, supple]

60. D/d of hematuria [10, supple]

61. M/m of Osteoporosis [10, supple]

62. Sickle cell ds. [10, supple]

COMPILED AND EDITED BY MEHBOOB-UL ISLAM 62


QUESTION BANK

63. Radiological features of mitral stenosis [09]

64. Spontaneous bacterial peritonitis [09]

65. Genetic mutation [09]

66. Psoriatic arthropathy [09]

67. Management of neurotoxic snake bite [09]

68. Dengue shock syndrome [09]

69. Management of acute variceal bleeding [09]

70. Complication of Mitral stenosis [09, supple]

71. M/m of Ac. Hepatic encephalopathy [09, supple]

72. M/m of Ac. Bacillary dysentery [09, supple]

73. Diagnostic criteria of Rheumatoid arthritis [09, supple]

74. Clinical features of vit- d deficiency in adults [09, supple]

75. BMI [09, supple]

76. Significance of examination of neck vein [09, supple]

77. Treatment of UTI in female [08]

78. Extraintestinal manifestation of inflammatory bowel disease [08]

79. Diagnostic criteria of SLE [08]

80. Clinical features of Vitamin – A [08]

81. Management of fviper snake bite [08]

82. Anaphylaxis [08]

83. Tropical Sprue [08]

84. Protein energy malnutrition [08, supple]

COMPILED AND EDITED BY MEHBOOB-UL ISLAM 63


QUESTION BANK

85. Clinical features of rheumatoid arthritis [08, supple]

86. Treatment of uncomplicated P. vivax malaria [08, supple]

87. Management of snake bite [08, supple]

88. Treatment of anaphylaxis [08, supple]

89. Complication of nephrotic syndrome [08, supple]

90. Causes of vomiting [08, supple]

COMPILED AND EDITED BY MEHBOOB-UL ISLAM 64


QUESTION BANK

SHORT NOTES
(each carrying 4 marks)

Gr- D
1. Myasthenia gravis [14]

2. Drug treatment of Kalaazar [14]

3. Recurrent urinary tract infection [14]

4. X-linked disorders [14]

5. Treatment of osteoarthritis [14]

6. Biology of aging [14]

7. Anion gap [14]

8. Tetany [14, Supple]

9. Lepra reaction [14, Supple]

10. Thrombolytic therapy in AMI [14, Supple]

11. APLA [14, Supple]

12. Post-exposure prophylaxis of Hep- B infection [14, Supple]

13. JVP [14, Supple]

14. Prophylaxis of Rheumatic fever [14, Supple]

15. Asymptomatic bacteria [13]

16. Vitamin D toxicity [13]

17. Diagnosis of atrial fibrillation [13]

18. Utricaria [13]

COMPILED AND EDITED BY MEHBOOB-UL ISLAM 65


QUESTION BANK

19. Anorexia nervosa [13]

20. Enumerate drugs in treatment of gout [13]

21. Hepatorenal syndrome [13]

22. Non ST elevation Acute myocardial infarction [13, Supple]

23. Refractory ascites [13, Supple]

24. Immune mediated transfusion reaction [13, Supple]

25. Gynaecomastia [13, Supple]

26. Down’s Syndrome [13, Supple]

27. DOTS in TB [13, Supple]

28. Common indications for Echocardiography [13, Supple]

29. Pulsus paradoxus [12]

30. Bone changes in chronic renal failure [12]

31. Management of ruptured esophageal varices [12]

32. Laboratory diagnosis of Kala-azar [12]

33. Treatment of uncomplicated falciparum malaria [12]

34. Neurotoxic snake bite [12]

35. Cytokines [12]

36. Corrigan sign [11]

37. Raynaud’s phenomenon [11]

38. Wide split of S2(2nd heart sound) [11]

39. Anaemia in C.K.D. [11]

40. Vasculotoxic snake bite [11]

COMPILED AND EDITED BY MEHBOOB-UL ISLAM 66


QUESTION BANK

41. Refractory Ascites [11]

42. Extra intestinal manifestation of Inflammatory Bowel Disease [11]

43. Osteomalacia [11, supple]

44. Rheumatoid hand deformity [11, supple]

45. Impetigo [11, supple]

46. Oncogene [11, supple]

47. Triceps skinfold thickness [11, supple]

48. Macroglossia [11, supple]

49. Spondyloarthrides: classification [11, supple]

50. Diabetic foot [10]

51. Complications of mitral regurtation [10]

52. Differential diagnosis of meningitis [10]

53. Hemiplegia in young [10]

54. Management of organophosphorous poisoning [10], [09]

55. DMARDs in Rheumatoid Arthritis [10]

56. Management of LVF [10]

57. Peripheral signs of AR [10, supple]

58. Drugs and liver [10, supple]

59. Erythema nodosum [10, supple]

60. Swine flu M/m [10, supple]

61. Complications of Ac. Falciparum malaria [10, supple]

62. Aetiology of Peptic Ulcer [10, supple]

COMPILED AND EDITED BY MEHBOOB-UL ISLAM 67


QUESTION BANK

63. Clinical signs of hyperthyroidism [10, supple]

64. Corrigan’s sign [09]

65. Acute adverse reactions of blood transfusion [09]

66. Clinical features of pericardial effusion [09]

67. Beriberi [09]

68. Drug-induced liver disease [09]

69. Renal manifestation of SLE [09]

70. Peripheral cyanosis [09, supple]

71. Microalbuminuria [09, supple]

72. Clinical features of dengue [09, supple]

73. Analysis of ascetic fluid [09, supple]

74. Tetanus prophylaxis [09, supple]

75. Ig A nephropathy [09, supple]

76. M/m of acute uncomplicated falciparum malaria [09, supple]

77. Aetiology of acute renal failure [08]

78. Management of Typhoid Fever [08]

79. Exudative Ascites [08]

80. Causes of Hyponatremia [08]

81. Local parenteral nutrition [08]

82. Modified Jone’s Criteria [08]

83. Innate Immune System [08]

84. Complication of typhoid fever [08, supple]

COMPILED AND EDITED BY MEHBOOB-UL ISLAM 68


QUESTION BANK

85. Treatment of viral diarrhea [08, supple]

86. Vit- A deficiency [08, supple]

87. Treatment of malignant hypertension [08, supple]

88. Side effects of anti-tubercular drugs [08, supple]

89. Clinical manifestation of SLE [08, supple]

90. Causes of chronic liver disease [08, supple]

COMPILED AND EDITED BY MEHBOOB-UL ISLAM 69


QUESTION BANK

COMPILED AND EDITED BY MEHBOOB-UL ISLAM 70


QUESTION BANK

LONG QUESTIONS
Gr- A
1. Define Pneumonia. Classify Pneumonia. How will you treat a case of
community acquired Pneumonia? (2+3+5) [14]

2. A 60yr old male presents with haemoptysis. What are the likely causes?
How will you manage such a case? (2+8) [14, Supple]

3. Describe in brief the management of acute severe asthma in an adult


patient. (10) [13]

4. Describe etiopathogenesis of parkinsonism. Describe C/f and M/m of


parkinsonism. (2+4+4) [13, Supple]

5. Define respiratory failure. Discuss the management of acute exacerbation


of chronic obstructive lung disease. (3+7) [12]

6. Describe the C/f and M/m of Subarachnoid haemorrhage. (5+5) [12,


Supple]

7. Discuss the etiology, clinical features and management of a case of GB


syndrome. (3+3+4) [11]

8. Outline the C/f, diagnosis and M/m of Subarachnoid haemorrhage.


(3+3+4) [11, Supple]

9. Discuss the aetiology, management and complication of status epilepticus.


(3+4+3) [10]

10. Discuss the etiology and M/m of CVA9stroke. (10) [10, Supple]

11. Draw a diagram of myoneural junction. Discuss pathogenesis, clinical


features, diagnosis and management of a disease ofd myoneural junction.
(3+5+2) [09]

COMPILED AND EDITED BY MEHBOOB-UL ISLAM 71


QUESTION BANK

12. Discuss the etiopathogenesis and M/m of Bronchial asthma. (5+5) [09,
Supple]

13. Enumerate Respiratoy Function Test. Name three obstructive and three
restrictive lung diseases. How will you differentiate obstructive and
restrictive lung diseases by respiratory function tests. (5+3+2) [08]

14. Define haemoptysis. What are the different causes? Discuss T/t of
haemoptysis. (2+4+4) [08, Supple]

COMPILED AND EDITED BY MEHBOOB-UL ISLAM 72


QUESTION BANK

LONG QUESTIONS
Gr- B
1. What is status epilepticus. Describe the management of status
epilepticus. (3+7) [14]

2. 0Enumerate the causes of generalized lymphadenopathy. How will you


proceed to diagnose a case of lymphadenopathy. (3+7) [14]

3. A 40 yr old male patient presents with history of gradual onset weakness


of both lower limbs. How will you approach the case for diagnosis? (10)
[14, Supple]

4. A 68 yrs old man presented with semi-conscious state, known diabetic


resides alone in a village. How will you investigate and manage the
patient? (4+6) [14, Supple]

5. A man aged 33 years presents with severe haematemesis. What may be


the likely causes? How will you proceed to diagnose the case? (4+6) [13]

6. An older patient is admitted with acute stroke. What may be the


important causes? Outline the management strategy in 1st 24 hours. (3+7)
[13]

7. A 19 yr male patient was referred from a rural hospital to your hospital


to your hospital with fever and altered sensorium of 5 days duration.
What might be the possible causes? Describe your plan of approach to
the problems in terms of diagnosis and M/m. (2+4+4) [13, Supple]

8. What is acute nephritic syndrome? Outline the M/m strategy of a boy


aged 18 yrs suffering from nephritic syndrome. (4+6) [13, Supple]

9. A man aged 70 years presented with lower GI bleeding. What may be the
likely causes? How will you proceed to come to diagnosis? (4+6) [12]

COMPILED AND EDITED BY MEHBOOB-UL ISLAM 73


QUESTION BANK

10. A young patient presents in the emergency with unconsciousness and


deep breathing. He has a history of weight loss and polyuria. Discuss how
will you manage the case? (10) [12]

11. Define bronchial asthma . State the aims of therapy and principles of
drug therapy in acute severe asthma. (2+4+4) [12, Supple]

12. A 26 yr old female came to emergency with sudden onset gum bleeding
and purpuric rash over trunk with a h/o fever for 3 days. How will you
examina and investigate the patient to reach a diagnosis? (10) [12,
Supple]

13. What are the poor prognostic clinical features of acute severe asthma?
How will you manage such a patient?(10) [11]

14. A man aged 50 year is losing weight for sometime, what may be the
possible causes? How will you proceed for diagnosis? (3+7) [11]

15. A 50 yr old male patient is refered from a peripheral hospital with


h/ofever, difficulty in breathing of recent onset and dullness on
percussion over the right side of the chest. How do you proceed to a
clinical diagnosis in the emergency department? How will you manage such
a case? (4+6) [11, Supple]

16. A lady of 30 yrs has presented with goiter. How do you proceed to come
to an etiological diagnosis? (10) [11, Supple]

17. Discuss the treatment of hyperosmolar, non-ketotic coma. Describe its


complication. (6+4) [10]

18. Describe the causes of “Pancytopenia”. How will you manage a case of
Aplastic Anaemia (6+4) [10]

19. Discuss causes, C/f and M/m of Idiopathic thrombocytopenic purpura.


(3+3+4) [10, Supple]

20. How will you stage chronic kidney diseases(CKD)? How will you manage
CKD stage IV? (5+5) [10, Supple]

COMPILED AND EDITED BY MEHBOOB-UL ISLAM 74


QUESTION BANK

21. A 50 year old male presents with loss of weight. What are the possible
causes? How will you approach the case to arrive at a diagnosis? (3+7)
[09]

22. A 60 year old diabetic patient has been brought to casualty department
in unconscious state. What are the possible causes? How will you
approach the case to arrive at a diagnosis? (3+7) [09]

23. Describe the C/f and investigations of CML. Outline its M/m. (3+4+3)
[09, Supple]

24. Mention the common causes of coma in diabetic patients. How will you
diagnose and manage a case of diabetic ketoacidosis? (3+7) [09, Supple]

25. Discuss the risk factors for acute stroke. What are the steps in the
management of a patient with acute ischemic stroke? (5+5) [08]

26. Discuss the clinical and laboratory features of hypothyroidism. How will
you manage myxedema coma? (5+5) [08]

27. What are the clinical manifestation of a patient with lobar pneumonia?
What are the diagnostic tests and treatment plan for lobar pneumonia?
(3+3+4) [08, Supple]

28. What are the C/f and diagnostic tests in a ptient with Diabetic
ketoacidosis(DKA)? Discuss the principle of treatment of DKA. (4+3+3)
[08, Supple]

COMPILED AND EDITED BY MEHBOOB-UL ISLAM 75


QUESTION BANK

BRIEF NOTES
(each carrying 4 marks)

Gr- C
1. Treatment of CML [14]

2. Stage of HIV infection [14]

3. Myxoedema Coma [14]

4. Classification of Diabetes mellitus [14]

5. Universal Prophylaxis [14]

6. Carpal Tunnel Syndrome [14]

7. Megaloblastic anaemia [14]

8. C/f of thyrotoxicosis [14, Supple]

9. Psoriasis [14, Supple]

10. HIV- post exposure prophylaxis [14, Supple]

11. Diabetic nephropathy- early diagnosis [14, Supple]

12. Peritoneal dialysis [14, Supple]

13. Koplik’s spot [14, Supple]

14. Neurologuic complication of diphtheria [14, Supple]

15. Clinical features of Parkinsonism [13]

16. Microcytic anaemia [13]

17. Extensor planter response [13]

18. Management of leprosy [13]

COMPILED AND EDITED BY MEHBOOB-UL ISLAM 76


QUESTION BANK

19. Bitot’s spot [13]

20. Manic depressive psychosis [13]

21. Management of ITP [13]

22. C/f of 3rd CN palsy [13, Supple]

23. M/m of diabetic ketoacidosis [13, Supple]

24. Tetany [13, Supple]

25. Diarrhoea in HIV infected patients. [13, Supple]

26. Thyrotoxic crisis [13, Supple]

27. Tension Pneumothorax [13, Supple]

28. Diagnosis of Crohn’s ds [13, Supple]

29. Hypertonia [12]

30. Clinical presentation of neurological tuberculosis [12]

31. Tropical Sprue [12]

32. Vesicular skin eruptions [12]

33. Clinical features of hyperthyroidism [12]

34. Anxiety neurosis [12]

35. Causes of haemolytic anaemia [12]

36. HAART therapy [12, Supple]

37. M/m of Community Acquired Pneumonia [12, Supple]

38. Chorea [12, Supple]

39. Diagnosis of beta thalassemia [12, Supple]

40. Causes of osteoporosis [12, Supple]

COMPILED AND EDITED BY MEHBOOB-UL ISLAM 77


QUESTION BANK

41. Scabies [12, Supple]

42. Diagnosis of Ac. Hep- B [12, Supple]

43. Management of chronic myeloid leukaemia [11]

44. Non-ketotic hyperosmolar diabetic coma [11]

45. Chemoprophylaxis of malaria [11]

46. HAART treatment [11]

47. Tropical eosinophilia [11]

48. MDR tuberculosis [11]

49. M/m of Migraine [11, Supple]

50. Skin manifestation of internal malignancy [11, Supple]

51. Causes of reversible dementia [11, Supple]

52. Manic depression [11, Supple]

53. Iron deficiency anemia [11, Supple]

54. Indication of Insulin therapy in DM [11, Supple]

55. Acute exacerbation of COPD [11, Supple]

56. Addison’s disease [10]

57. Bony complications of Sickle cell disease [10]

58. Complications of Tetanus [10]

59. Management of Community acquired Pneumonia [10]

60. Causes of Goitre [10]

61. Diagnostic work up of SLE [10]

62. Drugs and kidney [10]

COMPILED AND EDITED BY MEHBOOB-UL ISLAM 78


QUESTION BANK

63. T/t of Manic Dpressive Psychosis [10, Supple]

64. Pemphigius Vulgaris – T/t [10, Supple]

65. Value of reticulocyte count [10, Supple]

66. Tension pneumothorax [10, Supple]

67. AC. Dementia [10, Supple]

68. Dwarfism [10, Supple]

69. Causes of Peripheral Neuropathy [10, Supple]

70. Management of acute severe bronchial asthma [09]

71. Clinical features of 3rd Cranial nerve palsy [09]

72. Manic depression (Bipolar disorder) [09]

73. Gastrointestinal manifestation of HIV infection [09]

74. Lepra reaction [09]

75. Management of Cerebral malaria [09]

76. Aetiology and clinical features of megaloblastic anaemia [09]

77. Palpable purpura [09, Supple]

78. OCD [09, Supple]

79. C/f of Chikungunya [09, Supple]

80. DIC [09, Supple]

81. Erythema Nodosum [09, Supple]

82. M/m of herpes simplex encephalitis [09, Supple]

83. M/m of Lepromatous leprosy [09, Supple]

84. Idiopathic thrombocytopenic purpura [08]

COMPILED AND EDITED BY MEHBOOB-UL ISLAM 79


QUESTION BANK

85. Treatment of status epilepticus [08]

86. Opportunistic infections in an AIDS patient [08]

87. DOTS therapy [08]

88. Clinical manifestations of lepromatous leprosy [08]

89. Vertigo [08]

90. Management of a patient with depression [08]

91. Causes of iron deficiency anemia [08, Supple]

92. Antiretroviral therapy [08, Supple]

93. Intracranial haemorrhage [08, Supple]

94. Delirium [08, Supple]

95. T/t of Ac. Pyogenic meningitis [08, Supple]

96. C/f of syphilis [08, Supple]

97. T/t of leprosy [08, Supple]

COMPILED AND EDITED BY MEHBOOB-UL ISLAM 80


QUESTION BANK

SHORT NOTES
(each carrying 4 marks)

Gr- D
1. Coin lesion in chest Xray [14]

2. Demensia [14]

3. C-reactive protein(CRP) [14]

4. Steven Johnson Syndrome [14]

5. Transient Ischaemic Attack (TIA) [14]

6. Sheehan’s Syndrome [14]

7. Acute complications of blood transfusion [14]

8. DPP-4inhibitors: current status [14, Supple]

9. ARDS [14, Supple]

10. Dwarfism [14, Supple]

11. Molluscum contagiosum [14, Supple]

12. T/t of leprosy [14, Supple]

13. Refractory anemia [14, Supple]

14. Schizophrenia [14, Supple]

15. Anti malarial drugs in Falciparum Malaria [13]

16. DPP-4 inhibitors [13]

17. HIV-Post exposure prophylaxis [13]

18. Addisonian Crisis [13]

COMPILED AND EDITED BY MEHBOOB-UL ISLAM 81


QUESTION BANK

19. Diabetic Retinopathy [13]

20. Bronchiectasis [13]

21. Microalbuminuria [13]

22. Horner’s syndrome [13, Supple]

23. Lepra reaction [13, Supple]

24. Diagnosis of hemolytic anemia [13, Supple]

25. C/f of psoriasis [13, Supple]

26. Tropical sprue [13, Supple]

27. Depression- management issues [13, Supple]

28. C/f of diabetic neuropathy [13, Supple]

29. Management of Pyogenic Meningitis [12]

30. Common AIDS defining conditions [12]

31. Steven Johnson Syndrome [12]

32. Management of Migraine [12]

33. Causes of Hyponatremia [12]

34. Diagnosis of Diabetic neuropathy [12]

35. Management of Depression in elderly [12]

36. Subacute thyroiditis [12, Supple]

37. Phobia [12, Supple]

38. Alcohol dependence syndrome [12, Supple]

39. Osmotic diarrhea [12, Supple]

40. Prophylaxis of rabies [12, Supple]

COMPILED AND EDITED BY MEHBOOB-UL ISLAM 82


QUESTION BANK

41. M/m of tuberculous meningitis [12, Supple]

42. Cavitary lesions of the lung [12, Supple]

43. Bell’s palsy [11]

44. Tetany [11]

45. Amoebic liver abscess [11]

46. Dengue haemorrhagic fever [11]

47. Nosocomial pneumonia [11]

48. Generalized anxiety disorder [11], [09]

49. Lepra reaction [11]

50. Hazards of blood transfusion [11, Supple]

51. Chorea [11, Supple]

52. T/t of Psoriasis [11, Supple]

53. Diagnosis of Addison’s Ds. [11, Supple]

54. Opportunistic infections in AIDS [11, Supple]

55. Idiopathic Thrombocytopenic Purpura [11, Supple]

56. Drug abuse [11, Supple]

57. Signs of cortico-spinal tracts [10]

58. Side effects of steroids [10]

59. Causes of Papilloedema [10]

60. Treatment of Hyperkalemia [10]

61. Low backache in females [10]

62. Anti platelet agents [10]

COMPILED AND EDITED BY MEHBOOB-UL ISLAM 83


QUESTION BANK

63. Diagnostic workup of Polyuria [10]

64. Koilonychia [10, Supple]

65. Side effect of chloroquine [10, Supple]

66. Complication of diphtheria [10, Supple]

67. Ac. Loss of Vision in one eye [10, Supple]

68. Lepra reaction [10, Supple]

69. Sildenafil uses [10, Supple]

70. Dermatoid manifestations of diabetes [10, Supple]

71. DOT in tuberculosis [09]

72. Myxoedema coma [09]

73. Wernicke-Korsakoff syndrome [09]

74. Aetiology of Parkinson’s disorder [09]

75. Non-Thrombocytopenic purpura [09]

76. Non-metaststatic complications of bronchogenic carcinoma [09]

77. Tinea versicolor [09, Supple]

78. Diabetic retinopathy [09, Supple]

79. Bence- Jones Protein [09, Supple]

80. Skin manifestation of HIV infection [09, Supple]

81. Dementia [09, Supple]

82. Bell’s palsy [09, Supple]

83. M/m of TB meningitis [09, Supple]

84. Etiology of peripheral neuropathy [08]

COMPILED AND EDITED BY MEHBOOB-UL ISLAM 84


QUESTION BANK

85. Eosinophilia [08]

86. Kidney involvement in diabetes mellitus [08]

87. Anorexia nervosa [08]

88. Secondary syphilis [08]

89. Management of septic shock [08]

90. Clinical features of Parkinsonism [08]

91. Myxoedema Coma [08, Supple]

92. T/t of septic shock [08, Supple]

93. Causes of peripheral neuropathy [08, Supple]

94. C/f of hemolytic anemia [08, Supple]

95. Causes of chronic diarrhea [08, Supple]

96. Complications of cancer [08, Supple]

97. Risk factor of ischemic stroke [08, Supple]

COMPILED AND EDITED BY MEHBOOB-UL ISLAM 85


QUESTION BANK

COMPILED AND EDITED BY MEHBOOB-UL ISLAM 86


QUESTION BANK

LONG QUESTIONS
1. How will you diagnose and treat a case of Unstable Angina? (15) (MC,K) [8th Sem]

2. Define Diabiabetes Mellitus. Classify Diabetes Mellitus. Enumerate acute and


chronic causes of Diabetes Mellitusand outline the management of Diabetic
Ketoacidosis. (3+3+9) (NRS) [8th Sem]

3. What are the causes of Acute dyspnea? How will you approach a case of acute
dyspnea? (15) (CNMC) [8th Sem]

4. Treatment of Snake bite. (15) (CNMC) [8th Sem]

5. Treatment of Organophosphorous poisoning. (15) (CNMC) [8th Sem]

6. Define hypertension. Describe the procedure measuring blood pressure. How do you
evaluate the causes of Secondary hypertension at bedside based on clinical
examination alone. (2+3+5) (RG Kar) [8th Sem]

7. Enumerate the liver function tests. Describe the significance of elevation of


various hepatic enzymes. (4+6) (RG Kar) [8th Sem]

8. Define Acute Severe Asthma. How do you manage a case of Acute Severe Asthma.
(2+8) (RG Kar)) [8th Sem]

9. Define Status Epilepticus. How do you manage a case of Status


Epilepticus?(2+8)(RG Kar) [8th Sem]

10. Mention causes of Generalized lymphadenopathy. Write down the clinical features,
diagnosis and management of Hodgkin’s Lymphoma. (15) (IPGMER) [8th Sem]

11. What is stroke? Mention its clinical types. What are the risk factors of stroke and
its common causes in a young patients? Describe in short the pathophysiology and
outline its management. (15) (IPGMER) [8th Sem]

12. What are the major clinical types of infective Endocarditis and its common
causative organisms? Describe in brief the clinical features and management of
Infective Endocarditis. (15) (IPGMER) [8th Sem]

COMPILED AND EDITED BY MEHBOOB-UL ISLAM 87


QUESTION BANK

13. What is Anemia? What are the common causes of hypochromic microcytic anemia.
How will you diagnose and treat a case of iron deficiency anemia? (15) (IPGMER)
[8th Sem]

14. Define Pneumothorax. Enumerate types of Pneumothorax. Discuss clinical features,


diagnois and management of tension pneumothorax. (20) (Kalyani) [8th Sem]

15. Define heart failure. Enumerate the causes of left ventricular failure. Discuss
clinical features, diagnosis and treatment of left heart failure. (20) (Kalyani) [8th
Sem]

16. Describe patho-physiology of Heart failure. Treatment outline of acute left heart
failure in a case with atrial fibrillation. (BMC) [8th Sem]

17. Discuss systolic murmur in apex with and without radiation. (BMC) [8th Sem]

18. How will you differentiate ulcerative colitis from Crohn’s disease? Give an outline of
diagnosis and management of Ulcerative Collitis. (4+3+3) (BSMC) [8th Sem]

19. Enumerate five common causes of ascites. How will you diagnose and manage a
patient of ascites of 2 months duration. (2+5+8) (MMC) [8th Sem]

20. Briefly write the etiopathogenesis, clinical features and treatment of disease of
myo-neural junction? (3+3+4) (NBMC) [8th Sem]

21. Describe the etiology, clinical features and management of megaloblastic anemia.
(3+3+4) (NBMC) [8th Sem]

22. Describe the diagnostic criteria of SLE. Briefly discuss the classification and
management of lupup nephritis. (NBMC) [8th Sem]

23. Describe comprehensive management of Type 2 Diabetes Mellitus patient.(Kalyani)


[8th Sem]

24. Describe clinical features of thyrotoxicosis. (Kalyani) [8th Sem]

25. Clasiify Acute Glomerulonephritis. What are its complication? (Kalyani) [8th Sem]

26. Enumerate indications of hemodialysis. Briefly discuss complication of hemodialysis.


(Kalyani) [8th Sem]

COMPILED AND EDITED BY MEHBOOB-UL ISLAM 88


QUESTION BANK

27. Define and classify anemia. How will you investigate a case of Chronic hemolytic
anemia to pin point etiopathology? (Kalyani) [8th Sem]

28. Classify acute leukimias. How do you treat Chronic Myeloid Leukaemia? (Kalyani)
[8th Sem]

29. Discuss steps of investigations to confirm etiopathology of Generalized


lymphadenopathy. (Kalyani) [8th Sem]

30. Describe articular and non-articular management of Rheumatoid Arthritis. How will
you diagnose Rheumatoid Arthritis? (Kalyani) [8th Sem]

31. What are the clinical features of Superior Vena caval syndrome. Make a
differential diagnosis. (Kalyani) [8th Sem]

32. Enumerate four anti-hypertensive drugs of different groups detailing their generic
names, mechanism of action, doses and side effects. (Kalyani) [8th Sem]

33. Diagnosis and management of sub-acute infective endocarditis. (10) (BMC) [9th
Sem]

34. Enumerate risk factors of Coronary Artery Disease. Discuss management of Acute
Myocardial Infarction in 1st 6 hour. (2+8) (BSMC) [9th sem] {paper 1}

35. Describe clinical feature, complication and management of Ulcerative Colitis.


(4+2+4) (BSMC) [9th sem] {paper 1}

36. What are the common causes of Aortic Regurgitation. Describe the clinical
features of chronic aortic regurgitation. Give an outline of management of Chronic
heart failure. (4+5+6) (IPGMER) [9th sem]

37. What is acute renal failure? What are its major causes? How will you clinically
assess and manage a patient of ARF? (1+4+4+6) (IPGMER) [9th sem]

38. Outline the diagnostic criteria of diabetes mellitus? Enumerate the chronic
complication of diabetes mellitus. Describe in brief the management of DKA.
(2+5+8) (IPGMER) [9th sem]

COMPILED AND EDITED BY MEHBOOB-UL ISLAM 89


QUESTION BANK

39. Draw a diagram of conduction system of heart. Mention the etiology of irregular
pulse. Outline the management of atrial fibrillation. (3+3+4) (CNMC) [9th Sem]
{Paper 1}

40. How do you approach a case of Polyarthritis? Outline the management of


Rheumatoid Arthritis. (6+4) (CNMC) [9th Sem] {Paper 1}

41. Describe the conducting system of heart with a diagram. Enumerate the different
types of heart block & outline the T/t. (4+2+4) (MC,K) [9th Sem] {Paper-1}

42. What are the common causes of meningitis? How do you investigate a case of
meningitis? (4+6) (NRS) [9th Sem]

43. Define cirrhosis of liver. What are the clinical features of cirrhosis of liver? What
are the complication of cirrhosis of liver? (2+5+3) (NRS) [9th Sem]

44. Define atrial fibrillation. Enumerate the important causes. How will you diagnose
and treat a case of atrial fibrillation? (2+3+5) (KPC) [9th Sem] {Paper-1}

45. Outline the broncho pulmonary segments. Describe the C/f, diagnosis and M/m of
acute severe asthma. (3+4+3) (NBMC) [9th Sem]

46. Outline the pyramidal tract pathway. What are the risk factors for stroke? Briefly
outline the M/m of hemorrhagic sroke. (3+4+3) (NBMC) [9th Sem]

47. Describe the C/f, M/m & complications of acute falciparum malaria. (3+4+3)
(NBMC) [9th Sem]

48. Management and diagnosis of diabetic ketoacidosis. (10) (BMC) [9th Sem] {Paper-2}

49. Enumerate etiologies of pneumonia in different clinical settings. How will you
diagnose and treat community acquired pneumonia? (3+4+3) (BSMC) [9th sem] {paper
2}

50. Describe clinical features and management of Perkinson’s disease. (5+5) (BSMC) [9th
sem] {paper 2}

51. Discuss in brief clinical features, investigations ad treatment of Grave’s Disease


(RG Kar) [9th sem] {Paper 2}

COMPILED AND EDITED BY MEHBOOB-UL ISLAM 90


QUESTION BANK

52. Define epilepsy. Enumerate different types of epilepsy. Describe the management
of status epilepticus. (3+3+4) (CNMC) [9th Sem] {Paper 2}

53. Mention common causes of coma in diabetic patients. How will you diagnose and
manage a case of diabetic ketoacidosis. (3+7) (CNMC) [9th Sem] {Paper 2}

54. Define bronchial asthma. What are the clinical features of poor prognosis in acute
severe asthma? How will you manage such a patient? (2+4+4)

55. Describe the pathophysiology of pneumonia. Write a briefly the etiology, C/f &
M/m of community acquired pneumonia. (4+6) (MC,K) [9th Sem] {Paper-2}

56. When do you suspect a patient to be suffering from DKA? How to manage DKA in
ER? (4+6) (MC,K) [9th Sem] {Paper-2}

57. Calssify seizure disorder. What is status epilepticus. Briefly discuss its M/m. (4+6)
(MC,K) [9th Sem] {Paper-2}

58. Define chronic obstructive pulmonary ds. Enumerate causes of COPD.


Differentiating features of clinicxal bronchitis and empyema. (2+3+5) (KPC) [9th
Sem] {Paper-2}

59. Define heart failure. What are the types of heart failure? Describe the M/m of
acute left ventricular failure. (2+5+8) (Kalyani) [9th Sem] {Paper 1}

COMPILED AND EDITED BY MEHBOOB-UL ISLAM 91


QUESTION BANK

LONG QUESTIONS
(CLINICAL CASES)
1. How will you approach case of hematemesis, melena and treat the case. (15)
(MC,K) [8th Sem]

2. How will you approach case of fever, splenomegaly,recurrent jaundice and treat
the case? (MC,K) [8th Sem]

3. A 20 year old male patient presented with fever with skin rash for 3 days with
history of haematemesis and melena in the emergency room. How will you
approach to treat the case? (15) (NRS) [8th Sem]

4. A 35 year old female is presented with multiple joint pain, mild fever with
swelling of the whole body. How do you investigate this case? (15) (NRS) [8th
Sem]

5. How will you approach acase of paraparesis? (15)(CNMC) [8th Sem]

6. How will you approach a case of polyarthritis? (15)(CNMC) [8th Sem]

7. D/D of acute chest pain and approach to it. (15)(CNMC) [8th Sem]

8. Enumerate the causes of acute abdominal pain. How do you clinically approach
for a diagnosis? How do you manage the patient in the emergency room? (3+4+3)
(RG Kar) [8th Sem]

9. A 15 year aged young boy was admitted with unconsciousnesds, severe


dehydration and hyperventilation since morning. He had a preceding history of
polyphagia, polyuria, increased thirst and weight loss for last 6 weeks. What is
the most likely diagnosis and how will you manage the case? (2+3+5) (RG Kar)
[8th Sem]

10. A 25 years old man presents with chest pain in the right side and fever for 7
days. What are the possible causes? How will you proceed to arrive at diagnosis?
Describe in brief the management of most likely cause. (15) (IPGMER) [8th
Sem]

COMPILED AND EDITED BY MEHBOOB-UL ISLAM 92


QUESTION BANK

11. A male patient(smoker) after a severe bout of cough developed sudden onset
respiratory distress with chest pain. How will you approach for the diagnosis
and treatment of it. (BMC) [8th Sem]

12. A 22 year old male patient presented with fever for 5 days, progressive loss of
consciousness and 3 episodes of convulsion. Discuss the diagnostic approach in
this case. Outline the management of most probable cause in this case. (BSMC)
[8th Sem]

13. A patient of rheumatic heart disease came to the MOPD with fever of three
weeks duration. On examination apart from heart murmurs and tachycardia, she
had significant pallor and mild splenomegaly. How will you investigate the case to
reach the diagnosis. How will you manage? (8+7) (MMC) [8th Sem]

14. A 16 years old male patient referred from a peripheral hospital to MMCH, with
fever of 5 days duration, altered sensorium and jaundice. Name your provisional
diagnosis. How will you confirm the diagnosis? Describe the current treatment
protocol? (5+10) (MMC) [8th Sem]

15. Ram, a 14 year old male was admitted in emergency with history of weight loss
and polyuria for last 10 months. He had low grade fever wityh expectoration for
last 3 days. He was having pain abdomen and deep breathing on presentation.
How will you manage the patient? (10) (NBMC) [8th Sem]

16. Rattan, 56 year old male diabetic, presented to you with high grade fever with
chills for last 14 days and cough and copious foul smelling sputum. What is the
most probabale diagnosis? How would you investigate and treat the patient?
(2+4+4) (IPGMER) [8th Sem]

17. A 32 year old female presents with high rise of temperature, haematuria and
right sided flank pain. Discuss how would you proceed to arrive at a diagnosis?
Give the management plan. (NBMCH) [8th Sem]

18. How do you approach to diagnose a case of severe sudden onset chest pain. (10)
(BMC) [9th Sem] {Paper 1}

19. How do you approach to diagnose a case of Dyspnoea.(10) (BMC) [9th Sem]

20. A 26 year male presented with high fever, progressive deterioration of


consciousness and few episodes of convulsion in last 3 days. How will you

COMPILED AND EDITED BY MEHBOOB-UL ISLAM 93


QUESTION BANK

approach to the case? Discuss management of the most probable diagnosis.


(6+4) (BSMC) [9th sem] {paper 1}

21. A 28 years female presented with progressive pedal oedema, facial puffiness,
arthralgia of multiple joints and red rashes in both cheeks, for last 6 months.
How will you evaluate this case. Outline the treatment of this condition. (6+4)
(BSMC) [9th sem] {paper 1}

22. A 52 years old male attended the emergency with history of chest pain for 3
hours and respiratory distress for 2 hours. He is diabetic, hypertensive and
smoker. He gives history of fever for 3 days and palpitation for 2 years. What
re the possible diagnoses you will suggest? What clinical features you will
search? What minimum investigations will you advise to come at a diagnosis?
(3+4+3) (RG Kar) [9th sem] {Paper 1}

23. A 35 years old male attended the outdoor with history of melena for 5 days. He
gives h/o vomiting infrequently for last 3 months. He also gives h/o anorexia and
weight loss in last 3 months. He is alcoholic and smoker. On further enquiry he
gives history of palpitation and dizziness and shortness of breath in last 3
months. What are the differential diagnoses? What clinical features will you
search for? What routine or special investigation will you suggest to arrive at a
diagnosis? (3+4+3) (RG Kar) [9th sem] {Paper 1}

24. A 35 years old female patient presented with pain in multiple joints for last 3
months. She gives a history of intermittent grade fever with urgency of urine
for last 2 months. On further enquiry she gives h/o hematuria in 2-3 occasions
in last 3 months and swelling of face for last 1 month. She was treated outside.
What are the differential diagnoses? What clinical features will you search for
in general survey or systemic examination? What minimum investigation will you
advise to arrive at a diagnosis? (3+4+3) (RG Kar) [9th sem] {Paper 1}

25. A 30 years old man presented with weakness of both lower limbs. What are the
possible causes? How will you approach to arrive at a diagnosis in this patient?
(5+10) (IPGMER) [9th sem]

26. A 25 year old female complained of tremor, palpitation and weight loss in spite
of good appetite. What are the probable causes here? How do you confirm the
diagnosis and manage the patient? (1+6+8) (IPGMER) [9th sem]

27. A 40 yr old male patient presented with a history of severe epigastric pain with
radiation to the back. The pain gets relieved by sitting upright and leaning

COMPILED AND EDITED BY MEHBOOB-UL ISLAM 94


QUESTION BANK

forward. What is the most likely diagnosis? How will you investigate and manage
the patient? (2+4+4) (CNMC) [9th Sem] {Paper 1}

28. A 20 yr old female presented with generalized swelling of the body. How do you
approach to diagnose th case?(10) (MC,K) [9th Sem] {Paper-1}

29. A 25 year old male presented with fever and polyarthritis. What are the
differential diagnoses? How do you approach to diagnose the case? (10) (MC,K)
[9th Sem] {Paper-1}

30. A 60 yr old male patient presents with chest pain with mild fever and cough.
How do you approach to reach a diagnosis? (10) (NRS) [9th Sem]

31. A 35 year old presented with severe pain inepigastrium and vomiting. What are
the likely causes? How will you approach to diagnose the patient? Outline the
brief M/m plan. (2+4+4) (KPC) [9th Sem] {Paper-1}

32. A 55 yr old female with h/o DM for 10 yrs presented with anasarca. How will
you proceed to diagnose and treat the patient. (5+5) (KPC) [9th Sem] {Paper-1}

33. A male patient of 30 yrs came to your OPD with high fever of 5 days along witn
breathelessness. How do oyu proceed to investigate the case? Briefly outline
the M/m of lobar Pneumonia. (6+9) (MMC) [9th Sem]

34. A 60 yr old male known diabetic and smoker presented with acute central chest
pain. How do you investigate the case? Mention the differential diagnoses. (9+6)
(MMC) [9th Sem]

35. Enumerate the causes of hematemesis. How do you investigate a case of


hematemesis? Briefly outline the M/m of bleeding oesophageal varices? (4+5+6)
(MMC) [9th Sem]

36. How do you approach to diagnose a patient of long standing(more than 3 weeks
duration) fever. (10) (BMC) [9th Sem] {Paper 2}

37. How do you approach to diagnose a case of huge splenomegaly. (10) (BMC) [9th
Sem] {Paper 2}

38. A 16 year old male known diabetic presented with pain abdomen, repeated
vomiting and disorientation for last 3 days. How will you manage this case?(10)
(BSMC) [9th sem] {paper 2}

COMPILED AND EDITED BY MEHBOOB-UL ISLAM 95


QUESTION BANK

39. A 15 year’s female presented with progressive weakness and fatigue, pallor and
swelling of upper part of abdomen. How will you investigate the case. Outline the
management of most probable diagnosis. (7+3) (BSMC) [9th sem] {paper 2}

40. What are the causes of pyogenic meningitis in a person aged >60 years. Write
down the investigation and treatment of acute pyogenic meningitis. (2+4+4) (RG
Kar) [9th sem] {Paper 2}

41. A 40 year old lady presented with generalized lymphadenopathy for several
months. How do you approach in this case to reach a diagnosis and management.
(5+5) (RG Kar) [9th sem] {Paper 2}

42. A 55 yr old hypertensive female presented with sudden bursting headache,


vomiting with gradual loss of consciousness. What is you provisional diagnosis?
Write differential diagnoses. How can you investigate and confirm the
diagnoses? Write M/m plan of sub arachnoid haemorrhage. (1+2+3+4) (KPC) [9th
Sem] {Paper-2}

43. A 21 yr old male has been admitted with low grade fever and generalized
lymphadenopathy. What are the likely causes? How will you proceed to diagnose
the. (3+7) (KPC) [9th Sem] {Paper-2}

44. A 20 yrs old male presented with fever, icterus & altered sensorium. Mention
possible causes. How will you evaluate? Write down M/m of Acute hepatic
failure. (3+6+6) (Kalyani) [9th Sem] {Paper 1}

45. A 20 yr old female patient presented with anasarca for last 2 weeks. Dip stick
examination revealed proteinuria. How will oyu approach & manage the patient?
(8+7) (Kalyani) [9th Sem] {Paper 1}

46. One 50 yrs old chronic smoker male patient presented with cough, sudden onset
left sided chest pain & dyspnoea. What are the probable causes? What
investigation would you do? How would you manage this patient? (3+6+6)
(Kalyani) [9th Sem] {Paper 2}

47. A 20 year old female patient was briught to emergency in a drowsy state with
history of fever& occasional convulsion for last 5 days. Make a differential
diagnoses. How to pinpoint the diagnosis? How will you manage this patient?
(5+5+5) (Kalyani) [9th Sem] {Paper 2}

COMPILED AND EDITED BY MEHBOOB-UL ISLAM 96


QUESTION BANK

48. A 50 yr old male presents with significant weight oss. Mention possible causes.
How will you proceed to investigate & treat him? (5+5+5) (Kalyani) [9th Sem]
{Paper 2}

COMPILED AND EDITED BY MEHBOOB-UL ISLAM 97


QUESTION BANK

SHORT NOTES
1. RPGN(MC,K) [8th Sem]
2. Aspergillous Allergic Pneumonitis(MC,K) [8th Sem]
3. Pulsus Alterans(MC,K) [8th Sem]
4. Extra intestinal Manifestation of IBD(MC,K) [8th Sem]
5. HSP(MC,K) [8th Sem]
6. Renal Osteodystrophy(MC,K) [8th Sem]
7. Hepato Pulmonary Syndrome(MC,K) [8th Sem]
8. Pronator Drift (NRS) [8th Sem]
9. Renal Osteodystrophy (NRS), (IPGMER) [8th Sem]
10. Incretins (NRS) [8th Sem]
11. Anti Retroviral Therapy (NRS) [8th Sem]
12. Renovascular Hypertension (NRS) [8th Sem]
13. Myxedema Coma (NRS) [8th Sem]
14. The Planter (NRS) [8th Sem]
15. Chronic Hepatitis C (NRS) [8th Sem]
16. Reactive Arthritis (NRS) [8th Sem]
17. Anatomy of Internal Capsule (CNMC) [8th Sem]
18. Chest X-ray Findings of Mitral Stenosis (CNMC) [8th Sem]
19. Clinical Features of Aortic Regurgitation (CNMC) [8th Sem]
20. RNTCP Regimen of all 4 category of Tuberculosis (CNMC) [8th Sem]
21. Hepatomegaly (CNMC) [8th Sem]
22. Respiratory Failure (CNMC) [8th Sem]
23. Hepatic Encephalopathy (CNMC), (BSMC) [8th Sem]
24. Neck Veins (CNMC) [8th Sem]
25. Clubbing (CNMC), (NBMC) [8th Sem]
26. Management of Infective Endocarditis (CNMC) [8th Sem]
27. Treatment of Heart Failure (CNMC) [8th Sem]
28. Gold Criteria for COPD (CNMC) [8th Sem]
29. ECG Findings of AML (CNMC) [8th Sem]
30. Portal Hypertension (CNMC) [8th Sem]

COMPILED AND EDITED BY MEHBOOB-UL ISLAM 98


QUESTION BANK

31. Clinical Features of Parkinson’s Disease (CNMC) [8th Sem]


32. Treatment of Mitral Stenosis (CNMC) [8th Sem]
33. Examination of Pulse (CNMC) [8th Sem]
34. Anatomy of Basal Ganglion (CNMC) [8th Sem]
35. Hepatocellular Carcinoma (CNMC) [8th Sem]
36. Treatment of Acute Severe Asthma (CNMC) [8th Sem]
37. Radio femoral Delay (R G Kar) [8th Sem]
38. Acute Gout (R G Kar) [8th Sem]
39. Philadelphia Chromosome (R G Kar) [8th Sem]
40. Difference between P. falciparum and P. vivax trophozoites under microscope (R G Kar)
[8th Sem]
41. Differentiate between Congestive Heart Failure and Superior Mediastinal syndrome (R
G Kar) [8th Sem]
42. Stool for Occult Blood Test (R G Kar) [8th Sem]
43. Dengue Haemorrhagic Fever (R G Kar), (BMC) [8th Sem]
44. Enumerate Causes of Tender Splenomegaly (R G Kar) [8th Sem]
45. Bed side differentiation between Hematemesis and Haemoptysis (R G Kar) [8th Sem]
46. X-linked Genetic Disorders (R G Kar) [8th Sem]
47. Name the Drug for Management of Tuberculosis (R G Kar) [8th Sem]
48. Anti-hypertensive Drugs (R G Kar) [8th Sem]
49. Extra Articular Manifestations of Rheumatoid arthritis (R G Kar) [8th Sem]
50. Bell’s Palsy (R G Kar) [8th Sem]
51. Vertical transmission of HIV (R G Kar) [8th Sem]
52. Treatment of Cerebral Malaria (R G Kar), (BMC) [8th Sem]
53. Exudative Pleural Effusion(R G Kar) [8th Sem]
54. Solitary Pulmonary Nodules (R G Kar), (MMC) [8th Sem]
55. Risk Factors for Stroke (R G Kar) [8th Sem]
56. Trigeminal Neuralgia (R G Kar) [8th Sem]
57. Hypoglycemia (R G Kar), (KPC) [8th Sem]
58. Sub acute Thyroiditis (R G Kar) [8th Sem]
59. Acromegaly (R G Kar) [8th Sem]
60. Deep Vein Thrombosis (R G Kar) [8th Sem]
61. Etiology of Acute Leukemia (R G Kar) [8th Sem]

COMPILED AND EDITED BY MEHBOOB-UL ISLAM 99


QUESTION BANK

62. Aplastic Anemia (R G Kar) [8th Sem]


63. Japanese B Encephalitis (R G Kar) [8th Sem]
64. Hepatorenal Syndrome (IPGMER), (NBMC) [8th Sem]
65. Adrenal Crisis (IPGMER) [8th Sem]
66. Pulsus Paradoxus (IPGMER) [8th Sem]
67. Flapping Tremor (IPGMER) [8th Sem]
68. Dissociated Sensory Loss (IPGMER), (MMC) [8th Sem]
69. Relative Afferent pupillary Defect (IPGMER) [8th Sem]
70. Grave’s Ophthalmopathy (IPGMER) [8th Sem]
71. Bitemporal Hemianopia (IPGMER) [8th Sem]
72. Auscultatory findings of Mitral stenosis (IPGMER) [8th Sem]
73. Kussmaul’s Sign (IPGMER) [8th Sem]
74. Hyperosmolar Non-ketotic Coma (IPGMER) [8th Sem]
75. Pityrisis Versicolor (KPC) [8th Sem]
76. Convulsion Disorder (KPC) [8th Sem]
77. Eye Complications of Diabetes Mellitus (KPC) [8th Sem]
78. Reiter’s Syndrome (KPC) [8th Sem]
79. Management of Variceal Bleeding (KPC) [8th Sem]
80. Complication of Acute Viral Hepatitis (KPC) [8th Sem]
81. Treatment of Status Epilepticus (KPC) [8th Sem]
82. Diagnosis of Aplastic Anemia (KPC) [8th Sem]
83. Portal Hypertension (KPC) [8th Sem]
84. Tetany (KPC) [8th Sem]
85. Schizophrenia (KPC) [8th Sem]
86. Acne (KPC) [8th Sem]
87. Treatment of Thyrotoxicosis (KPC) [8th Sem]
88. Iron Deficiency Anemia (BMC), (MMC) [8th Sem]
89. Renal Replacement Therapy (BMC) [8th Sem]
90. Status Epilepticus (BMC) [8th Sem]
91. LFT and Its Application (BMC) [8th Sem]
92. Bile Acid Induced Gastritis and Diarrhoea (BMC) [8th Sem]
93. Vitamin A Deficiency (BMC) [8th Sem]
94. G B Syndrome (BMC) [8th Sem]

10
COMPILED AND EDITED BY MEHBOOB-UL ISLAM
0
QUESTION BANK

95. Manifestations and Management of OP Poisoning (BMC) [8th Sem]


96. Care of A patient with Dog Bite with Unknown Status (BMC) [8th Sem]
97. Viral Hepatitis- B, Clinical Features, Lab Findings and Management (BMC) [8th Sem]
98. Management of Uncomplicated Malaria (BSMC) [8th Sem]
99. Pre-exposure Prophylaxis of AIDS (BSMC) [8th Sem]
100. Steven Johnson Syndrome (BSMC) [8th Sem]
101. Tinea Corporis (BSMC) [8th Sem]
102. Allergic Angitis and Granulomatosis of Wegner (BSMC) [8th Sem]
103. Myasthenia Graves (BSMC) [8th Sem]
104. Celiac Disease (BSMC) [8th Sem]
105. Water Hammer Pulse (MMC) [8th Sem]
106. Diagnosis of CML (MMC) [8th Sem]
107. Pediculosis Capitis (MMC) [8th Sem]
108. TIA (MMC) [8th Sem]
109. Rheumatoid Nodules (MMC) [8th Sem]
110. Malignant Hypertension (MMC) [8th Sem]
111. Atypical Mycobacteria (MMC) [8th Sem]
112. Clinical Manifestation of OP Poisoning (NBMC) [8th Sem]
113. Paraneoplastic Manifestations of Lung Cancer (NBMC) [8th Sem]
114. Weil’s Disease (NBMC) [8th Sem]
115. Depression (NBMC) [8th Sem]
116. Autosomal Dominant Disease (NBMC) [8th Sem]
117. Kaposi Sarcoma (NBMC) [8th Sem]
118. Third Heart Sound (NBMC) [8th Sem]
119. Bronchial Breath Sound (NBMC) [8th Sem]
120. Hypertonia (NBMC) [8th Sem]
121. Dermatological Manifestations of Cirrhosis (NBMC) [8th Sem]
122. Hand in Rheumatology (NBMC) [8th Sem]
123. Maculo Papular Rash (NBMC) [8th Sem]
124. Generalised Lymphadenopathy (NBMC) [8th Sem]
125. Urine Analysis in Nephrotic Syndrome (Kalyani) [8th Sem]
126. Cachexia (Kalyani) [8th Sem]
127. Zoonosis (Kalyani) [8th Sem]

10
COMPILED AND EDITED BY MEHBOOB-UL ISLAM
1
QUESTION BANK

128. K W Syndrome (Kalyani) [8th Sem]


129. Obesity and Co morbidity (Kalyani) [8th Sem]
130. Adverse Effects of Steroids (Kalyani) [8th Sem]
131. Down Syndrome (BMC) [9th Sem] {Paper 1}
132. Extra articular manifestation of Rheumatoid arthritis. (BMC) [9th Sem] {Paper 1}
133. Hepatic encephalopathy (management only) (BMC) [9th Sem] {Paper 1}
134. Vitamin A deficiency and its management (BMC) [9th Sem] {Paper 1}
135. Nephrotic syndrome (BMC) [9th Sem] {Paper 1}
136. Continuous murmur (BMC) [9th Sem] {Paper 1}
137. Management of a Neurotoxic Snake Bite (BMC) [9th Sem] {Paper 1}
138. Management of hematoxic snake bite (BSMC) [9th sem] {paper 1}
139. Clinical features of Organophosphorous poisoning (BSMC) [9th sem] {paper 1}
140. Diagnostic of Visceral Leishmaniasis (Kala- azar) (BSMC) [9th sem] {paper 1}
141. Treatment of Uncomplicated malaria (BSMC) [9th sem] {paper 1}
142. Arterial Septal Defect (BSMC) [9th sem] {paper 1}
143. Significance of Proteinuria (RG Kar) [9th sem] {Paper 1}
144. Causes of Dwarfism (RG Kar) [9th sem] {Paper 1}
145. Different presentations of Gout (RG Kar) [9th sem] {Paper 1}
146. Type IV hypersensitivity and enumerate the disease associated with such
hypersensitivity (RG Kar) [9th sem] {Paper 1}
147. HLA linkage and .clinical significance (RG Kar) [9th sem] {Paper 1}
148. Different manifestations of portal hypertension (RG Kar) [9th sem] {Paper 1}
149. Acute Nephrotic Syndrome (CNMC) [9th Sem] {Paper 1}
150. Non thrombocytopenic purpura (CNMC) [9th Sem] {Paper 1}
151. Pellagra (CNMC) [9th Sem] {Paper 1}
152. Treatment of kala azar (CNMC) [9th Sem] {Paper 1}
153. Gene therapy (CNMC) [9th Sem] {Paper 1}
154. Gilbert’s syndrome (CNMC) [9th Sem] {Paper 1}
155. Anaphylaxis (CNMC) [9th Sem] {Paper 1}
156. Pulsus paradoxus (MC,K) [9th Sem] {Paper-1}
157. Ig A nephropathy (MC,K) [9th Sem] {Paper-1}
158. CREST (MC,K) [9th Sem] {Paper-1}
159. Topical splenomegaly syndrome (MC,K) [9th Sem] {Paper-1}
160. Disease amenable to gene therapy (MC,K) [9th Sem] {Paper-1}
161. Zolinger Ellison Syndrome (MC,K) [9th Sem] {Paper-1}
162. Beriberi (MC,K) [9th Sem] {Paper-1}
163. Japanese encephelaitis (NRS) [9th Sem]
164. Ebola virus (NRS) [9th Sem]
165. AIDS defining illness (NRS) [9th Sem]
166. Constrictive Pericarditis (NRS) [9th Sem]
167. Reactive arthritis (NRS) [9th Sem]

10
COMPILED AND EDITED BY MEHBOOB-UL ISLAM
2
QUESTION BANK

168. Dengue Haemorrhagic Fever (NRS) [9th Sem]


169. Extraintestinal manifestation of ulcerative colitis (KPC) [9th Sem] {Paper-1}
170. Jones criteria (KPC) [9th Sem] {Paper-1}
171. Vit-D deficiency (KPC) [9th Sem] {Paper-1}
172. Anaphylaxis (KPC) [9th Sem] {Paper-1}
173. T/t of falciparum malaria (KPC) [9th Sem] {Paper-1}
174. T/t of Ankylosing Spondylitis (KPC) [9th Sem] {Paper-1}
175. Gene therapy (KPC) [9th Sem] {Paper-1}
176. Neurotoxic snake bite (MMC) [9th Sem], (NBMC) [9th Sem]
177. Lepra reaction (MMC) [9th Sem]
178. Encephalitis syndrome (MMC) [9th Sem]
179. M/m of falciparum malaria (MMC) [9th Sem]
180. Steven Johnson Syndrome (MMC) [9th Sem]
181. Delirium (NBMC) [9th Sem]
182. Acute nephrotic syndrome (NBMC) [9th Sem]
183. Atopic dermatitis (NBMC) [9th Sem]
184. Hypogonadism (NBMC) [9th Sem]
185. Osteoporosis (NBMC) [9th Sem]
186. Anti epileptics (NBMC) [9th Sem]
187. Management of acute asthma (BMC) [9th Sem]
188. Management of cerebral malaria (BMC) [9th Sem]
189. Atrial fibrillation- etiology, clinical features and management (BMC) [9th Sem]
190. Dysphagia (BMC) [9th Sem]
191. Diagnosis of rheumatic fever (BMC) [9th Sem]
192. Spontaneous bacterial peritonitis (BMC) [9th Sem]
193. Viral encephalitis (BMC) [9th Sem]
194. Diagnosis of Rheumatic Fever (any 3) (BSMC) [9th sem] {paper 1}
195. Amebic Liver Abscess (BSMC) [9th sem] {paper 1}
196. Vitamin D deficiency (BSMC) [9th sem] {paper 1}
197. Rayneaud’s Phenomenon (BSMC) [9th sem] {paper 1}
198. Osteomalacia (RG Kar) [9th sem] {Paper 1}
199. Pellagra (RG Kar) [9th sem] {Paper 1}
200. Neural TB (RG Kar) [9th sem] {Paper 1}
201. Common clinical manifestation of hyperuricemia (RG Kar) [9th sem] {Paper 1}
202. Oncogene and its clinical significance (RG Kar) [9th sem] {Paper 1}
203. Genetic mutation and its causes (RG Kar) [9th sem] {Paper 1}
204. Renal manifestation of SLE (IPGMER) [9th sem]
205. Bell’s Palsy (IPGMER) [9th sem]
206. Adverse reaction of blood transfusion (IPGMER) [9th sem]
207. Spontaneous bacterial peritonitis (IPGMER) [9th sem]
208. AIDS defining conditions (IPGMER) [9th sem]
209. Conversion Reaction (Hysteria) (IPGMER) [9th sem]

10
COMPILED AND EDITED BY MEHBOOB-UL ISLAM
3
QUESTION BANK

210. ENL (IPGMER) [9th sem]


211. Management of Organophosphorous poisoning (IPGMER) [9th sem]
212. Side effects of AT drugs (IPGMER) [9th sem]
213. Wide splitting of 2nd heart sound (CNMC) [9th Sem] {Paper 1}
214. Ig A nephropathy (CNMC) [9th Sem] {Paper 1}
215. Pulsus paradoxus (CNMC) [9th Sem] {Paper 1}
216. Cytokines (CNMC) [9th Sem] {Paper 1}
217. Anion gap- definition and relevance (CNMC) [9th Sem] {Paper 1}
218. Sleep apnoea (CNMC) [9th Sem] {Paper 1}
219. Classification of causes of portal hypertension (CNMC) [9th Sem] {Paper 1}
220. Lupus nephritis (MC,K) [9th Sem] {Paper-1}
221. M/m of chronic Hep-B (MC,K) [9th Sem] {Paper-1}
222. HOCM (MC,K) [9th Sem] {Paper-1}
223. HUS (MC,K) [9th Sem] {Paper-1}
224. Alpha 1 antitrypsin deficiency (MC,K) [9th Sem] {Paper-1}
225. DHF (MC,K) [9th Sem] {Paper-1}
226. Spontaneous Bacterial Peritonitis (MC,K) [9th Sem] {Paper-1}
227. M/m of snake bite (NRS) [9th Sem]
228. SIRS (NRS) [9th Sem]
229. Importance of sputum examination (NRS) [9th Sem]
230. Renal osteodystrophy (NRS) [9th Sem]
231. Severe falciparum malaria (NRS) [9th Sem]
232. Dilated cardiomyopathy (NRS) [9th Sem]
233. Extra articular manifestation of rheumatoid arthritis (KPC) [9th Sem] {Paper-1}
234. Innate immunity (KPC) [9th Sem] {Paper-1}
235. Causes of microscopic & macroscopic hematuria (KPC) [9th Sem] {Paper-1}
236. Proton pump inhibitor – use & adverse effect (KPC) [9th Sem] {Paper-1}
237. Refractory ascites (KPC) [9th Sem] {Paper-1}
238. Diagnosis of Kala azar (KPC) [9th Sem] {Paper-1}
239. Radiologiacal features of mitral stenosis (KPC) [9th Sem] {Paper-1}
240. Post exposure prophylaxis of HIV (MMC) [9th Sem]
241. Obsessive Compulsive Disorder (MMC) [9th Sem]
242. Extra articular manifestation of RA (MMC) [9th Sem]
243. Acute severe asthma (MMC) [9th Sem]
244. Myxedema Coma (MMC) [9th Sem]
245. Blood picture of iron deficiency anemia (NBMC) [9th Sem]
246. Cardiac examination- signs of aortic regurgitation (NBMC) [9th Sem]
247. M/m approach of acute heart failure (NBMC) [9th Sem]
248. Enteropathic arthropathy (NBMC) [9th Sem]
249. T/t of Hansen’s ds (NBMC) [9th Sem]
250. Celiac ds (NBMC) [9th Sem]
251. Acute Mountain Sickness (NBMC) [9th Sem]

10
COMPILED AND EDITED BY MEHBOOB-UL ISLAM
4
QUESTION BANK

252. Lepra reaction (BMC) [9th Sem] {Paper 2}


253. Cutaneous manifestation of SLE (BMC) [9th Sem] {Paper 2}
254. Prevention of Rabies in acase of unknown dog bite of 24 hrs duration (BMC) [9th
Sem] {Paper 2}
255. Management and diagnosis of CML (BMC) [9th Sem] {Paper 2}
256. Diagnosis and management of SAH (BMC) [9th Sem] {Paper 2}
257. Diagnosis of HIV infection (BMC) [9th Sem] {Paper 2}
258. Diabetes insipidus (BMC) [9th Sem] {Paper 2}
259. Post-exposure prophylaxis of HIV infection. (BSMC) [9th sem] {paper 2}
260. Clinical features of cerebellar disorder (BSMC) [9th sem] {paper 2}
261. Direct Observation and Treatment- short course chemotherapy of Tuberculosis
(DOTS) (BSMC) [9th sem] {paper 2}
262. Grave’s Disease- clinical features and diagnosis (BSMC) [9th sem] {paper 2}
263. Idiopathic Thrombocytopenic Purpura (BSMC) [9th sem] {paper 2}
264. Chronic Kalazar treatment (RG Kar) [9th sem] {Paper 2}
265. Percussion and Auscultation findings in left sided pleural effusion (RG Kar) [9th sem]
{Paper 2}
266. Indication of insulin therapy in type 2 DM (RG Kar) [9th sem] {Paper 2}
267. Bells palsy (RG Kar) [9th sem] {Paper 2}
268. Obsessive Compulsive Disorder (RG Kar) [9th sem] {Paper 2}
269. Scabies (RG Kar) [9th sem] {Paper 2}
270. Treatment od ITP (RG Kar) [9th sem] {Paper 2}
271. Bell’s Palsy (CNMC) [9th Sem] {Paper 2}
272. Nosocomial pneumonia (CNMC) [9th Sem] {Paper 2}
273. HAART therapy (CNMC) [9th Sem] {Paper 2}
274. Manic phase of MDP (CNMC) [9th Sem] {Paper 2}
275. Dengue Haemorrhagic Fever (CNMC) [9th Sem] {Paper 2}
276. Guttate Psoriasis (CNMC) [9th Sem] {Paper 2}
277. Examination of Nail (CNMC) [9th Sem] {Paper 2}
278. M/m of lung abscess (MC,K) [9th Sem] {Paper-2}
279. C/f of Grave’s ds (MC,K) [9th Sem] {Paper-2}
280. Seizures in HIV infected patient (MC,K) [9th Sem] {Paper-2}
281. C/f & M/m of Wilson’s ds. (MC,K) [9th Sem] {Paper-2}
282. M/m of Manic Depresive Psychosis (MC,K) [9th Sem] {Paper-2}
283. M/m of ITP (MC,K) [9th Sem] {Paper-2}
284. M/m of Psoriasis (MC,K) [9th Sem] {Paper-2}
285. Thyroid crisis (KPC) [9th Sem] {Paper-2}
286. Hypercalcemia (KPC) [9th Sem] {Paper-2}
287. Manic depressive psychosis (KPC) [9th Sem] {Paper-2}
288. Paraneoplastic syndrome (KPC) [9th Sem] {Paper-2}
289. Tuberculoid leprosy (KPC) [9th Sem] {Paper-2}
290. M/m of Neurotoxic snake bite (KPC) [9th Sem] {Paper-2}

10
COMPILED AND EDITED BY MEHBOOB-UL ISLAM
5
QUESTION BANK

291. M/m of hypoglycemic coma (KPC) [9th Sem] {Paper-2}


292. Hemolytic anemia (BMC) [9th Sem] {Paper 1}
293. Sequale of hepatitis B infection (BMC) [9th Sem] {Paper 1}
294. Management of tetanus (BMC) [9th Sem] {Paper 1}
295. Management of Hemicrania(migraine) (BMC) [9th Sem] {Paper 1}
296. Management of Kala-azar (BMC) [9th Sem] {Paper 1}
297. Thyrotoxicosis (clinical features only) (BMC) [9th Sem] {Paper 1}
298. Bell’s palsy (Definition, clinical features and treatment) (BMC) [9th Sem] {Paper 1}
299. Down Syndrome (BSMC) [9th sem] {paper 2}
300. Utricaria (BSMC) [9th sem] {paper 2}
301. Pemphigius Vulgaris (BSMC) [9th sem] {paper 2}
302. Clinical features of Schigophrenia (BSMC) [9th sem] {paper 2}
303. General Anxiety Disorder (BSMC) [9th sem] {paper 2}
304. Lung Abscess- Clinical features and treatment (RG Kar) [9th sem] {Paper 2}
305. Subclinical Hypothyroidism (RG Kar) [9th sem] {Paper 2}
306. Differential diagnoses of massive splenomegaly (RG Kar) [9th sem] {Paper 2}
307. Diabetic Neuropathy (RG Kar) [9th sem] {Paper 2}
308. Schizophrenia (RG Kar) [9th sem] {Paper 2}
309. Psoriasis (RG Kar) [9th sem] {Paper 2}
310. CML treatment (RG Kar) [9th sem] {Paper 2}
311. Management of Snake bite (CNMC) [9th Sem] {Paper 2}
312. Pontine haemorrahge (CNMC) [9th Sem] {Paper 2}
313. Aleukaemic leukaemia (CNMC) [9th Sem] {Paper 2}
314. DOTS therapy (CNMC) [9th Sem] {Paper 2}
315. G6PD deficiency (CNMC) [9th Sem] {Paper 2}
316. Diagnostic work up for polyuria (CNMC) [9th Sem] {Paper 2}
317. Acute adverse reaction of Blood Transfusion (CNMC) [9th Sem] {Paper 2}
318. Cystic fibrosis (MC,K) [9th Sem] {Paper-2}
319. Severe falciparum malaria (MC,K) [9th Sem] {Paper-2}
320. Myxedema coma (MC,K) [9th Sem] {Paper-2}
321. Anorexia nervosa (MC,K) [9th Sem] {Paper-2}
322. Sickle cell ds. (MC,K) [9th Sem] {Paper-2}
323. Intra cerebral He (MC,K) [9th Sem] {Paper-2}
324. Tertiary syphilis (MC,K) [9th Sem] {Paper-2}
325. Aplastic anemia (KPC) [9th Sem] {Paper-2}
326. Psoeiasis (KPC) [9th Sem] {Paper-2}
327. Dengue haemorrhagic fever (KPC) [9th Sem] {Paper-2}
328. Treatment of Tetanus (KPC) [9th Sem] {Paper-2}
329. Hyponatremia (KPC) [9th Sem] {Paper-2}
330. Toxic epidermal necrolysis (KPC) [9th Sem] {Paper-2}
331. Conversion disorder (KPC) [9th Sem] {Paper-2}
332. Vit A deficiency (Kalyani) [9th Sem] {Paper 1}

10
COMPILED AND EDITED BY MEHBOOB-UL ISLAM
6
QUESTION BANK

333. Pathogenesis & complication of falciparum malaria (Kalyani) [9th Sem] {Paper 1}
334. C/f of Rheumatoid arthritis (Kalyani) [9th Sem] {Paper 1}
335. Sex-linked recissive disorder (Kalyani) [9th Sem] {Paper 1}
336. P-wave abnormalities in ECG (Kalyani) [9th Sem] {Paper 1}
337. Diagnostic criteria of SLE (Kalyani) [9th Sem] {Paper 1}
338. M/m of massive Variceal bleeding (Kalyani) [9th Sem] {Paper 1}
339. Cell mediated immunity (Kalyani) [9th Sem] {Paper 1}
340. Diagnosis of chronic hemolytic anemia (Kalyani) [9th Sem] {Paper 2}
341. VZV(Kalyani) [9th Sem] {Paper 2}
342. Metabolic syndrome (Kalyani) [9th Sem] {Paper 2}
343. Schizophrenia (Kalyani) [9th Sem] {Paper 2}
344. Diagnosis & T/t of Scabies (Kalyani) [9th Sem] {Paper 2}
345. Tic Douloureux (Kalyani) [9th Sem] {Paper 2}
346. Diabetic retinopathy (Kalyani) [9th Sem] {Paper 2}
347. Tension Pneumothorax (IPGMER) [9th sem], (NRS) [9th Sem]

10
COMPILED AND EDITED BY MEHBOOB-UL ISLAM
7
QUESTION BANK

10
COMPILED AND EDITED BY MEHBOOB-UL ISLAM
8
QUESTION BANK

10
COMPILED AND EDITED BY MEHBOOB-UL ISLAM
9
QUESTION BANK

LONG QUESTIONS
Gr- A
1. A primigravida at 34 weeks of gestation with vaginal bleeding attends
hospital emergency. Enumerate the possible causes. How will you diagnose it?
What will be the management of such a case? (2+4+4) [14]

2. What are the indications of induction of labour? What are the different
methods of induction? Write briefly its complication. (3+3+4) [14, Supple]

3. Enumerate the causes of bleeding per vagina in early months of pregnancy.


Discuss the diagnosis of molar pregnancy and its management. (2+4+4) [13]

4. Define antenatal care. Discuss the benefits of antenatal care and outline the
standard antenatal care methods. (2+4+4) [13, Supple]

5. A primigravida of 36 weeks of gestation was admitted with convulsion and a


blood pressure of 160/110 mm of Hg. What is your provisional diagnosis?
How will you manage the case? (2+8) [12]

6. Define ectopic pregnancy. What are the c/f onruptured tubal ectopic
pregnancy? Discuss briefly the different modalities of T/t of unruptured
tubal pregnancy. (2+3+5) [12, Supple]

7. A multi gravid carrying 32 weeks of gestation comes to O & G Emergency


with the c/o painful bleeding per vagina. Mention the diagnosis and outline
the management in short. (4+6) [11]

8. Enumerate the causes of bleeding PV in early months of pregnancy. Discuss


the diagnosis and M/m of threatened abortion. (3+3+4) [11, Supple]

9. A primigravida is admitted at 34 weeks pregnancy with bleeding per vagina.


How will you investigate and manage such a case? (4+6) [10]

10. A primigravida at 34 wks of gestation is brought to hospital with convulsion


and hypertension- discuss the M/m of such a case.(10) [10, Supple]

11
COMPILED AND EDITED BY MEHBOOB-UL ISLAM
0
QUESTION BANK

11. A primigravida had assisted delivery with outlet forceps. She started severe
vaginal bleeding four hours after child birth. Enumerate the causes of this
bleeding. How will you manage such a patient? (2+8) [09]

12. A 26 yrs old primigravida carrying twinshas just delivered the first baby.
Write in brief your subsequent M/m till the second baby is born. What are
the post partum complications of twin pregnancy? (7+3) [09, Supple]

13. Define Pre-eclampsia. What are the diagnostic criteria? What are the
immediate and remote comkplications? Outline a protocol of management of
Eclampsia with Magnesium Sulphate. (2+2+3+3) [08]

14. Describe the symptoms and sign of iron deficiency anemia. How do you
investigate such case? What are the complications of severe anemia in
pregnancy? (2+4+4) [08, Supple]

11
COMPILED AND EDITED BY MEHBOOB-UL ISLAM
1
QUESTION BANK

LONG QUESTIONS
Gr- B
1. Define normal labour. What are the different stages of labour? How will
you monitor the progress of labour? What precautions are taken during
delivery of head in vertex presentation? (2+2+4+2) [14]

2. Define PPH. How would you classify PPH? How would you manage a case of
atonic PPH?(2+3+5) [14, Supple]

3. Discuss the diagnosis and complication of twin pregnancy. Enumerate


briefly the management of twin pregnancy in labour. (3+3+4) [13]

4. Define anemia in pregnancy. Enumerate the causes and complications of


anemia in pregnancy. Dicuss its M/m in a lady with 32 wks of pregnancy.
(2+2+2+4) [13, Supple]

5. Define intrauterine foetal death. Enumerate the causes of intrauterine


foetal death. Outline its management. (2+3+5) [12]

6. A woman is having profuse vaginal bleeding shortly after delivery of


placentaof her 4th child birth. On examination, her uterus feels flabby.
What is your diagnosis? What are the common causes of bleeding shortly
after childbirth? How will you manage this case? How such bleeding can be
prevented? (2+3+2+3) [12, Supple]

7. Define Recurrent Abortion. Outline the investigation in such a case. (2+8)


[11]

8. Discuss the diagnosis and M/m of a case of Rh isoimmunisation from 1st


trimester of pregnancy till delivery. (5+5) [11, Supple]

9. A multigravida is admitted at 32 weeks pregnancy with a haemoglobin


value of 4gm%. Discuss investigation and management of such a case. (3+7)
[10]

11
COMPILED AND EDITED BY MEHBOOB-UL ISLAM
2
QUESTION BANK

10. What are the causes of IUFD? Discuss the diagnosis and M/m of such a
case. (4+3+3) [10, Supple]

11. A primigravida aged 30 years attends your clinic at 32 weeks of gestation


with breech presentation. How will you manage the case till birth of the
baby? What injuries may occur to the mother and the baby during vaginal
breech delivery? (5+5) [09]

12. How will you diagnose IUFD? Outline the management of such a case in a
diabetic primigravida carrying 36 wks of pregnancy. (5+5) [09, Supple]

13. Define habitual abortion. How will you investigate a patient with history of
habitual abortion? Briefly outline the treatment of cervical incompetence
in pregnancy. (2+5+3) [08]

14. Define IUGR. What are the factors that contributeto IUGR? How can you
diagnose such acse and confirm it? (2+3+5) [08, Supple]

11
COMPILED AND EDITED BY MEHBOOB-UL ISLAM
3
QUESTION BANK

SHORT NOTES
(each carrying 5 marks)

Gr- C
1. Retained placenta [14]

2. Lower Uterine Segment [14]

3. Apgar Score [14]

4. Vulval haematoma [14]

5. Importance of USG n obstetrics [14, Supple]

6. Morbidly adherent placenta [14, Supple]

7. Follow-up in molar pregnancy [14, Supple]

8. Importance of ischial spine in obstetrics [14, Supple]

9. Obstetric outlet [13]

10. Partogram [13], [11, Supple]

11. Cord prolapsed [13]

12. Presenting part [13, Supple]

13. Amniotic fluid [13, Supple]

14. Missed abortion [13, Supple]

15. Perinatal transmission of HIV [13, Supple]

16. External Caphalic Version [12]

17. Retained Placenta [12]

18. Maternal complication of multifocal pregnancy [12]

19. Cardiovascular changes in normal pregnancy [12]

20. Prophylaxis against the isoimmunisation [12, Supple]

21. M/m of premature rupture of membrane [12, Supple]

22. M/m of iron deficiency anemia in pregnancy [12, Supple]

23. Bishop’s Score [11]

11
COMPILED AND EDITED BY MEHBOOB-UL ISLAM
4
QUESTION BANK

24. Physiological Anaemia of Pregnancy [11]

25. Induction of Labour [11]

26. Role of ultrasound in early pregnancy [11, Supple]

27. Deep transverse arrest in labour [11, Supple]

28. Screening for Down’s syndrome in pregnancy [10]

29. Non-stress test [10]

30. Polyhydramnos [10]

31. Active M/m of 3rd stage labour [10, Supple]

32. Complications of monochorionic twins [10, Supple]

33. Complete perineal tear [10, Supple]

34. Face presentation [09]

35. Daily foetal movement count beyond 36 weeks of pregnancy [09]

36. Partogram [09]

37. Obstetric outlet of pelvis [09, Supple]

38. Artificial rupture of membrane [09, Supple]

39. Acute puerperal mastitis [09, Supple]

40. Lower Uterine Segment [08]

41. Vulval Haematoma [08]

42. Non-Stress Test [08]

43. Bishop’s score [08, Supple]

44. Dep transverse arrest [08, Supple]

45. HELLP syndrome [08, Supple]

11
COMPILED AND EDITED BY MEHBOOB-UL ISLAM
5
QUESTION BANK

JUSTIFY
(each carrying 5 marks)

Gr- D

1. Maternal mortality is mostly preventable – justify [14]

2. All pregnant women should undergo routine ultrasound in 2nd trimester(18-


20 weeks) – Justify [14]

3. Screening for HIV infection should be done on all pregnant women –


Justify [14]

4. External cephalic version still has got a place in the management of


breech presentation – Comment [14]

5. Manual Vacuum Aspiration is preferable over other methods of


termination of pregnancy in early 1st trimester- justify [14, Supple]

6. Ventouse is preferable over obstetric forceps- comment [14, Supple]

7. All post caesarean pregnancy at term do not require caesarean section to


terminate the pregnancy –comment [14, Supple]

8. Treatment of anemia in pregnancy reduces maternal mortality- justify [14,


Supple]

9. All post caesarean section Pregnancy cases may not be delivered by


caesarean section again- Give reasons [13]

10. Magnesium sulphate is the drug of choice in the treatment of Eclampsia –


Justify [13], [09]

11. Active management of 3rd stage labour should be done in all cases [13]

12. Prophylactic Iron therapy should be given to all pregnant women [13]

11
COMPILED AND EDITED BY MEHBOOB-UL ISLAM
6
QUESTION BANK

13. All vaginal delivery should be conducted bt trained birth attendants [13,
Supple]

14. Eclampsia is a preventable disorder in pregnancy [13, Supple]

15. Ectopic pregnancy may also be managed medically [13, Supple]

16. Not all cases of vaginal delivery need episiotomy [13, Supple]

17. All cases of placenta praevia should be delivered by caesarean section-


Comment [12]

18. Forceps is losing its place to ventouse as a method of instrumental


delivery – Comment [12]

19. Routine ultrasonography in all asymptomatic low risk pregnant women is


not recommended- Justify [12]

20. Vaginal delivery in breech presentation is more dangerous than in vertex


presentation- Justify [12]

21. Emergency obstetric care is one of the most effective strategies for
preventing maternal deaths- explain how [12, Supple]

22. Breast milk is the best food for a newborn- justify [12, Supple]

23. Partograph has reduced perinatal mortality and maternal morbidity


considerably- justify [12, Supple]

24. Routine screening for foetal congenital malformation is a must even in low
risk young primigravidas- comment [12, Supple]

25. How antenatal care can reduce maternal mortality in our country? [11]

26. HIV testing should be done in all pregnant women – Justify [11]

27. Episotomy reduces many of the gynaecological disorder – clarify it [11]

28. Pre-eclampsia is not preventable always whereas Eclampsia is always


preventable- Clarify it [11]

29. Vaginal delivery of breech presentation increases perinatal morbidity and


mortality- how and when? [11, Supple]
11
COMPILED AND EDITED BY MEHBOOB-UL ISLAM
7
QUESTION BANK

30. Breast feeding reduces neonatal morbidity and mortality- discuss in short
[11, Supple]

31. Vaginal birth is possible in post caesarean pregnancy- when and how? [11,
Supple]

32. All pregnant women should be given iron and folic acid. Comment [10]

33. All pregnant women should undergo ultrasound in 2nd trimester [10]

34. All women should be advised about early breastfeeding. Why? [10]

35. Injection betamethasone should be given to all women in preterm labour –


Justify [10]

36. Prophylactic forceps delivery is beneficial in post-caesarean vaginal


delivery- justify [10, Supple]

37. Ergometrine should not be given to prevent PPH in a woman with heart ds-
give reasons [10, Supple]

38. All pregnant women should undergo Hb estimation and blood group
testing- justify [10, Supple]

39. Manual removal of placenta should be under general anaesthesia- give


reasons [10, Supple]

40. All pregnant women should be offered screening for HIV infection in early
pregnancy. Justify [09]

41. Maternal mortality is mostly avoidable – Comment [09]

42. All cases of post-caesarean pregnancy at term should be delivered by


caesarean section- comment [09, Supple]

43. Height of fundus of gravid uterus may not always corroborate with the
period of amenorrhoea- give reasons [09, Supple]

44. Anti- D prophylaxis is amust in a Rh –ve foetus in situatiions of likey


foeto-maternal haemorrhage- justify [09, Supple]

11
COMPILED AND EDITED BY MEHBOOB-UL ISLAM
8
QUESTION BANK

45. Ultrasonography should be as routine antenatal investigation


weeks of pregnancy- comment [09, Supple]

46. Prenatal counseling is a must – Justify [08]

47. External Cephalic version has got a place in management of breech


presentation - critically evaluate [08]

48. Misoprostol has almost replaced othere drugs for pregnancy


termination(MTP) - Comment [08]

49. Twin pregnancy is a high risk pregnancy – Justify [08]

50. Once caesarean section doesn’t mean always a caesaren section – comment
[08, Supple]

51. Most of the maternal deaths are preventable – justify [08, Supple]

52. Management of a case of eclampsia needs special set up- justify [08,
Supple]

53. Follow up of all hydatidiform mole patients should be done – justify [08,
Supple]

11
COMPILED AND EDITED BY MEHBOOB-UL ISLAM
9
QUESTION BANK

12
COMPILED AND EDITED BY MEHBOOB-UL ISLAM
0
QUESTION BANK

LONG QUESTIONS
Gr- A
1. What are the different types of genito-urinary fistula? What are its
causes? How will you diagnose a case of V.V.F.? (2+4+4)[14]

2. Define menopause. What are the clinical problems associated with


menopause? How are those problems addressed? (2+3+5) [14, supple]

3. What are the causes of menorrhagia. Discuss the management of uterine


fibroid. (4+6) [13]

4. What is amenorrhoea? What are the types of amenorrhoea? Discuss the


M/m of PCOD. (1+3+6) [13, supple]

5. Discuss the support of uterus. Enumerate the etiological factors for Pelvic
organ Prolapse. Outline the management of procidentia in a post-menopausal
lady. (4+3+3) [12]

6. A 45 yearold woman presented with irregular vaginal bleeding for one year
duration. Discuss the differential diagnosis. Discuss M/m of early stage
cervical cancer. (4+6) [12, Supple]

7. Define Menorrhagia. What are its causes? How would you manage a case of
Menorrhagia due to fibroid uterus? (2+2+6) [11]

8. Mention the causes of primary amenorrhoea. Mention modern day tools for
diagnostic evaluation of case of primary amenorrhoea. (4+6) [11, Supple]

9. Enumerate causes of Menorrhagia in a 40 year old women. Discuss the


management of such a case with fibroids. (3+7) [10]

10. Enumerate the causes of post-menopausal bleeding per avgina. Describe ethe
M/m of Ca endometrium in a 50 yr old woman. (3+7) [10, Supple]

12
COMPILED AND EDITED BY MEHBOOB-UL ISLAM
1
QUESTION BANK

11. Name the cervical premalignant lesions. How do screen these lesions?
Discuss the FIGO staining of Carcinoma of cervix. (2+3+5) [09]

12. Calssify benign ovarian tumors. What are the clinical features of malignancy
in a patient with ovarian tumour? What is stage I ovarian malignancy
according to FIGO? (5+3+2) [09, Supple]

13. Define post menopausal bleeding. What are the causes? How you will arrive
at a diagnosis in a case of post menopausal bleeding? (1+4+5) [08]

14. Define infertility. What are indirect or presumptive evidences of ovulation


and how they are inferred? What are the direct and conclusive evidence of
ovulation? (2+4+4) [08, Supple]

12
COMPILED AND EDITED BY MEHBOOB-UL ISLAM
2
QUESTION BANK

LONG QUESTIONS
Gr- B
1. What is dysmenorrhoea? How will you differentiate between primary and
secondary dysmenorrhoea? How will you treat a case of primary
dysmenorrhoea? (1+4+5) [14]a lady, 40 yrs of age, presents with a lower
abdominal lump. Enumerate the possible causes. How would you
investigate such a case? (5+5) [14, supple]

2. What are the causes of infertility in female partner. Discuss the


methods of detection of ovulation & how to induce it? (2+4+4) [13]

3. What is cervical intraepithelial neoplasia? How to diagnose premalignant


lesionof cervix? Discuss the M/m of CIN. (2+4+4) [13, supple]

4. A 45 years old lady with a lower abdominal lump complains of pelvic pain.
What are the possible causes? Briefly outline the management of such a
case. (3+7) [12]

5. Enumerate the common site of endometriosis. What are the clinical features of
pelvic endometriosis? Briefly discuss the medical M/m of pelvic endometriosis.
(2+2+6) [12, Supple]

6. Define secondary infertility. What are the causes of secondary


infertility? Outline the investigation of such a case. (2+4+4) [11]

7. Define endometriosis. Mention the common sites of endometriosis.


Outline the treatment options available for pelvic endometriosis. (2+2+6)
[11, Supple]

8. Discuss investigation of a couple with primary infertility. Describe drugs


used for ovulation induction. (5+5) [10]

12
COMPILED AND EDITED BY MEHBOOB-UL ISLAM
3
QUESTION BANK

9. Write briefly the cause of genital prolapse. Discuss the M/m of a case of
uterine prolapse wth cystocele and rectocele in a 35 year old woman (4+6)
[10, Supple]

10. What are the types of urinary incontinence? A primipara aged 22 years
who had forceps delivery following prolonged delivery, complains of
continuous leakage of urine per vaginum which started about 7 days after
child birth. Mention the likely causes and your method of diagnosis of the
condition. (5+1+4) [09]

11. How will you proceed to investigate and treat a case of abnormal uterine
blleding in a parous woman aged 38 years? (5+5) [09, Supple]

12. Define secondary amenorrhoea? What are the pathological causes of


secondary amenorrhoea? Mention the investigations necessary for the
diagnosis of PCOS. (1+4+5) [08]

13. A 14 year old girl attended G&O emergency with the complaint of primary
amenorrhoea, lower abdominal pain, acute retention of urine and lower abdominal
mass. Wghat is the probable diagnosis? How will you investigate the case and
tret her? (2+5+3) [08, Supple]

12
COMPILED AND EDITED BY MEHBOOB-UL ISLAM
4
QUESTION BANK

SHORT NOTES
(each carrying 5 marks)

Gr- C
1. Support of pelvic organs [14]

2. Mucinous cystadenoma [14]

3. Contraindication of OCP [14]

4. Causes of primary amenorrhoea [14]

5. Metrorrhagia [14, supple]

6. Stress urinary incontinence [14, supple]

7. Granulosa cell tumor [14, supple]

8. Findings of normal semen analysis [14, supple]

9. Cervical erosion [13], [10]

10. Dermoid cyst of ovary [13], [09]

11. Cryptomenorrhoea [13]

12. Tubal patency [13, supple]

13. Hormonal control of physiology of menstruation [13, supple]

14. Trichomonal vaginitis [12]

15. Submucous myoma [12]

16. Progesterone only pill (POP) [12]

17. Puberty menorrhagia [12, supple]

18. Bacterial vaginosis [12, supple]

19. LNG-IVS (levonorgestrel containing IVS) [12, supple]

20. Imperforate Hymen [11]

21. Emergency Contraception [11], [13, supple]

22. Ovarian causes of Secondary Amenorrhoea [11]

23. Laparoscopic female sterilization- advantages and disadvantages [11, Supple]

12
COMPILED AND EDITED BY MEHBOOB-UL ISLAM
5
QUESTION BANK

24. Diagnosis of malignant ovarian tumors [11, Supple]

25. Turner’s Syndrome [11, Supple]

26. Methods of 1st trimester MTP [10]

27. Post coital contraceptives [10]

28. Methods of 2nd trimester MTP [10, Supple]

29. Tests of ovulation [10, Supple]

30. Investigations of case of vesico-vaginal fistula [10, Supple]

31. Pelvic part of ureter [09]

32. Complication of intrauterine contraceptive devices [09]

33. Gartner duct cyst [09, Supple]

34. Normal semenogram (WHO) [09, Supple]

35. Emergency contraception [09, Supple]

36. Lymphatic drainage of cervix [08]

37. LNG-IUD [08]

38. Solid tumors of ovary [08]

39. Lymphatic drainage of vulva [08, Supple]

40. Medical management of ectopic pregnancy [08, Supple]

41. CA-125 [08, Supple]

12
COMPILED AND EDITED BY MEHBOOB-UL ISLAM
6
QUESTION BANK

JUSTIFY
(each carrying 5 marks)

Gr- D
1. Laparoscopy is essential in gynaecological practice – Comment [14]

2. Male partner should be investigated first to evaluate an infertile couple –


Justify [14]

3. Vaginal bleeding in post-menopausal woman should always be investigated –


Justify [14]

4. There is no appropriate method for screening carcinoma of ovary –


Comment [14]

5. IUCD is not contraceptive of choice in a newly married woman- justify [14,


supple]

6. Something coming down per vagina does not necessarily mean- it is a case
of genital prolapsed- comment [14, supple]

7. Combined OCP have more benefits than risks- comment [14, supple]

8. Medical therapy is preferred over surgical therapy in the management of


DUB cases- comment [14, supple]

9. Non-contraceptive use of male condoms- Discuss [13]

10. Role of Laparoscopy in diagnosis & management of Endometriosis [13]

11. Methods of choice of second trimester MTP, Justify the methods [13]

12. Tumour Marker has great prognostic value in ovarian malignancy – Justify
[13]

13. PCOS is an endocrine and metabolic disorder – justify [13, supple]

14. Trichomonal vaginitis is sexually transmitted ds – justify [13, supple]

12
COMPILED AND EDITED BY MEHBOOB-UL ISLAM
7
QUESTION BANK

15. Pregnancy can be avoided even following unprotected intercourse- justify


[13, supple]

16. Ovarian conservation during hysterectomy has both advantages and


disadvantages- discuss [13, supple]

17. Cervical Screening can effectively reduce Cancer Cervix – Justify [12]

18. The scope of laparoscopic surgery is enhancing – Comment [12]

19. Combined oral Contraceptive is the best Contraceptive option for newly
married couple – Justify [12]

20. Age of the patient should be considered before undertaking


hysterectomy for benign conditions [12]

21. Dysfunctional uterine bleeding is a diagnosis by exclusion- justify [12,


supple]

22. HRT should be used selectively- justify [12, supple]

23. Laparoscopy is preferable to hysteron salpingography in investigation of


female infertility- comment [12, supple]

24. No scalpel vasectomy should gradually replace tubectomy as a method of


permanent contraception-comment [12, supple]

25. Any case of post-menopausal bleeding should be carefully investigated -


Justify [11]

26. Indication of Dilatation of Curettage’ operation should be neglected –


mention and discuss the indication [11]

27. Analyse the importance of Diagnostic Laparoscopy [11]

28. Hysterectomy is not always necessary to treat DUH- critically evaluate


[11, Supple]

29. Laparoscopy is an important investigation for infertility- justify [11,


Supple]

12
COMPILED AND EDITED BY MEHBOOB-UL ISLAM
8
QUESTION BANK

30. Hormone replacement therapy in a post menopausal woman – mention


advantages as well as disadvantages [11, Supple]

31. All married women should undergo PAP smear examination. Justify [10]

32. Good counseling can increase contraceptive acceptance. How? [10]

33. All women with bilateral tubal block on hysterosalpingography should


undergo diagnostic laparoscopy. Justify [10]

34. Prolapsed of uterus is preventable. How? [10]

35. Laparoscopic ligation is the best method of ligation- critically evaluate [10,
Supple]

36. All women of reproductive age should undergo PAP smear- support [10,
Supple]

37. Emergency contraception should be widely available – justify [10, Supple]

38. All adolescent girl should be given HPV vaccine- justify [10, Supple]

39. Hormone replacement therapy should be advised in all post menopausal


women – critically evaluate [09]

40. Laparoscopy is a better procedure than hysterosalpingography in


evaluation of infertility – justify [09]

41. Method of your choice for second trimester medical termination of


pregnancy – justify your choice [09]

42. Chemotherapy is mainstay of treatment in chorio-carcinoma of uterus –


Justify [09]

43. All cases of infertility should have laparoscopy- justify [09, Supple]

44. Polycystic ovarian syndrome is a diagnostic riddle- comment [09, Supple]

45. Complete perineal tear is d/t mismanaged 2nd stage of labour- comment
[09, Supple]

46. As a sterilization procedure, the ideal site for partial salpingectomy is not
ampulla- comment [09, Supple]
12
COMPILED AND EDITED BY MEHBOOB-UL ISLAM
9
QUESTION BANK

47. Justify the place of H.S.G. in the workup protocol of infertility [08]

48. Early diagnosis of ovarian cancer is still not possible – Give reasons [08]

49. Male partner should be investigated first in case of infertility – Give


reasons [08]

50. Selection of cases must be meticulous before prescribing HRT – Justify


[08]

51. Genito urinary prolapse is a sequel to multiparity – comment [08, Supple]

52. Gynaecological malignancy- screening may or may not be helpful- comment


[08, Supple]

53. Chronic pelvic pain needs proper evaluation [08, Supple]

54. Ovaries need to be preserved during hysterectomy in patients below 45


yrs of age- justification [08, Supple]

13
COMPILED AND EDITED BY MEHBOOB-UL ISLAM
0
QUESTION BANK

13
COMPILED AND EDITED BY MEHBOOB-UL ISLAM
1
QUESTION BANK

LONG QUESTIONS
1. What is induction of Labour? How will you diagnose a case of normal labour
and outline its management? (2+4+4) (MC,K) [8th Sem]

2. What is Primary infertility? How will you proceed to diagnose the causes of
primary infertility? Outline the management of female partner with
ovulation disorder. (1+4+5) (MC,K) [8th Sem]

3. Define Antenatal care. Enumerate the routine investigation that are advised
to a pregnant woman in the antenatal clinic. Discuss in brief the physiological
changes in the cardiovascular system during pregnancy. (2+3+5) (NRS) [8th
Sem]

4. Define Pre eclampsia. Enumerate the complications of severe pre-eclampsia.


Discuss briefly the management of severe pre eclampsia at 37 weeks of
gestation. (2+3+5) (NRS) [8th Sem]

5. Define Menorrhagia. What are the causes of menorrhagia? Outline the


surgical management of uterine fibromyoma. (2+3+5) (NRS) [8th Sem]

6. Discuss the support of uterus. Discuss the management of second degree


uterovaginal prolapsed in a 40 year old woman. (5+5) (NRS) [8th Sem]

7. What are the haematological changes in pregnancy? Mention the


investigations to be done in a pregnant woman with anemia. Write briefly on
parenteral form of therapy in the management of anemia in pregnancy.
(3+4+3) (CNMC) [8th Sem]

8. Define infertility. Mention the common causes of female infertility. Discuss


the methods of detection of ovulation. (1+3+6) (CNMC) [8th Sem]

9. What is pre eclampsia? What are criteria of severe pre eclampsia?Give the
outline of management of severe pre eclampsia. (1+3+6) (CNMC) [8th Sem]

13
COMPILED AND EDITED BY MEHBOOB-UL ISLAM
2
QUESTION BANK

10. What is menorrhagia?what are the causes? Name the medicine s used in the
treatment of menorrhagia in a case of DUB/Medical management of DUB.
(1+3+6) (CNMC ) [8th Sem]/ (2+3+5) (BMC) [9th Sem]

11. Enumerate the benign ovarian neoplasms. Mention the complication of benign
ovarian neoplasms. Write briefly on dermoid cyst. (2+3+5) (CNMC ) [8th Sem]

12. Define PPH.What are the different causes of PPH. Outline the management
of profuse bleeding per vagina in a multigravida who has delivered vaginally
half an hour ago. (3+7+10) (RG Kar) [8th Sem]

13. Define Pelvic Organ Prolapse. Describe the different causes and types of
POP. Outline the management of different types of prolapse.(3+7+10) (RG
Kar) ) [8th Sem]

14. Define PPH. what are the causes of PPH? What are the measures to be
taken to prevent it? How will you manage a case of atonic PPH? (3+3+4)
(IPGMER) [8th Sem]

15. What is infertility? What are the main causes of infertility? How will you
investigate tubal factor infertility? (2+4+4) (IPGMER) [8th Sem]

16. What are the causes of pelvic pain? Briefly discuss the diagnosis and
management of pelvic endometriosis in a 24 year old woman. (3+3+4) (KPC)
[8th Sem]

17. Discuss the complications of pregnancy in a diabetic woman. How do you


manage diabetes in a pregnant woman? (4+6) (BMC) [8th Sem]

18. Define antepartum haemorrhage. Enumerate the causes of APH. What are
the differences between Placenta previa and abruption placenta? How do you
manage a case of placenta previa in a case of 36 weeks of
pregnancy?(2+3+5+10) (BSMC) [8th Sem]

19. What do you mean by pregnancy induced hypertension? How do you diagnose
a case of severe pre eclampsia? How do you manage acase of pre eclampsia
at 34 weeks of pregnancy? Mention the complications of severe pre
eclampsia. (2+3+10+5) (BSMC) [8th Sem]

13
COMPILED AND EDITED BY MEHBOOB-UL ISLAM
3
QUESTION BANK

20. What is genitourinary prolapse? Mention the causes of genitourinary


prolapse. How do you manage a case of second degree uterine prolapse in a
40 year old lady? What are the complications of Hysterectomy? (2+5+10+3)
(BSMC) [8th Sem]

21. Define Pre eclampsia. Enumerate the diagnostic features of pre eclampsia.
What are the common antihypertensive used to treat pre eclampsia and
eclampsia? What is the anticonvulsion of choice in pre eclampsia? Write a
suitable and commonly used regimen of that drug in eclampsia. (1+2+2+1+4)
(MMC) [8th Sem]

22. Define PPH. Classify it. What are the common causes of PPH within half an
hour of child birth? How wil you prevent atonic PPH? (1+2+3+4) (MMC) [8th
Sem]

23. Define infertility. What are the common causes for anovulation? What are
the evidences of ovulation? What methods are used to test tubal
patency?(1+3+3+3) (MMC) [8th Sem]

24. What are the types of fibroid uterus? Enumerate the menstrual symptoms
of fibroid. How will you manage a case of symptomatic fibroid uterus in a 41
year old multiparous woman?(2+3+5) (MMC) [8th Sem]

25. Describe human placenta at term. Write in short thr development of


placenta. Enumerate the protein and steroid hormones secreted by placenta.
What is placenta succenturiata? (3+3+2+2) (BSMC) [9th sem] {paper 1}

26. Enumerate different epithelial ovarian tumors. Discuss the diagnosis and
management of Benign Ovarian tumors. (2+4+4) (BSMC) [9th sem] {paper 2}

27. Define infertility. How do you investigate a case of primary infertility? How
will you induce ovulation? Enumerate the names of different techniques of
Assisted Reproductive Techniques (ART) (3+3+2+2) (BSMC) [9th sem] {paper 2}

28. Define Normal labour . What are the different stages of labour. Outline the
management of 1st stage of labour. (2+2+6) (RG Kar) [9th sem] {Paper 1}

13
COMPILED AND EDITED BY MEHBOOB-UL ISLAM
4
QUESTION BANK

29. What are the different female factors in infertility. Outline the evaluation
of female factors of infertility. (3+7) (RG Kar) [9th sem] {Paper 2}

30. Classify Epithelial Ovarian Carcinoma. Outline the management of a 48 years old
post menopausl lady presenting with heterogenous ovarian lump with ascites. (3+7)
(RG Kar) [9th sem] {Paper 2}

31. Define Primary amenorrhoea. What are the common causes of primary
amenorrhoea? Describe the clinical features and treatment of imperforate
hymen. (2+3+5) (NRS) [9th sem]

32. Describe the latest FIGO staging of Ca Cervix. Describe the management of stage
I Ca Cervix. (5+5) (NRS) [9th sem]

33. Define antepartum Haemorrhage. A 28 yr old P0+0 36 wks of gestation with


hypertension presented at antenatal emergency with pain abdomen and
bleeding PV. What is your possible diagnosis? How will you diagnose the
case? How will you manage the case? (2+1+2+5) (IPGMER) [9th sem]

34. Describe briefly the development of placenta. Mention any type of placental
abnormality with its clinical significance. Describe the methods of placental
delivery. (4+3+3) (IPGMER) [9th sem]

35. Define infertility. Write common causes of female infertility. How will you
investigate the tubal factor for female infertility? (2+4+4) (IPGMER) [9th sem]

36. Mention different types of uterine fibroid. What are the different
secondary changes in fibroids. Outline the management option for a woman
with a symptomatic fibroid uterus. (2+2+6) (CNMC) [9th Sem] {Paper 2}

37. What is CIN and what are the diferent types of CIN? Mention the risk factors for
CIN and cervical cancer. Discuss in brief the diagnosis of CIN. (3+2+5) (CNMC) [9th
Sem] {Paper 2}

38. What is APH? What are the causes? How will you manage a case of APH at 33 wks
GA in labour room? (2+2+6) (CMC) [9th Sem] {Paper 1}

13
COMPILED AND EDITED BY MEHBOOB-UL ISLAM
5
QUESTION BANK

39. What is physiological anemia in pregnancy? Discuss the diagnosis and management
of Iron deficiency anemia in pregnancy and labour? (2+4+4) (CMC) [9th Sem] {Paper
1}
40. What is maternal mortality? What are the causes?how can we reduce the maternal
mortality in our hospital? (2+4+4) (CMC) [9th Sem] {Paper 1}

41. What are the causes of vesico vaginal fistula in our country? How can you diagnose
a case of VVF and outline its treatment? (3+4+3) (CMC) [9th Sem] {Paper 2}

42. Define Post partum haemorrhage. What are the causes of postpartum haemorrhage.
Describe in brief the M/m of PPH. (2+3+5) (KPC) [9th Sem] {Paper-1}

43. Define menopause. What are the c/f of menopause? Outline the M/m of menopause.
(2+4+4) (KPC) [9th Sem] {Paper-2}

44. Define normal labour. What are the stages of labour? How will you monitor the
progress of labour? How will oyu practice active M/m of 3rd stage of labour?
(2+1+4+3) (MMC) [9th Sem]

45. Define antepartum haemorrhage. What are the causes of APH? How will you
clinically differentiate between placenta previa & abruption placenta? When will you
manage placenta previa expectantly and when will you terminate such expectant
management? (2+1+3+2+2) (MMC) [9th Sem]

46. What are the characteristics of normal menstruation? What are the important
causes of the secondary amenorrhoea in young adults? What are the common drugs
used for the induction of ovulation? Describe the use of one such drug in the
treatment of polycystic ovarian syndrome. (2+3+2+3) (MMC) [9th Sem]

47. Define pre eclampsia. Enumerate symptoms and signs of severe pre eclampsia or
imminent eclampsia. Outline the M/m of Eclampsia. (2+3+5) (NBMC) [9th Sem]

48. Outline the M/m of 3rd stage of labour. Enumerate the complication of 3rd
stage of labour. How will you manage the commonest complication of 3rd
stage of labour? (5+2+8) (Kalyani) [9th Sem] {Paper 1}

49. Supports of uterus. Enumerate the types of female Genital prolapse.


Preventive measures in acquired type of genital prolapse. (7+3) (Kalyani) [9th
Sem] {Paper 2}

13
COMPILED AND EDITED BY MEHBOOB-UL ISLAM
6
QUESTION BANK

LONG QUESTIONS
(CLINICAL CASES)
1. One 26 year old pregnant mother (32weeks of GA) admitted in our labour
room with H/o passage of watery discharge per vagina for last 2 days. What
is your provisional diagnosis? How will you confirm your diagnosis and manage
such a case? (1+4+5) (MC,K) [8th Sem]

2. One 32 year old mother P2+2 came to the GOPDwith H/O menorrhagia for
last 8 months. How will you diagnose and manage such acase? (5+5) (MC,K)
[8th Sem]

3. A 30 year old lady presented at the ANC at 34 weeks of pregnancy with


breech presentation. Outline the management of such acase. Mention the
risks of vaginal breech delivery. (7+3) (CNMC) [8th Sem]

4. A 35 year ol lady attended the gynaecology clinic for menorrhagia with


hypogastric lumpof 14 weeks pregnant uterus size. Mention the differential
diagnoses. How would manage the case, had it been a case of fibroid of the
body of the uterus? (3+7) (CNMC) [8th Sem]

5. Define gestational hypertension. Classify hypertensive disorder of


pregnancy. Outline the management of 22 years primigravida attending
emergency at 34 weeks of pregnancy with eclampsia. (3+7+10) (RG Kar) [8th
Sem]

6. Define CIN. Describe different stages of Ca cervix and how is it staged?


Outline the management of a 42 year old lady attending OOD with stage II a
Ca cervix. (3+7+10) (RG Kar) [8th Sem]

7. What is pre eclampsia? A 24 year old primigravida at 32 weeks of gestation


comes to you with BP 160/110. What are the complications she may develop?
How will you manage the case? (2+4+4) (IPGMER) [8th Sem]

13
COMPILED AND EDITED BY MEHBOOB-UL ISLAM
7
QUESTION BANK

8. A 35 year old P1+0 lady presented with menorrhagia of gradually increasing


severity for last 6 months along with a swelling in her lower abdomen. What
is your differential diagnosis? What investigations will you perform? What
are the treatment option you can offer her? (3+3+4) (IPGMER) [8th Sem]

9. A 35 year old woman presented with menorrhagia. Discuss the investigations


and management of such a case. (4+6) (BMC) [8th Sem]

10. A 25 years old primigravida at 33 weeks of gestation presented with


bleeding per vagina. What are the differential diagnoses? Discuss the
management of one of the major causes. (3+7) (BMC)

11. Enumerate the causes of bleeding per vagina early in pregnancy. A woman has
two months of amenorrhoea. She presents at the emergency room with
severe pain abdomen, bleeding per vagina with fainting attack. What is your
provisional diagnosis and how will you manage the case? (2+8) (NBMC) [8th
Sem]

12. A 35 year old patient with no preceeding amenorrhoea has menorrhagoea for
2 months. How will you investiagate the case? Write the medical
management and the minimal access conservative surgeries in this case.
(3+4+3)(NBMC) [8th Sem]

13. One early morning a 30 years old G3 P1 A1 carrying 32 weeks pregnancy


came to obstetrics emergency with profuse bleeding per vaginum. On enquiry
her relative told that she was discovered lying in a pool of blood. What is
your provisional diagnosis? How will you manage the patient? (1+4+5) (Kalyani)
[8th Sem]

14. A 40 year old female came to gynaeOPD with the complain of swelling of
lower abdomen from past 6 months. Enumerate the differential diagnosis.
Describe how can you reach final diagnosis? (3+7) (Kalyani) [8th Sem]

15. A primigravida had assisted delivery with outlet forceps. She started severe
vaginal bleeding four hours after child birth. Enumerate the causes of this
bleeding. How will you manage such a patient? (2+8) (BSMC) [9th sem] {paper 1}

13
COMPILED AND EDITED BY MEHBOOB-UL ISLAM
8
QUESTION BANK

16. Define APH. Outline the management of 27 years old 2nd gravid (Previous
term vaginal delivery) attending emergency with complaints of Painless
Vaginal Bleeding at 32 weeks of gestational age. (3+7) (RG Kar) [9th sem]
{Paper 1}

17. Classify Epithelial Ovarian Carcinoma. Outline the management of a 48 years old
post menopausl lady presenting with heterogenous ovarian lump with ascites. (3+7)
(RG Kar) [9th sem] {Paper 2}

18. Enumerate the causes of anemia in pregnancy and its complications. How will
you manage a woman at 32 wks gestation having Hb level of 6gm/dl? (2+3+5)
(NRS) [9th sem]

19. Define IUGR of fetus. Enumerate causes of IUGR. How will you manage a case of
primigravida with mild IUGR at 33 wks of gestation? (2+3+5) (NRS) [9th sem]

20. A 60 year old menopausal pt came with h/o bleeding PV. What will be the
cause of bleeding? what are the investigation you can do for the patient for
diagnosis? What are the treatment option in your hand for this patient?
(2+4+4) (IPGMER) [9th Sem]

21. A young primi gravida at 36 week of pregnancy was admitted with convulsion.
Enumerate the possible causes of convulsion in this patient. How will you
reach the definitive diagnosis? Write in brief the management of such a
case. (2+2+6) (CNMC) [9th Sem] {Paper 1}

22. A 2nd gravida was admitted in labour and she developed excessive bleeding
pervagina following forceps delivery. What are the likely indications of
forceps delivery in this patient? Enumerate the causes of excessive vaginal
bleeding after delivery of placenta. Describe in brief the management of
such a case. (1+3+6) (CNMC) [9th Sem] {Paper 1}

23. Mrs SM 62 yrs old woman having her menopause 14 yrs ago, presented with
bleeding per vagina for last 7 days. What are the common causes of bleeding
in the case? How can you establish your diagnosis? (3+7) (CMC) [9th Sem]
{Paper 2}

13
COMPILED AND EDITED BY MEHBOOB-UL ISLAM
9
QUESTION BANK

24. Mrs BD 26 yr old infertile woman with h/o menorrhagia for last 10 months,
presented with a farm mass sized 18x16 cm2 in lower abdomen. What is your
provisional diagnosis? How will you confirm your diagnosis and treat the woman?
(1+5+4) (CMC) [9th Sem] {Paper 2}

25. A 26 yr old primigravida was admitted at 30 wks with abdominal pain and
intermittent uterine contractions. What could be the causes? How wil you assess a
woman briefly outline the M/m principles. (2+3+5) (KPC) [9th Sem] {Pape 1}

26. A 20 yr old mother of 4 who is on combined OCP for 2 yearscomplaints of


post coital vaginal bleeding since last 3 months. She got married at the age
of 16. What may be the causes? Describe the possible clinical signs that you
may find. Outline the investigation. How will you treat the woman? (2+2+2+4)
(KPC) [9th Sem] {Paper-2}

27. What are the common causes of lower abdominal lump in women of
reproductive age? How will you clinically distinguish between uterine &
ovarian lump? What are the features suggestive eof malignancy in an ovarian
lump? What are the common types of ovarian malignancy? (2+3+3+2) (MMC)
[9th Sem]

28. Enumerate the causes of white discharge per vagina. Outline the M/m of Pruritic
white discharge per vagina. Enumerate the normal defense mechanism of vagina.
(3+4+3) (NBMC) [9th Sem]

14
COMPILED AND EDITED BY MEHBOOB-UL ISLAM
0
QUESTION BANK

SHORT NOTES
1. Lymphatic Drainage of Female Pelvis (MC,K) [8th Sem]
2. Role of GNRH Analogue in Gynaecology (MC,K) [8th Sem]
3. Surgical Management of PPH (MC,K) [8th Sem]
4. Post Exposure Prophylaxis for HIV (MC,K) [8th Sem]
5. Atonic PPH (NRS) [8th Sem]
6. Complication of Twin Pregnancy (NRS) [8th Sem]
7. Non-contraceptive benefits of OCP (NRS) [8th Sem]
8. Complication of IUCD (NRS) [8th Sem]
9. Physiological Changes in CVS in Pregnancy (CNMC) [8th Sem]
10. Active Management of 3rd stage of labour (CNMC) [8th Sem]
11. CIN- diagnosis and outline of Management (CNMC) [8th Sem]
12. Injectable Contraceptives (CNMC) [8th Sem]
13. Atonic PPH- causes and oxytocicdrugs used in the management (CNMC) [8th Sem]
14. Complication of Abruptio Placentae (CNMC) [8th Sem]
15. Pelvic Part of Ureter (CNMC) [8th Sem]
16. Changes in Fibromyoma of Uterus (CNMC) [8th Sem]
17. Follow up in a Case of H. Mole (RG Kar) [8th Sem]
18. Deep Transverse Arrest (RG Kar) [8th Sem]
19. Post exposure Prophylaxis in HIV (RG Kar) [8th Sem]
20. Medical Management of Fibroid (RG Kar) [8th Sem]
21. Tumour Marker (RG Kar) [8th Sem]
22. Emergency Contraceptives (RG Kar) [8th Sem]
23. Medical Eligibility Criteria for Contraception (IPGMER) [8th Sem]
24. Semen Analysis (IPGMER) [8th Sem]
25. PAP Smear (IPGMER) [8th Sem]
26. Dermoid Cyst (IPGMER) [8th Sem]
27. Cervical Incompetence (IPGMER) [8th Sem]
28. Caput Succedaneum (IPGMER) [8th Sem]
29. Complication of IUFD (IPGMER) [8th Sem]
30. Routine Antenatal USG (IPGMER) [8th Sem]
31. Progestin Only Contraceptives (KPC) [8th Sem]
32. Conservative management of Genital Prolapse (KPC) [8th Sem]
33. Iron Deficiency Anaemia (KPC) [8th Sem]
34. Tubal Patency Test (KPC) [8th Sem]
35. Ovarian conservation during Hysterectomy – discuss (KPC) [8th Sem]
36. Medical Management of Tubal Pregnancy (KPC) [8th Sem]
37. Torsion/Twisted of Ovarian cyst (BMC) [8th Sem] / [9th Sem]
38. Emergency Contraception (BMC) [8th Sem], (MMC) [9th Sem]

14
COMPILED AND EDITED BY MEHBOOB-UL ISLAM
1
QUESTION BANK

39. Anencephaly (BMC) [8th Sem]


40. Missed Abortion (BMC) [8th Sem]
41. RKH Syndrome [9th Sem]
42. Magnesium sulphate [9th Sem]
43. Barrier method of Contraception [9th Sem]
44. Cervical incompetence [9th Sem]
45. Support of uterus (BSMC) [8th Sem]
46. Causes of menorrhagia (BSMC) [8th Sem]
47. Dermoid Cyst Of Ovary (BSMC) [8th Sem]
48. Recurrent Pregnancy Loss (BSMC) [8th Sem]
49. Diagnosis of Pregnancy (BSMC) [8th Sem]
50. HELLP Syndrome (BSMC) [8th Sem]
51. Causes of Anaeemia during Pregnancy (MMC) [8th Sem]
52. Conservative management of Placenta Previa (MMC) [8th Sem]
53. Imperforate Hymen (MMC) [8th Sem]
54. Causes of Menorrhagia (MMC) [8th Sem]
55. Active Management of Third Stage of Labour (NBMC) [8th Sem]
56. Complication of Rh negative Pregnancy (NBMC) [8th Sem]
57. Cryptomenorrhoea (NBMC) [8th Sem]
58. Investigation of Primary Amenorrhoea (NBMC) [8th Sem]
59. Partograph (NBMC) [8th Sem]
60. Causes of PPH (Kalyani) [8th Sem]
61. MgSO4 Therapy in Ecclampsia (Kalyani) [8th Sem]
62. Menorrhagia (Kalyani) [8th Sem]
63. Oral Contraceptive Pills (Kalyani) [8th Sem]
64. Secondary Amenorrhoea (Kalyani) [8th Sem]
65. Active management of 3rd stage of labour (BSMC) [9th sem] {paper 1}
66. Lower uterine segment (BSMC) [9th sem] {paper 1}
67. Bishop’s Score (BSMC) [9th sem] {paper 1}
68. Dermoid Cyst of Ovary (BSMC) [9th sem] {paper 2}
69. Cryptomenorrhagia (BSMC) [9th sem] {paper 2}
70. Different hormonal interplay in the physiology of menstruation (BSMC) [9th sem] {paper
2}
71. Screening of Cancer Cervix (BSMC) [9th sem] {paper 2}
72. Degenerative Changes of fibroid (BSMC) [9th sem] {paper 2}
73. Blood Supply of Uterus (BSMC) [9th sem] {paper 2}
74. Complication of twin pregnancy (RG Kar) [9th sem] {Paper 1}
75. Foetal Complication of Vaginal Breech Delivery (RG Kar) [9th sem] {Paper 1}
76. Forceps- indication and criterias to be fulfilled prior to its application (RG Kar) [9th
sem] {Paper 1}
77. Medical management of endometriosis (RG Kar) [9th sem] {Paper 2}
78. Causes of menorrhagia (RG Kar) [9th sem] {Paper 2}

14
COMPILED AND EDITED BY MEHBOOB-UL ISLAM
2
QUESTION BANK

79. Cryptomenorrhoea (RG Kar) [9th sem] {Paper 2}


80. Cardiovascular changes in normal pregnancy (NRS) [9th sem]
81. Active management of 3rd stage labour (NRS) [9th sem]
82. Benefits of Barrier contraception (NRS) [9th sem]
83. Complication of benign ovarian tumor (NRS) [9th sem]
84. Threatened abortion (IPGMER) [9th sem]
85. Bishop’s score (IPGMER) [9th sem]
86. Support of uterus (IPGMER) [9th sem]
87. Precocious puberty (IPGMER) [9th sem]
88. Early pregnancy bleeding (CNMC) [9th Sem] {Paper 1}
89. Management of severe anemia in pregnancy (CNMC) [9th Sem] {Paper 1}
90. Delivery of 2nd twin (CNMC) [9th Sem] {Paper 1}
91. Management of post dated pregnancy (CNMC) [9th Sem] {Paper 1}
92. Advantages and disadvantages of vaginal hysterectomy (CNMC) [9th Sem] {Paper 2}
93. Trichomonas vaginitis (CNMC) [9th Sem] {Paper 2}
94. Methods tubal patency assessment (CNMC) [9th Sem] {Paper 2}
95. Injectable progestins for contraception (CNMC) [9th Sem] {Paper 2}
96. Oxytocin (CMC) [9th Sem] {Paper 1}
97. Medical management of ectopic pregnancy (CMC) [9th Sem] {Paper 1}
98. Gestational diabetes mellitus (CMC) [9th Sem] {Paper 1}
99. Hormone replacement therapy (CMC) [9th Sem] {Paper 2}
100. Chocolate cyst (CMC) [9th Sem] {Paper 2}
101. Cervical pap smear (CMC) [9th Sem] {Paper 2}
102. Polyhydramnios (KPC) [9th Sem] {Pape 1}
103. Deep transverse arrest (KPC) [9th Sem] {Pape 1}
104. HELLP syndrome (KPC) [9th Sem] {Pape 1}
105. Drugs used in induction of ovulation (KPC) [9th Sem] {Paper-2}
106. Emergency contraception (KPC) [9th Sem] {Paper-2}
107. Role of laparoscopy in gynaecology (KPC) [9th Sem] {Paper-2}
108. Perinatal mortality (MMC) [9th Sem]
109. Pubertal menorrhagia (MMC) [9th Sem]
110. Anti- D immunoglobulin prophylaxis in Rh –ve mother (MMC) [9th Sem]
111. Engagement (NBMC) [9th Sem]
112. Puerperal sepsis (NBMC) [9th Sem]
113. Laboratory parameters of Normal Semen Analysis (NBMC) [9th Sem]
114. IUCD- different timing of insertion (NBMC) [9th Sem]
115. Diagnosis of ectopic pregnancy (Kalyani) [9th Sem] {Paper 1}
116. Cord prolapse in labour near term (Kalyani) [9th Sem] {Paper 1}
117. Pregnancy in grand multipara (Kalyani) [9th Sem] {Paper 1}
118. M/m of Eclampisa (Kalyani) [9th Sem] {Paper 1}
119. Outline the M/m of PROM (Kalyani) [9th Sem] {Paper 1}
120. External Cephalic Version (Kalyani) [9th Sem] {Paper 1}

14
COMPILED AND EDITED BY MEHBOOB-UL ISLAM
3
QUESTION BANK

121. Differential diagnoses of 1st trimester bleeding (Kalyani) [9th Sem] {Paper 1}
122. Complication of fibroid uterus (Kalyani) [9th Sem] {Paper 2}
123. PCOD (Kalyani) [9th Sem] {Paper 2}
124. Ca Cervix – Screening & Early Diagnosis (Kalyani) [9th Sem] {Paper 2}
125. Emergency Contraception (Kalyani) [9th Sem] {Paper 2}
126. Diagnosis of ovulation (Kalyani) [9th Sem] {Paper 2}
127. Semen analysis (Kalyani) [9th Sem] {Paper 2}

14
COMPILED AND EDITED BY MEHBOOB-UL ISLAM
4
QUESTION BANK

JUSTIFY
1. By examining placenta and cord, we can diagnose a number of obstetrical problems
(MC,K) [8th Sem]
2. Pre eclampsia is a preventable disease (MC,K) [8th Sem]
3. Once caesarian section is not always caesarean section (NRS) [8th Sem]
4. All women with APH should be hospitalized (NRS) [8th Sem]
5. Invasive CA of cervix is a preventable disease (NRS) [8th Sem]
6. Investigations of primary infertility should start with semen analysis (NRS) [8th Sem]
7. A woman with molar pregnancy needs proper follow up (CNMC) [8th Sem]
8. MgSO4 is essential in the management of severe pre eclampsia and eclampsia (CNMC)
[8th Sem]
9. HIP vaccine is recommended in the prevention of CA cervix (CNMC) [8th Sem]
10. HSG has got a place in the management of infertility (CNMC) [8th Sem]
11. Endometrium should be properly evaluated in all cases of postmenopausal bleeding
(CNMC) [8th Sem]
12. Husband’s semen analysis is essential in all infertile couple (CNMC) [8th Sem]
13. Once caesarean section always caesarean (RG Kar) [8th Sem]
14. ECV should be done routinely in breech presentation (CNMC) [8th Sem]
15. Third stage of labour should always be managed actively (CNMC) [8th Sem]
16. While investigating infertility male should be evaluated first (CNMC) [8th Sem]
17. If followed correctly LAM is a good contraceptive option (CNMC) [8th Sem]
18. Urinary incontinence should be prior to prolapse management (CNMC) [8th Sem]
19. Ovarian conservation during hysterectomy (KPC) [8th Sem]
20. Medical management of tubal pregnancy (KPC) [8th Sem]
21. Obstetric ultrasound should be routinely done at 18-20 weeks of pregnancy (KPC) [8th
Sem]
22. Cervical cancer is a preventable disease (KPC) [8th Sem]
23. External cephalic version must be attempted in all term breech presentations (BMC)
[8th Sem]
24. All post menopausal bleeding patients must have a D & C (BMC) [8th Sem]
25. All perinatal asphyxia can be prevented (BMC) [8th Sem]
26. Cord blood must be sent for testing in case of Rh negative mother [9th Sem]
27. Laparoscopy is mandatory for evaluation of tubal factor in female infertility [9th Sem]
28. USG in 1st trimester pregnancy [9th Sem]
29. Breast feeding in HIV +ve mother [9th Sem]
30. Partography can improve labour management (MMC) [8th Sem]
31. Availability of effective blood transfusion service can save many mothers’ life (MMC)
[8th Sem]
32. Investigating the male factor in infertility is an important (MMC) [8th Sem]

14
COMPILED AND EDITED BY MEHBOOB-UL ISLAM
5
QUESTION BANK

33. Diuretics should not be used in PET (NBMC) [8th Sem]


34. Magnesium sulphate has drastically reduces the MMR in Eclampsia (NBMC) [8th Sem]
35. Most of the pelvic organ prolapse are preventable (NBMC) [8th Sem]
36. Routine use of Partograph can reduce the incidence of Vesicovaginal fistula (NBMC)
[8th Sem]
37. Maternal mortality is mostly avoidable-comment (BSMC) [9th sem] {paper 1}
38. Non contraceptive use of oral contraception (BSMC) [9th sem] {paper 1}
39. Role of ultrasonography in 1st trimester of pregnancy (BSMC) [9th sem] {paper 1}
40. PPH is a life threatening emergency (RG Kar) [9th sem] {Paper 1}
41. Eclampsia is preventable (RG Kar) [9th sem] {Paper 1}
42. Breast feeding in HIV +ve mother- comment (RG Kar) [9th sem] {Paper 1}
43. Routine screening for CA cervix (RG Kar) [9th sem] {Paper 2}
44. Surgery is not the only option for prolapse management (RG Kar) [9th sem] {Paper 2}
45. Contraception- not the only use of OCP (RG Kar) [9th sem] {Paper 2}
46. HIV screening should be done for all antenatal mothers (NRS) [9th sem]
47. Eclampsia is apreventable disease (NRS) [9th sem]
48. Hysterectomy is the last option for the management of DUH(NRS) [9th sem]
49. HRT is not essential for all post menopausal women (NRS) [9th sem]
50. All antenatal mother should be supplemented with iron to reduce maternal mortality
rate. (IPGMER) [9th sem]
51. Active management of 3rd stage of labour is mandatory for all vaginal delivery
(IPGMER) [9th sem]
52. Semen analysis of male partner should be the first investigation in a case of infertility
(IPGMER) [9th sem]
53. Ovarian cancer is usually diagnosed at a late stage (IPGMER) [9th sem]
54. Lower segment caesarean section has become very safe in modern obstetrics- give
reasons (CNMC) [9th Sem] {Paper 1}
55. Active management of 3rd stage labour should be practice in delivery of all pregnant
woman- justify (CNMC) [9th Sem] {Paper 1}
56. Use of partograph can reduce labour complication- explain (CNMC) [9th Sem] {Paper 1}
57. All molar pregnancy should have post evacuation follow up- comment (CNMC) [9th Sem]
{Paper 1}
58. For pelvic organ prolapse, vaginal hysterectomy is not the only option of management-
discuss (CNMC) [9th Sem] {Paper 2}
59. Pelvic endometriosis is an important cause of female infertility- give reasons (CNMC)
[9th Sem] {Paper 2}
60. Dilatation and curettage is a simple and informative operation- Discuss (CNMC) [9th
Sem] {Paper 2}
61. In the investigation of secondary amenorrhoea many ovarian factors are to be
considered- justify (CNMC) [9th Sem] {Paper 2}
62. Corticosteroids are useful drugs in obstetrics (CMC) [9th Sem] {Paper 1}

14
COMPILED AND EDITED BY MEHBOOB-UL ISLAM
6
QUESTION BANK

63. Antihypertensives should be used judiciously in hypertensive disorders in pregnancy


(CMC) [9th Sem] {Paper 1}
64. After evacuation of hydatidiform mole, proper follow up is essential for management
(CMC) [9th Sem] {Paper 1}
65. Diagnosis of ovarian cancer is always delayed (CMC) [9th Sem] {Paper 2}
66. Hysterosalpingography should be done on D5-D10 of menstruation (CMC) [9th Sem]
{Paper 2}
67. Proper counselling should be done before insertion of IUCD (CMC) [9th Sem] {Paper 2}
68. Termination of pregnancy is the only T/t of severe pre eclampsia- justify the
statement (KPC) [9th Sem] {Pape 1}
69. Determination of the zygocity is important in a twin pregnancy- justify the statement
(KPC) [9th Sem] {Pape 1}
70. Application of forceps is the best option for delayed 2nd stage of labour- criticize the
statement (KPC) [9th Sem] {Pape 1}
71. Once a caesarean a;ways a caesarean- criticize the statement (KPC) [9th Sem] {Paper-1}
72. Both HSG & Laparoscopy are important in the evolution of female sub fertility- justify
the statement (KPC) [9th Sem] {Paper-2}
73. Chemotherapy is the mainstay in the treatment of choriocarcinoma- justify the
statement (KPC) [9th Sem] {Paper-2}
74. Vaginal hysterectomy and pelvic floor repair is the only T/t for genital prolapse-
criticize the statement (KPC) [9th Sem] {Paper-2}
75. Genito-urinary fistula is a preventable condition- justify the statement (KPC) [9th Sem]
{Paper-2}
76. Post menopausal bleeding must be investigated (MMC) [9th Sem]
77. All pregnant women must be counelled for HIV testing during pregnancy (MMC) [9th
Sem]
78. Screening for cervical cancer can reduce mortality due to cervical cancer (MMC) [9th
Sem]
79. All pregnant women should receive iron folic acid supplementation (MMC) [9th Sem]
80. Immune hydrops fetalis is a preventable ds of newborn (NBMC) [9th Sem]
81. Long term follow up is necessary in all women who presents with hydatidiform mole
(NBMC) [9th Sem]
82. Ca Cervix is a sexually transmitted ds (NBMC) [9th Sem]
83. Ovarian malignancy is usually diagnosed late (NBMC) [9th Sem]
84. Lower uterine CS incision is better than classical CS (Kalyani) [9th Sem] {Paper 1}
85. Rh incompatiability in 2nd gravida onwadrds is high risk (Kalyani) [9th Sem] {Paper 1}
86. Antenatal checkup is important for each & every pregnant woman (Kalyani) [9th Sem]
{Paper 1}
87. Uncontrolled Diabetes Mellitus is dangerous in pregnancy state (Kalyani) [9th Sem]
{Paper 1}
88. Partograph is helpful in M/m of every labour case (Kalyani) [9th Sem] {Paper 1}
89. Plane of least pelvic dimension has obstetrics importance (Kalyani) [9th Sem] {Paper 1}

14
COMPILED AND EDITED BY MEHBOOB-UL ISLAM
7
QUESTION BANK

90. Indications & criteria for forceps delivery (Kalyani) [9th Sem] {Paper 1}
91. Obstetric fistula is more common in developing countries than in developed countries.
(Kalyani) [9th Sem] {Paper 2}
92. Ovarian cyst <5 cm in diameter are usually managed conservatively (Kalyani) [9th Sem]
{Paper 2}
93. Endometrial biopsy is mandatory in post menopausal bleeding PV (Kalyani) [9th Sem]
{Paper 2}
94. Straight X-ray abdomen is needed to find out the causes of septic abortion with
peritonitis (Kalyani) [9th Sem] {Paper 2}
95. USG –lower abdomen is needed to find out the cause of recurrent abortion (Kalyani)
[9th Sem] {Paper 2}
96. Laparoscopy is the investigation of choice in endometriosis (Kalyani) [9th Sem] {Paper 2}
97. X-ray of elbow is needed sometimes in case of primary amenorrhoea (Kalyani) [9th Sem]
{Paper 2}
98. Serial USG may be needed in case of M/m of female infertility (Kalyani) [9th Sem]
{Paper 2}

14
COMPILED AND EDITED BY MEHBOOB-UL ISLAM
8
QUESTION BANK

14
COMPILED AND EDITED BY MEHBOOB-UL ISLAM
9
QUESTION BANK

15
COMPILED AND EDITED BY MEHBOOB-UL ISLAM
0
QUESTION BANK

LONG QUESTIONS
Gr- A
1. Describe the formation and circulation of CSF in brain. Describe the clinical
features of raised intracranial pressure of a 6 year old child. Give an outline
of treatment of such a patient. (3+3+4) [14]

2. Define hemostasis. Describe the physiology of hemostasis. Write a brief


note on bleeding disorder. (2+4+4) [14, Supple]

3. Pathophysiology of large VSD and the future changes in haemodynamics if


remain untreated. Mention the complication of VSD. Outline the treatment
of VSD with CCF. (5+3+2) [13]

4. Describe the C/f & M/m of minimal change nephrotic syndrome. (5+5) [13,
Supple]

5. Discuss briefly the synthesis of Thyroid hormones. Outline the clinical


features and treatment of Cretinism. (5+3+2) [12]

6. How the tone of child is maintained? What are the causes of hypotonic
child? Write the M/m of GB syndrome. (4+3+3) [12, Supple]

7. Describe formation, circulation, absorption and composition of CSF of a


normal child. Give outline of management of Tubercular Meningitis of 2 year
old child. (1.5+1.5+1.5+1.5+4) [11]

8. Write down the pathway of Vit-D metabolism. Pathological change in bone in


nutritional rickets. (5+5) [11, Supple]

9. Outline the Metabolism of iron in the body. Compare the laboratory findings
of iron deficiency Anaemia and Thalassemia. (4+6) [10]

10. Define PEM and write two classifications of PEM. Write the steps of M/m of
a case of severe PEM presenting with Ac. GE with some dehydrations. (5+5)
[10, Supple]

15
COMPILED AND EDITED BY MEHBOOB-UL ISLAM
1
QUESTION BANK

11. Give an outline of bilirubin metabolism. Describe the underlying mechanism


for physiological jaundice in the newborn. (6+4) [09]

12. Describe the physiology of human laceration. Briefly mention the anti-
infective properties of breast milk. (6+4) [09, supple]

13. What is glomerular filtration? How it is affected in Ac. PSGN? Describe the
underlying pathogenic mechanism for the clinical picture of PSGN? (1+3+6)
[08]

14. What is balanced diet? What are the constituents of balanced diet?
Describe dietary source, absorption and function of Vit- A. (1+3+6) [08,
Supple]

15
COMPILED AND EDITED BY MEHBOOB-UL ISLAM
2
QUESTION BANK

BRIEF NOTES
Gr- B
1. Hypoglycemia in neonate [14]

2. Hemorrhagic disease of new born [14], [11, Supple]

3. Neonatal seizure [14, Supple]

4. Acute constrictive peroicarditis [14, Supple]

5. Enumerate the vaccines that can be given to an unimmunized 2 year old child [14]

6. Kangaroo Mother Care [13]

7. Clinical features and laboratory investigation of Dengue fever in children [13]

8. Neonatal Sepsis screening [13], [14, Supple]

9. Physiological jaundice [13, Supple]

10. Cerebral malaria and complication [13, Supple]

11. Management of Cyanotic Spell in Tetralogy of Fallot [12]

12. Prevention of Hypothermia in newborn [12]

13. Diagnosis of Rheumatic Arthritis [12]

14. Assessment of gestational age in newborn [12, Supple]

15. Kernicterus [12, Supple]

16. Bell’s palsy [11]

17. Kangaroo-mother care [11]

18. Sepsis screen of newborn [11]

19. Mongolian spot [11, Supple]

20. WHO classification of malnutrition [11, Supple]

21. Breast milk versus Cow’s milk [10]

22. Developmental milestone of one year old child [10]

23. Urinary findings of acute post streptococcal glomerulonephritis [10]

24. Radiological changes of rickets and scurvy [10, Supple]

15
COMPILED AND EDITED BY MEHBOOB-UL ISLAM
3
QUESTION BANK

25. Complications of Measles [10, Supple], [12, Supple]

26. C/f & T/t of Ac. Otitis media [10, Supple]

27. Causes of failure of breast feeding [09]

28. Routine care of newborn in the delivery room [09]

29. Clinical features and treatment of scabies in children [09]

30. Lose stools in newborn [09, Supple]

31. Hypothermia in newborn [09, Supple]

32. Complication of falciparum malaria [09, Supple]

33. Give an outline of the clinical feature of neonatal sepsis [08]

34. Describe briefly the complications of LBW babies [08], [13, Supple]

35. Write in short the indication and complication of phototherapy [08]

36. Advantages of breast feeding [08, Supple]

37. C/f and M/m of Haemorrhagic ds of newborn [08, Supple]

38. Causes and M/m of hypothermia in newborn [08, Supple]

15
COMPILED AND EDITED BY MEHBOOB-UL ISLAM
4
QUESTION BANK

SHORT NOTES
Gr- C
1. Modified Jones Criteria [14]

2. Pneumatocele [14]

3. Common causes and laboratory diagnosis of iron deficiency anaemia in children


[14]

4. Causes and clinical features of hyponatremia [14]

5. Status epilepticus- causes and management [14, Supple]

6. ARDS [14, Supple]

7. Diet of 1 yr old infant [14, Supple]

8. Utility of growth chart [13]

9. Rabies Prophylaxis [13]

10. Miliary tuberculosis in children [13]

11. Pulse polio immunization [13]

12. Common causes of hypertension in children [13, Supple]

13. Early predictor of cerebral palsy [13, Supple]

14. Radiological features of active and healing rickets [13, Supple]

15. Developmental milestones of 2 yr old child [13, Supple]

16. Pulsus- paradoxus [12]

17. Complication of acute glomerulonephritis [12]

18. Biochemical changes in Rickets [12]

19. Milestones of development in a one year old child [12]

15
COMPILED AND EDITED BY MEHBOOB-UL ISLAM
5
QUESTION BANK

20. Neonatal transport [12, Supple]

21. Bronchodilator aerosol [12, Supple]

22. Clinical features of Chikungunya fever [12, Supple]

23. WHO classification of PEM [12, Supple]

24. Laboratory diagnosis of Acute glomerulonephritis [11]

25. Febrile convulsion [11]

26. Diet of one year old infant [11]

27. Head-circumference [11]

28. Low osmolar ORS [11, Supple]

29. T/t of Congestive cadiac failure in children [11, Supple]

30. Complication of nephritic syndrome [11, Supple]

31. Persistent diarrhea in children [11, Supple]

32. Transient Tachypnea of new born [10]

33. Clinical presentation and management of Acute Bronchihtis [10]

34. OPV vs IPV [10]

35. Phototherapy [10]

36. BCG [10, Supple]

37. Anterior fontanelle [10, Supple]

38. Cyanotic soell [10, Supple]

39. Breath holding spell [10, Supple]

40. Rheumatic chorea [09]

41. Laboratory evaluation of Ascites of a child [09]

42. Tuberculin test [09]

43. Vascular purpura [09]

15
COMPILED AND EDITED BY MEHBOOB-UL ISLAM
6
QUESTION BANK

44. Simple febrile convulsion [09, Supple]

45. CSF changes in TB meningitis [09, Supple]

46. C/f of acute bronchiolitis [09, Supple]

47. Milestones of development in a child of one year [09, Supple]

48. Dietary management of severe PEM [08]

49. Congenital hypothyroidism [08], [14, Supple]

50. C/f of pyogenic meningitis in children [08]

51. Complications of VSD [08]

52. BCG vaccination [08, Supple]

53. Modified Jones criteria for Rheumatic fever [08, Supple]

54. Acute laryngotracheobronchitis [08, Supple]

55. Laboratory diagnosis of childhood nephritic syndrome [08, Supple]

15
COMPILED AND EDITED BY MEHBOOB-UL ISLAM
7
QUESTION BANK

LONG QUESTIONS
Gr- D
1. A 6-year-old girl child is admitted with hematuria, moderate edema and
headache. Mention the possible differential diagnosis. How will you
evaluate such a case to reach to a definite diagnosis. (2+6) [14]

2. A 4 yr old child presented with fever, pallor, gum bleeding and 1.5cm
palpable spleen. Mention the d/d of the case & investigate to confirm the
diagnosis. (3+5) [14, Supple]

3. A 4 years old child prescribed with h/o fever for 7 days and recurrent
convulsion for last two days and headache. How will you proceed for
diagnosis clinically and by laboratory investigations? (4+4) [13]

4. A 3 yr old child presented with cough & cold for 5 days & respiratory
distress for 5 days. On examination child look toxic & drowsy with head
nodding & central cyanosis. What is the diagnosis as per WHO criteria?
How will oyu manage the case? (1+7) [13, Supple]

5. A 4 year old child presented with pallor, fever, gum bleeding and 1.5 cm
palpable spleen. Mention the diagnostic possibility and investigations to
confirm diagnosis. (3+5) [12]

6. A 3 yr old child with generalized seizure for >45 mins. What is your
provisional diagnosis? How will you manage the case? (1+7) [12, Supple]

7. A two year old child has presented with fever for 20 days. Examination
revealed severe pallor, hepato-splenomegaly and purpuric spots all over
the body. (a) Write the differential diagnosis. (b)Suggest
investigation to reach the final diagnosis. (5+3) [11]

8. A child presented with respiratory distress for last 2 days. On


examination, patient looks toxic and there is dullness on the right side pof
chest. Name 2 imp cause for this condition. How do you proceed for
diagnosis of the case. (2+6) [11, Supple]

15
COMPILED AND EDITED BY MEHBOOB-UL ISLAM
8
QUESTION BANK

9. A seven year old with History of exchange transfusion in neonatal period,


presents with Haematemesis, Physical examination is unremarkable except
for splenomegaly (6cm). what is your differential Diagnosis? Describe
steps in the management of this child. (1+7) [10]

10. A 2 yr old child withnose and cough for 2 days, develops fever followed by
GTCS. Outline your approach for M/m of seizures in the emergency and
what advice will you give to parents of this child. (8) [10, Supple]

11. A four year old child has been brought to the emergency with convulsion
persisting for more than 30 minutes. How you will diagnose the case?
Briefly narrate the management of the case. [09]

12. One eight year old child presented with fever, chest pain and respiratory
distress for one week. On examination the child is febrile, toxic and
respiratory rate 60/min, trachea shifted to the left and stony dullness on
right side of chest on percussion. What is your provisional diagnosis? How
are you going to manage this case? (1+7) [09, Supple]

13. A 4 yr old boy was presented at the emergency room with acute onset of
cough and respiratory distress. He has no fever. His father also suffers
from recurrent episodes of similar problem. The child was admitted 3
times with similar complications in the preceeding 1 year. What is your
most probable diagnosis? How will you manage this condition? (1+7) [08]

14. A 2 yr old boy presented with acute onset weakness of both lower limbs.
Discuss the differential diagnosis. What is AFP surveillance? (4+4) [08,
Supple]

15
COMPILED AND EDITED BY MEHBOOB-UL ISLAM
9
QUESTION BANK

16
COMPILED AND EDITED BY MEHBOOB-UL ISLAM
0
QUESTION BANK

LONG QUESTIONS
Gr- A
1. Define Nephrotic Syndrome. How do you investigate and manage a case of 1st attack
of Minimal Change Nephrotic Syndrome. (2+8) (MC,K) [8th Sem]

2. Discuss Bilirubin Metabolism in relation to physiological jaundice in newborn.


Mention the Criteria of Pathological Jaundice. (7+3) (NRS) [8th Sem]

3. 4 year old girl presented with haematuria of 1st week duration. Discuss the possible
differential diagnosis and management of UTI. (4+4) (CNMC) [8th Sem]

4. Enumerate the coagulation cascade(pathway). Describe the clinical pictures of


haemophilia and its management. (4+4) (CNMC) [8th Sem]

5. Describe in short Vitamin D metabolism. (12) (RG Kar) [8th Sem]

6. Outline the investigations and management of a case of Rickets. (6+6) (RG Kar) [8th
Sem]

7. Outline briefly the metabolic interaction of Vitamin D, calcium and parathormone.


Write the treatments of nutritional rickets. (6+4) (IPGMER) [8th Sem]

8. Enumerate the external ocular muscles innervated by 3rd Cranial nerve and action of
each muscle. A patient gets diplopia when looking towards nose. Which muscle is
involved? What are the causes of 3rd Nerve Palsy? (6+1+3) (KPC) [8th Sem]

9. Enumerate the causes of congestive cardiac failure in children. Outline the


management of Congestive Cardiac Failure. (5+5) (BMC) [8th Sem]

10. Mention the causative organisms of acute viral hepatitis in children. Complication of
acute viral hepatitis and its management. (5+5) (BMC) [9th Sem]

11. Describe the formation, circulation and absorption of Normal CSF along with its
complication in children. Give the outline of management of tuberculosis meningitis?
(6+4) (MMC) [8th Sem]

16
COMPILED AND EDITED BY MEHBOOB-UL ISLAM
1
QUESTION BANK

12. Enumerate Common causes, describe the clinical features and outline the
management of heart failure in children. (2+4+4) (NBMC) [8th Sem]

13. How to assess growth and development in a one year old baby. (4+4) (Kalyani) [8th
Sem]

14. Write in brief about pathgogenic basis of major clinical manifestation of Rheumatic
Fever. Give an outline of Pharmacologic management of Congestive Cardiac Failure.
(5+5) (BSMC) [9th sem]

50. Define Anemia. Classify anemia according to RBC morphology citing examples.
Describe the peripheral blood pictures of chronic hemolytic anemia. (2+5+3) (RG
Kar) [9th sem]

51. Discuss Bilirubin Metabolism in relation to physiological jaundice in newborn.


Mention the criterias of pathological jaundice. (7+3) (NRS) [9th sem]

52. Outline the mechanism of hemostasis in children. Briefly write the management of
recently diagnosed of acute ITP. (5+5) (IPGMER) [9th sem]

53. Discuss about physiology of coagulation. Outline the management of ITP.


(5+5) (CNMC) [9th Sem]

54. Describe bilirubin etabolism. Enumerate the mechanism of action and side effects
of phototherapy. (4+4+2) (CMC) [9th Sem]

55. Describe in brief the bilirubin metabolism. A 7 day old term newborn is having
jaundice. The serum bilirubin level is 15 mg/dl (predominantly unconjugated).
Enumerate the possible causes. How will you manage the baby? (6+1+3) (KPC) [9th
Sem]

56. Outline the structure of a glomerulus and enumerate functions of each part of it.
Give the outline of M/m of diarrhea with severe dehydration. Write the
composition of low osmolar ORS. Write the National Immunization Schedule upto 1
yr of age. (2+2+2+2+2) (MMC) [9th Sem]

57. Describe hemodynamics of a large VSD in children. Describe the management of an


infant presenting with VSD with cardiac failure. (5+5) (NBMC) [9th Sem]

16
COMPILED AND EDITED BY MEHBOOB-UL ISLAM
2
QUESTION BANK

58. Enumerate the steps of hemoglobin synthesis. Briefly mention the


pathophysiology of thalassemia. How to approach for a 12 month old boy
presenting with severe pallor & splenomegaly. (2+3+5) (Kalyani) [9th Sem]

16
COMPILED AND EDITED BY MEHBOOB-UL ISLAM
3
QUESTION BANK

SHORT NOTES
Gr- B & C
1. Haemorrhagic disease of Newborn (MC,K) [8th Sem]

2. Acute Encephalitic Syndrome (MC,K) [8th Sem]

3. Severe Malaria (MC,K) [8th Sem]

4. Management of Hypothermia in newborn (MC,K) [8th Sem]

5. Management of Cyanotic Spell (MC,K), (MMC) [8th Sem]

6. Complication of SAM (MC,K) [8th Sem]

7. Typhoid Vaccine (MC,K) [8th Sem]

8. Steps of Management of Severe Acute Malnutrition (NRS) [8th Sem]

9. Organ Involvement in Perinatal Asphyxia (NRS) [8th Sem]

10. Developmental Milestones of 3 yrs old Child (NRS) [8th Sem]

11. Neonatal Hypoglycaemia (NRS), (CNMC) [8th Sem]

12. Signs of Vitamin D deficiency (NRS) [8th Sem]

13. Hyponatremia (NRS) [8th Sem]

14. Severe Acute Malnutrition (CNMC) [8th Sem]

15. Developmental Achievement at 1 yr of age (CNMC) [8th Sem]

16. Clinical Manifestations of Vit- A deficiency (CNMC) [8th Sem]

17. Complication of Acute Glomerulonephritis (CNMC) [8th Sem]

18. Management of hypothermia in new born (CNMC) [8th Sem]

19. Hyaline Membrane Disease (CNMC) [8th Sem]

20. Sepsis Screen (CNMC) [8th Sem], (NBMC) [9th Sem]

21. Phototherapy (CNMC) [8th Sem], (NBMC) [9th Sem]

16
COMPILED AND EDITED BY MEHBOOB-UL ISLAM
4
QUESTION BANK

22. Turner Syndrome (CNMC) [8th Sem]

23. Hemolytic Uremic Syndrome (CNMC) [8th Sem]

24. Polio Vaccine (CNMC) [8th Sem]

25. Bone Age Estimation (CNMC) [8th Sem]

26. Clinical features and management of Congenital Hypothyroidism (CNMC) [8th Sem]

27. Thermoregulation of New Born (CNMC) [8th Sem]

28. Coarctation of Aorta (CNMC) [8th Sem]

29. Auto immune Hemolytic Anemia (CNMC) [8th Sem]

30. Criteria of Clinical Staging of Asthma in Children (CNMC) [8th Sem]

31. Haemorrhagic Disease of New born (RG Kar) [8th Sem]

32. Feeding of Low Birth Weight New born (RG Kar) [8th Sem]

33. Treatment of Status Epilepticus (RG Kar) [8th Sem]

34. Management of Cyanotic Spell in a Congenital Heart disease (RG Kar) [8th Sem]

35. Steroid dependent Nephrotic Syndrome (RG Kar) [8th Sem]

36. Rh Isoimmunisation (IPGMER) [8th Sem]

37. Causes of Respiratory Distress in New born (IPGMER) [8th Sem]

38. Colostrum (IPGMER) [8th Sem]

39. Hemodynamics of Large VSD (IPGMER) [8th Sem]

40. Evaluation of Urinary Tract Infection (IPGMER) [8th Sem]

41. Simple Febrile Convulsion (IPGMER) [8th Sem]

42. Prognostic Factors in ALL (IPGMER) [8th Sem]

43. Investigation of 1 year old child with clinical Diagnosis of Thalassemia Major (KPC)

[8th Sem]

44. Management of a Child Wth a Chloroquine resistant Falciparum Malaria (KPC) [8th

Sem]

45. Category 1 therapy of 1 year old child with Tuberculosis (KPC) [8th Sem]

16
COMPILED AND EDITED BY MEHBOOB-UL ISLAM
5
QUESTION BANK

46. Xerophthalmia (KPC) [8th Sem]

47. Modified Jone’s Criteria of Rheumatic Fever (KPC), (NBMC) [8th Sem]

48. Mantoux Test (KPC) [8th Sem]

49. Laboratory Diagnosis of Neonatal Sepsis (KPC) [8th Sem]

50. Outline the Physiology of Water Balance (BMC) [8th Sem]

51. Feeding of a 7 months old Baby (BMC) [8th Sem]

52. Vitamin D3 deficiency in Children (BMC) [8th Sem]

53. Utility of WHO Growth Chart (BMC) [8th Sem]

54. Prevention of Acute Rheumatic Fever (BMC) [8th Sem]

55. Management of Severe Dehydration in a PEM Child (BMC) [8th Sem]

56. Treatment of Cerebral Malaria (BMC) [8th Sem]

57. Feeding of LBW baby (BMC) [9th Sem], (Kalyani) [9th Sem]

58. Neonatal hypothermia (BMC) [9th Sem]

59. Physiological jaundice (BMC) [9th Sem]

60. Chelation therapy in thalassemia (BMC) [9th Sem]

61. D/D of Fallot’s tetralogy (BMC) [9th Sem]

62. Jones criteria (BMC) [9th Sem]

63. Management of empyema thoracis (BMC) [9th Sem]

64. Indication and Contraindication of Bag mask Ventilation (BSMC) [8th Sem]

65. AFP Surveillance (BSMC), (MMC) [8th Sem]

66. Hemodynamics of VSD (BSMC) [8th Sem]

67. Age independent Marker of Growth (BSMC) [8th Sem]

68. Management of Edema in Nephritic Syndrome (BSMC) [8th Sem]

69. Causes of Neonatal Hyperbilirubinemia (BSMC) [8th Sem]

70. Complication of PEM (BSMC) [8th Sem]

71. Signs of Severe Dehydration (BSMC) [8th Sem]

16
COMPILED AND EDITED BY MEHBOOB-UL ISLAM
6
QUESTION BANK

72. Causes of Neonatal Jaundice (MMC) [8th Sem]

73. Diagnosis of Hypothyroidism in infant (MMC) [8th Sem]

74. Rheumatic Carditis (MMC) [8th Sem]

75. Clinical Features of Rickets (MMC) [8th Sem]

76. Clinical Features of Acute Post Streptococcal Glomerulonephritis (MMC) [8th Sem]

77. Growth Chart (MMC) [8th Sem]

78. Febrile Convulsion (NBMC) [8th Sem], (NBMC) [9th Sem]

79. Chelation Therapy in Thalassemia (NBMC) [8th Sem]

80. Clinical Feature of Tetanus (NBMC) [8th Sem]

81. Physiological Jaundice in New born (NBMC) [8th Sem]

82. Congenital Diaphragmatic Hernia (NBMC) [8th Sem]

83. Hypothermia of New born (NBMC) [8th Sem]

84. Management of Acute Bronchiolitis (Kalyani) [8th Sem]

85. Diagnosis of Rheumatic Fever (Kalyani) [8th Sem]

86. Clinical Manifestation of Fe-deficiency Anemia in Children (Kalyani) [8th Sem]

87. Complication of Mumps (Kalyani) [8th Sem]

88. Oral Rehydration Therapy (Kalyani) [8th Sem]

89. Prevention of Vitamin –A deficiency (Kalyani) [8th Sem]

90. Investigation in a case of Suspected Nephrotic Syndrome (Kalyani) [8th Sem]

91. Management of Neonatal Sepsis (BSMC) [9th sem]

92. Respiratory distress syndrome (Hyaline Membrane Disease) (BSMC) [9th sem]

93. Complication of low birth weight newborn (BSMC) [9th sem]

94. Age independent criteria of malnutrition (BSMC) [9th sem]

95. Liver function tests (BSMC) [9th sem]

96. Management of ITP (BSMC) [9th sem], (NBMC) [9th Sem]

97. Status Epilepticus (BSMC) [9th sem]

16
COMPILED AND EDITED BY MEHBOOB-UL ISLAM
7
QUESTION BANK

98. When will you diagnose neonatal jaundice to be pathological? Write characteristics

of breast milk jaundice. (2+3) (RG Kar) [9th sem]

99. How will you take care of umbilical stump in a newborn baby? Mention the causes of

delayed fall of umbilical cord. (3+2) (RG Kar) [9th sem]

100. Mention the steps of prevention of hypothermia in a new born. Which babies

do particularly need additional heat source? (4+1) (RG Kar) [9th sem]

101. Primary Complex (RG Kar) [9th sem]

102. Kwashiorkor dermatitis (RG Kar) [9th sem]

103. Simple febrile seizure (RG Kar) [9th sem]

104. Modified Jone’s criteria (RG Kar) [9th sem]

105. Steps of management of severe acute malnutrition (NRS) [9th sem]

106. Organ involvement in perinatal asphyxia (NRS) [9th sem]

107. Developmental milestones in 3 yrs old child (NRS) [9th sem]

108. Neonatal hypoglycemia (NRS) [9th sem], (NBMC) [9th Sem]

109. Initial steps of resuscitation (NRS) [9th sem]

110. Hyponatremia (NRS) [9th sem]

111. Signs of vitamin D deficiency (NRS) [9th sem]

112. Treatment of neonatal seizure (IPGMER) [9th sem]

113. Cold sress in newborn (IPGMER) [9th sem]

114. Complication of preterm babies (IPGMER) [9th sem]

115. Prophylaxis of rheumatic fever (IPGMER) [9th sem]

116. Tumor lysis syndrome (IPGMER) [9th sem]

117. Assessment of asthma control (IPGMER) [9th sem]

118. Complication of bacterial meningitis in a child (IPGMER) [9th sem]

119. Differential diagnosis of ascites in children (CNMC) [9th Sem]

120. Clinical manifestations of ALL in children (CNMC) [9th Sem]

121. Management of acute severe asthma in children (CNMC) [9th Sem]

122. Clinical features of Dengue fever (CNMC) [9th Sem]

16
COMPILED AND EDITED BY MEHBOOB-UL ISLAM
8
QUESTION BANK

123. Hepatitis infection in children(CNMC) [9th Sem]

124. Clinical manifestations of tubercular meningitis (CNMC) [9th Sem]

125. BCG vaccination (CNMC) [9th Sem]

126. Management of severe dehydration (CMC) [9th Sem]

127. Clinical features of severe pneumonia (CMC) [9th Sem]

128. Thermogenesis in newborn babies. (CMC) [9th Sem]

129. Developmental milestones at 2 yrs of age (CMC) [9th Sem]

130. D/d of microcytic hypochromic anemia (CMC) [9th Sem]

131. Diagnosis of acute idiopathic thrombocytopenic purpura (CMC) [9th Sem]

132. Write briefly M/m of shock in children (KPC) [9th Sem]

133. M/m of consestive cardiac failure in children (KPC) [9th Sem]

134. Etiology and relevant investigation in a 2 yr old febrile child with convulsions

(KPC) [9th Sem]

135. Oral rehydration therapy (KPC) [9th Sem]

136. Etiology of hypothermia in a newborn (KPC) [9th Sem]

137. Investigation in a 2 yr old child with urinary tract infection (KPC) [9th Sem]

138. Vit- D deficiency Rickets (KPC) [9th Sem]

139. M/m of acute rheumatic fever (MMC) [9th Sem]

140. C/f of dengue in children (MMC) [9th Sem]

141. Febrile seizure (MMC) [9th Sem]

142. Staging of bronchial asthma in children (MMC) [9th Sem]

143. C/f of Chronic hemolytic anemia (MMC) [9th Sem]

144. Causes of jaundice in a new born child (MMC) [9th Sem]

145. Kangaroo mother care (NBMC) [9th Sem], (Kalyani) [9th Sem]

146. M/m of 1st episode of Nephrotic Syndrome (NBMC) [9th Sem]

147. Persistence of Neonatal hunconjugated hyperbilirubinemia (Kalyani) [9th

Sem]

148. Prevention of Rabies (Kalyani) [9th Sem]

16
COMPILED AND EDITED BY MEHBOOB-UL ISLAM
9
QUESTION BANK

149. Cretinism (Kalyani) [9th Sem]

150. Hemodynamics of PDA (Kalyani) [9th Sem]

151. Steps of M/m of SAM (Kalyani) [9th Sem]

17
COMPILED AND EDITED BY MEHBOOB-UL ISLAM
0
QUESTION BANK

LONG QUESTIONS
Gr- D
1. A 2 year old male child presented with sudden onset wheeze with altered
sensorium. What are the differential diagnoses? (MC,K) [8th Sem]

2. A seven year old girl presents with convulsion in triage room. She had pedal oedema,
decreased urinary output. What is your diagnosis? What are the other complication
of the disease? What relevant investigation you want to perform to arrive at your
diagnosis? (2+3+3) (NRS) [8th Sem]

3. A 2 year old child admitted with history of fever and cough for last 5 days and
developed respiratory distress for last 2 days. On examination tachypnea and chest
indrawing are present. What is the most probable diagnoses? How do you manage
the case? (1+7) (CNMC) [8th Sem]

4. A 10year old boy presented in the emergency with high grade fever and vomiting
for 2 days. The child had one episode of Generalised Tonic Clonic Convulsion
followed by unconsciousness. Enumerate the etiology and management of this case?
(4+4) (CNMC) [8th Sem]

5. A 9 month old infant presented in the children ward with acute respiratory
distress. Mention the common causes. How will you proceed to manage the case?
(10+12) (RG Kar) [8th Sem]

6. A 7 year old boy havibg jaundice for 6 months with hepatomegaly, admitted for
massive haematemesis. Mention the diagnostic possibilities. Write how will you
evaluate such a case to confirm the diagnosis. (2+6) (IPGMER) [8th Sem]

7. A 2 year old baby of a diabetic mother weighing 4 kg was brought to NICU with
active convulsions. Mother had history of Prolonged rupture of membrane and the
baby was born by Emergency Caesarean section. Enumerate the probable cause.
How will you manage the case. (2+6) (KPC) [8th Sem]

8. A child aged 1 year presented with cough and cold. Write the differential
diagnoses. Suggest the investigations to reach the final diagnosis. ( 3+5) (BMC) [8th
Sem]
17
COMPILED AND EDITED BY MEHBOOB-UL ISLAM
1
QUESTION BANK

9. A 4 year old boy presented with anasarca with history of similar illness 6 months
back. Mention D/D and how to proceed to the diagnosis. (2+6) (BMC) [9th Sem]

10. A 6 yaer old girl presented with fever, migrating joint pain for last 5 days. She had
a past history of sore throat. What is your provisional diagnosis and how will you
investigate and mange the case? (12) (BSMC) [8th Sem]

11. 5 year old boy presented with oliguria and cola coloured urine with edema. How will
you approach this case? (8) (BSMC) [8th Sem]

12. An 8 year old boy presents with fever, epistaxis, severe pallor, generalized
lymphadenopathy and hepatosplenomegaly. What may be the most likely diagnosis?
Give the D/D.Investigate the case to arrive at diagnosis. (1+2+5) (MMC) [8th Sem]

13. A nine year old girl presented with mild pallor, multiple purpuric spots over
different parts of the body. Clinical examination did not reveal any significant
lymphadenopathy, hepatosplenomegaly. How do you approach the case for diagnosis.
(NBMC) [8th Sem]

14. Discuss the process of bilirubin metabolism. One 3 day old baby presented with
jaundice extending up to the sole. How will you approach for the diagnosis and
management? (3+7) (kalyani) [8th Sem]

15. A 7 year old boy presented with the swelling of the whole body starting in the face
and scanty reddish urine for last 3 days. There is h/o few pustules of the leg 3
weeks back. What is your provisional diagnosis? What investigation would you like to
do? Name the dreaded complication of the disease. Briefly outline the management
of the patient. (1+2+2+3) (BSMC) [9th sem]

16. An 8-month old baby has presented with acute watery diarrhea. He has been
diagnosed to have severe dehydration. Write the clinical features that suggested
dehydration. Give an outline of rehydration therapy for this child. (3+5) (RG Kar)
[9th sem]

17. A 7 yr old girl presents with convulsion in triage room. She has pedal oedema,
decreased urine output. What is your diagnosis? What are the other complication
of then disease? What relevant investigation will you want to perform to arrive at
your diagnosis? (2+3+3) (NRS) [9th sem]

18. A 2 yr old girl presented with loose motion and cough for last 7 days. Her body
weight is 7 kg, height 87 cm and mid upper arm circumference (MUAC) 11 cm. what

17
COMPILED AND EDITED BY MEHBOOB-UL ISLAM
2
QUESTION BANK

is the provisional diagnosis. Outline the management of this child. (2+6) (IPGMER)
[9th sem]

19. A 10 month old baby having high fever since morning and developed generalized
tonic clonic seizure persisting for about 5 minutes and became well alert after that
episode. What is the most possible diagnosis? How do you manage the case? (1+7)
(CNMC) [9th Sem]

20. A 10 m male child presented at emergency with high grade fever for last 2 days and
sudden onset generalized tonic clonic seizure for last 10 minutes. What are the
differential diagnoses? Describe the CSF picture of different condition and
management of Pyogenic meningitis. (2+4+4) (CMC) [9th Sem]

21. A 4 year old child is admitted with severe respiratory distress, pallor and
generalized lymphadenopathy. Enumerate the possibilities. How will you investigate
such a case? (3+5) (KPC) [9th Sem]

22. A 4 yr old child presents with high grade fever, running nose, red eyes followed by
maculo-papular rash on face and rest of the body 4 days later. What is the most
likely diagnosis? Outline the differential diagnoses, complication, prevention and
M/m of the case. (1+2+2+1+2) (MMC) [9th Sem]

23. A 3 yr old child presented in emergency with history of fever and cough for 3 days
and respiratory distress for 1 day. Enumerate differential diagnoses. How do you
approach such a case for diagnosis. (3+5) (NBMC) [9th Sem]

24. A 1½ yr old child presented with high fever and generalized convulsion. How will you
approach to diagnosis and M/m. (4+4) (Kalyani) [9th Sem]

17
COMPILED AND EDITED BY MEHBOOB-UL ISLAM
3

You might also like